OB 2 chungus

Lakukan tugas rumah & ujian kamu dengan baik sekarang menggunakan Quizwiz!

While assessing a young female client for reproductive disorders, a nurse understands that the client has amenorrhea. Which specific question should the nurse ask the client? "Do you have frequent vaginal infections?" "Do you have any thyroid gland disorders?" "Do you have diabetes mellitus?" "Do you have continuous or intermittent pelvic pain?"

"Do you have any thyroid gland disorders?" A female client with a thyroid gland disorder may experience the absence of a menstrual cycle, which is known as amenorrhea.

A 14-year-old female client with constitutional growth delay tells a nurse, "I am not getting any taller! I am the same height I was last year!" How should the nurse respond? "I will refer you to an endocrinologist." "You should start taking growth supplements." "You will reach full height by adulthood." "I am afraid you will not grow any taller."

"You will reach full height by adulthood." Constitutional growth delay is also called delayed puberty. Normal stature and sexual maturity is reached by adulthood. Therefore, this statement made by the nurse is the most appropriate response to the client.

A woman in the active phase of the first stage of labor is using a shallow pattern of breathing, which is approximately twice the normal adult breathing rate. She starts to complain about feeling lightheaded and dizzy and states that her fingers are tingling. Which intervention should the nurse immediately initiate? a.Contact the woman's physician. b.Tell the woman to slow her pace of her breathing. c.Administer oxygen via a mask or nasal cannula. d.Help her breathe into a paper bag.

ANS: D This woman is experiencing the side effects of hyperventilation, which include the symptoms of lightheadedness, dizziness, tingling of the fingers, or circumoral numbness. Having the woman breathe into a paper bag held tightly around her mouth and nose may eliminate respiratory alkalosis and enable her to rebreathe carbon dioxide and replace the bicarbonate ion.

20. The nurse is palpating the abdomen of a woman who is 35 weeks pregnant and notices that the fetal head is facing downward toward the pelvis. The nurse would document this as fetal: A. Lie. B. Variety. C. Attitude. D. Presentation.

ANS: D Fetal presentation describes the part of the fetus that is entering the pelvis first. Fetal lie is orientation of the fetal spine to the maternal spine. Attitude is the position of fetal parts in relation to each other, and fetal variety is the location of the fetal back to the maternal pelvis.

8. A woman in her second trimester of pregnancy complains of heartburn and indigestion. When discussing this with the woman, the nurse considers which explanation for these problems? A. Tone and motility of the gastrointestinal tract increase during the second trimester. B. Sluggish emptying of the gallbladder, resulting from the effects of progesterone, often causes heartburn. C. Lower blood pressure at this time decreases blood flow to the stomach and gastrointestinal tract. D. The enlarging uterus and altered esophageal sphincter tone predispose the woman to have heartburn.

ANS: D Stomach displacement from the enlarging uterus plus altered esophageal sphincter and gastric tone as a result of progesterone predispose the woman to heartburn. The tone and motility of the gastrointestinal tract are decreased, not increased, during pregnancy. Emptying of the gallbladder may become more sluggish during pregnancy, but it is not related to indigestion. Instead, some women are predisposed to gallstone formation. A lower blood pressure may occur during the second semester, but it does not affect digestion.

A nurse observes that the endometrial lining is absent is a client who underwent a dilation and curettage (D&C) abortion. Which syndrome will the nurse suspect in the client? Turner syndrome Sheehan's syndrome Asherman's syndrome Mayer-Rokitansky-Hauser syndrome

Asherman's syndrome Asherman's syndrome is characterized by the lack of uterine endometrial lining. It is mostly observed after D&C surgery and can be a cause of secondary amenorrhea.

Which of the following statements about the prevalence of perinatal mental health problems is true? (Select all that apply.) a. The World Health Organization (WHO) recognizes postpartum depression as the leading cause of disability in women. b. Up to a ¼ of pregnant women will experience some aspect of depression during their pregnancies. c. Income status plays a significant role in the presentation of perinatal mental health problems. d. A higher incidence of postpartum depression (PPD) is found in about 50% of the population. e. Between 30 and 50 billion dollars accounts for productivity and direct medical costs related to depression in women.

BCE

Which physiological changes occur in a female during the perimenopausal stage? Decreased estrogen secretion Increased follicle-stimulating hormone (FSH) level Both A and B None of the above

Both A and B During the perimenopausal stage, the ovaries start degenerating and thus estrogen secretion is decreased. The pituitary gland secretes high amount of FSH in an attempt to increase estrogen secretion leading to increased FSH level. Therefore, both A and B are correct answers.

A 23-year-old sexually active female presents with white copious discharge and itch and is diagnosed with yeast vaginitis. This condition is caused by overgrowth of which microorganism? a. Escherichia coli b. Lactobacillus acidophilus c. Candida albicans d. Neisseria gonorrhoeae

C. Candida albicans The most common cause of yeast vaginitis is Candida albicans. Changes in the vaginal pH may predisposed

Which laboratory test can assess a client's blood to rule out the possibility of ovarian cancer? Estrogen Urine calcium oxalate Carcinoembryonic antigen 125 (CA-125) Follicle-stimulating hormone (FSH)

Carcinoembryonic antigen 125 (CA-125) CA-125 is a biomarker that is released into the bloodstream of clients with ovarian cancer. Therefore, it helps in ruling out the possibility of ovarian cancer.

13. What is the most dangerous effect on the fetus of a mother who smokes cigarettes while pregnant? a. Genetic changes and anomalies b. Extensive CNS damage c. Fetal addiction to the substance inhaled d. Intrauterine growth restriction

D The major consequences of smoking tobacco during pregnancy are low-birth-weight infants, prematurity, and increased perinatal loss. Cigarettes will not normally cause genetic changes or extensive CNS damage. Addiction to tobacco is not a usual concern related to the neonate.

26. The nurse is evaluating the EFM tracing of the client who is in active labor. Suddenly, the FHR drops from its baseline of 125 down to 80 beats per minute. The mother is repositioned, and the nurse provides oxygen, increased IV fluids, and performs a vaginal examination. The cervix has not changed. Five minutes have passed, and the FHR remains in the 80s. What additional nursing measures should the nurse take next? a. Call for help. b. Insert a Foley catheter. c. Start administering Pitocin. d. Immediately notify the care provider.

D (To relieve an FHR deceleration, the nurse can reposition the mother, increase IV fluids, and provide oxygen. If oxytocin is infusing, then it should be discontinued. If the FHR does not resolve, then the primary care provider should be immediately notified. Inserting a Foley catheter is an inappropriate nursing action. If the FHR were to continue in a nonreassuring pattern, then a cesarean section could be warranted, which would require a Foley catheter. However, the physician must make that determination. The administration of Pitocin may place additional stress on the fetus.)

Prioritize the steps involved in menstrual cycle (A-F). (Enter the letter of each step in the proper sequence; do not use commas or spaces.) A. Ovulation B. Production of progesterone C. Formation of the corpus luteum D. Release of luteinizing hormone E. Increased follicle-stimulating hormone (FSH) F. Inhibition of FSH and luteinizing hormone

E. Increased follicle-stimulating hormone (FSH) D. Release of luteinizing hormone A. Ovulation C. Formation of the corpus luteum B. Production of progesterone F. Inhibition of FSH and luteinizing hormone

What is an indicator for performing a contraction stress test? D. Increased fetal movement and small for gestational age E. Maternal diabetes mellitus and postmaturity F. Adolescent pregnancy and poor prenatal care G. History of preterm labor and intrauterine growth restriction

E. Maternal diabetes mellitus and postmaturity Rationale:Decreased fetal movement is an indicator for performing a contraction stress test; the size (small for gestational age) is not an indicator. Maternal diabetes mellitus and postmaturity are two indications for performing a contraction stress test. Although adolescent pregnancy and poor prenatal care are risk factors of poor fetal outcomes, they are not indicators for performing a contraction stress test. Intrauterine growth restriction is an indicator; but history of a previous stillbirth, not preterm labor, is the other indicator.

A female client is diagnosed to be infertile due to the production of antibodies against the ovarian tissue. Which treatment will the nurse expect for the client? Estrogen replacement therapy Oral contraceptive therapy Clomiphene citrate Dilatation and curettage surgery

Estrogen replacement therapy Infertility caused due to the production of antibodies against the ovarian tissue is known as premature ovarian failure (POF). This leads to loss of ova and decreased estrogen production. Therefore, the nurse expects estrogen replacement therapy for the client.

A female client tells a nurse, "I have excessive bleeding that lasts for more than 7 days every time I menstruate." Which menstrual disorder does the nurse suspect in the client? Menorrhagia Metrorrhagia Oligomenorrhea Secondary amenorrhea

Menorrhagia Menorrhagia is a menstrual disorder in which a client will experience excessive uterine bleeding and the menstrual cycle usually lasts for more than 7 days. Therefore, the nurse suspects menorrhagia in the client.

In a client with a uterine disorder, a nurse observes that the endometrial tissue grows inside the muscular layer of the uterus. Which complications will the nurse observe in the client? Select all that apply. Menorrhagia Infertility Pelvic pain Dysmenorrhea Dyspareunia

Menorrhagia Pelvic pain Dysmenorrhea Dyspareunia

The nurse assisting a laboring patient is aware that the birth of the fetus is imminent. What is the station of the presenting part? A. -1 B. +1 C. +3 D. +5

Station is the relationship of the presenting fetal part to an imaginary line drawn between the maternal ischial spines. The placement of the presenting part is measured in centimeters above or below the ischial spines. Birth is imminent when the presenting part is at +4 to +5 cm. When the lowermost portion of the presenting part is 1 cm above the spine, it is noted as minus (-)1. When the presenting part is 1 cm below the spine, the station is said to be plus (+)1. At +3, the presenting part is still descending the birth canal. Birth is imminent when the presenting part is at +4 to +5 cm.

A nurse is caring for a 19-year-old female client who has never had a menstrual cycle and has a webbed neck, short stature, and a broad shield-like chest. What is the most probable cause of the client's condition? Turner syndrome Sheehan's syndromeAsherman's syndrome Mayer-Rokitansky-Hauser syndrome

Turner syndrome If a 19-year-old female has never had a menstrual cycle, the client would have primary amenorrhea. The most probable cause of primary amenorrhea is Turner syndrome, which is characterized by a webbed neck, short stature, and a broad shield-like chest.

3. The nurse sees a woman for the first time when she is 30 weeks pregnant. The woman has smoked throughout the pregnancy, and fundal height measurements now are suggestive of growth restriction in the fetus. In addition to ultrasound to measure fetal size, what other tool would be useful in confirming the diagnosis? a. Doppler blood flow analysis b. Contraction stress test (CST) c. Amniocentesis d. Daily fetal movement counts

a. Doppler blood flow analysis

16. Compared with contraction stress test (CST), nonstress test (NST) for antepartum fetal assessment: a. Has no known contraindications. b. Has fewer false-positive results. c. Is more sensitive in detecting fetal compromise. d. Is slightly more expensive.

a. Has no known contraindications.

7. Maternal serum alpha-fetoprotein (MSAFP) screening indicates an elevated level. MSAFP screening is repeated and again is reported as higher than normal. What would be the next step in the assessment sequence to determine the well-being of the fetus? a. Percutaneous umbilical blood sampling (PUBS) b. Ultrasound for fetal anomalies c. Biophysical profile (BPP) for fetal well-being d. Amniocentesis for genetic anomalies

b. Ultrasound for fetal anomalies

A client who is scheduled for a hysteroscopy asks a nurse about the procedure involved in this technique. Which statement made by the client indicates effective understanding? "A radiopaque dye is used during a hysteroscopy." "A scope is inserted into the uterus to take a small amount of tissue." "A scope is inserted through a small surgical incision in the abdomen." "A thin telescopic type device is inserted via the vagina into the uterus."

"A thin telescopic type device is inserted via the vagina into the uterus."

A registered nurse is teaching a client with dysmenorrhea about her treatment plan. Which statement made by the client indicates the need for further teaching? "I should avoid the use of oral contraceptives." "I should take ibuprofen as prescribed." "I should consume a low-fat diet." "I should perform regular exercise."

"I should avoid the use of oral contraceptives." Oral contraceptives stop ovulation and decrease prostaglandin synthesis and myometrial contractility. This helps in relieving the symptoms of dysmenorrhea. They should not be avoided in clients with dysmenorrhea.

A pregnant patient reports abdominal pain in the right lower quadrant, along with nausea and vomiting. The patient's urinalysis report shows an absence of any urinary tract infection in the patient. A chest x-ray also rules out lower-lobe pneumonia. Which condition does the nurse suspect in the patient? 1 Appendicitis 2 Cholelithiasis 3 Placenta previa 4 Uterine rupture

1 Abdominal pain in the right lower quadrant, accompanied by nausea and vomiting, indicates appendicitis in a pregnant patient. Cholelithiasis is characterized by right upper quadrant pain. Placenta previa is a condition wherein the placenta is implanted in the lower uterine segment covering the cervix, which causes bleeding when the cervix dilates. Uterine rupture is seen in a pregnant patient as a result of trauma, which may cause fetal death.

A woman at 37 weeks of gestation is admitted with a placental abruption after a motor vehicle accident. Which assessment data are most indicative of her condition worsening? 1 Pulse (P) 112, respiration (R) 32, blood pressure (BP) 108/60; fetal heart rate (FHR) 166--178 2 P 98, R 22, BP 110/74; FHR 150--162 3 P 88, R 20, BP 114/70; FHR 140--158 4 P 80, R 18, BP 120/78; FHR 138--150

1 Bleeding is the most dangerous problem, which impacts the mother's well-being as well as that of her fetus. The decreasing blood volume would cause increases in pulse and respirations and a decrease in blood pressure. The fetus often responds to decreased oxygenation as a result of bleeding, causing a decrease in perfusion. This causes the fetus' heart rate to increase above the normal range of 120--160 beats per minute. The other options have measurements that are in the "normal" range and would not reflect a deterioration of the patient's physical status. Test-Taking Tip: Do not worry if you select the same numbered answer repeatedly, because there usually is no pattern to the answers.

A woman diagnosed with marginal placenta previa gave birth vaginally 15 minutes ago. At the present time, she is at the greatest risk for: 1 hemorrhage. 2 infection. 3 urinary retention. 4 thrombophlebitis.

1 Hemorrhage is the most immediate risk because the lower uterine segment has limited ability to contract to reduce blood loss. Infection is a risk because of the location of the placental attachment site; however, it is not a priority concern at this time. Placenta previa poses no greater risk for urinary retention than with a normally implanted placenta. There is no greater risk for thrombophlebitis than with a normally implanted placenta.

The quantitative human chorionic gonadotropin (β-hCG) levels are high in a patient who is on methotrexate therapy for dissolving abdominal pregnancy. Which instruction does the nurse give to this patient? 1 "Avoid sexual activity." 2 "Avoid next pregnancy." 3 "Avoid feeling sad and low." 4 "Take folic acid without fail."

1 High β-hCG levels indicate that the abdominal pregnancy is not yet dissolved. Therefore the nurse advises the patient to avoid sexual activity until the β-hCG levels drop and the pregnancy is dissolved completely. If the patient engages in vaginal intercourse, the pelvic pressure may rupture the mass and cause pain. Abdominal pregnancy increases the chances of infertility or recurrent ectopic pregnancy in patients. However, the nurse need not instruct the patient to avoid further pregnancy, because it may increase the feelings of sadness and guilt in the patient. The nurse encourages the patient to share feelings of guilt or sadness related to pregnancy loss. Folic acid is contraindicated with methotrexate therapy, because it may exacerbate ectopic rupture.

A woman with severe preeclampsia has been receiving magnesium sulfate by intravenous infusion for 8 hours. The nurse assesses the woman and documents the following findings: temperature 37.1° C, pulse rate 96 beats/min, respiratory rate 24 breaths/min, blood pressure 155/112 mm Hg, 3+ deep tendon reflexes, and no ankle clonus. The nurse calls the physician, anticipating an order for: 1 hydralazine. 2 magnesium sulfate bolus. 3 diazepam.

1 Hydralazine is an antihypertensive commonly used to treat hypertension in severe preeclampsia. An additional bolus of magnesium sulfate may be ordered for increasing signs of central nervous system irritability related to severe preeclampsia (e.g., clonus) or if eclampsia develops. Diazepam sometimes is used to stop or shorten eclamptic seizures. Calcium gluconate is used as the antidote for magnesium sulfate toxicity. The patient is not currently displaying any signs or symptoms of magnesium toxicity.

A patient with gestational hypertension is prescribed labetalol hydrochloride (Normodyne) therapy, which is continued after giving birth. What does the nurse instruct the patient about breastfeeding? 1 "You may breastfeed the infant if you desire." 2 "Breastfeeding may cause convulsions in the infant." 3 "Breastfeed only once a day and use infant formulas." 4 "There may be high levels of the drug in the breast milk."

1 Labetalol hydrochloride (Normodyne) has a low concentration in breast milk, so the patient can breastfeed the infant. Breastfeeding is safe and will not cause convulsions or any side effects in the infant. Infant formulas are used only if the mother is unable to breastfeed the infant or if the mother does not desire to breastfeed.

After being rehydrated in the emergency department, a 24 year-old primipara in her 18th week of pregnancy is at home and is to rest at home for the next two days and take in small but frequent fluids and food as possible. Discharge teaching at the hospital by the nurse has been effective if the patient makes which statement? 1 "I'm going to eat five to six small servings per day, which contain such foods and fluids as tea, crackers, or a few bites of baked potato." 2 "A strip of bacon and a fried egg will really taste good as long as I eat them slowly." 3 "As long as I eat small amounts and allow enough time for digestion, I can eat almost anything, like barbequed chicken or spaghetti." 4 "I'm going to stay only on clear fluids for the next 24 hours and then add dairy products like eggs and milk."

1 Once the vomiting has stopped, feedings are started in small amounts at frequent intervals. In the beginning, limited amounts of oral fluids and bland foods such as crackers, toast, or baked chicken are offered. Clear fluids alone do not contain enough calories and contain no protein. Most women are able to take nourishment by mouth after several days of treatment. They should be encouraged to eat small, frequent meals and foods that sound appealing (e.g., nongreasy, dry, sweet, and salty foods). Test-Taking Tip: Many times the correct answer is the longest alternative given, but do not count on it. NCLEX item writers (those who write the questions) are also aware of this and attempt to avoid offering you such "helpful hints."

Which condition is seen in a pregnant patient if uterine artery Doppler measurements in the second trimester of pregnancy are abnormal? 1 Preeclampsia 2 HELLP syndrome 3 Molar pregnancy 4 Gestational hypertension

1 Preeclampsia is a condition in which patients develop hypertension and proteinuria after 20 weeks' gestation. It can be diagnosed if uterine artery Doppler measurements in the second trimester of pregnancy are abnormal. HELLP syndrome is characterized by hemolysis (H), elevated liver enzymes (EL), and low platelet count (LP) in a patient with preeclampsia. Molar pregnancy refers to the growth of the placental trophoblast due to abnormal fertilization. Gestational hypertension is a condition in which hypertension develops in a patient after 20 weeks' gestation.

Which condition in a pregnant patient with severe preeclampsia is an indication for administering magnesium sulfate? 1 Seizure activity 2 Renal dysfunction 3 Pulmonary edema 4 Low blood pressure (BP)

1 Severe preeclampsia may cause seizure activity or eclampsia in the patient, which is treated with magnesium sulfate. Magnesium sulfate is not administered for renal dysfunction and can cause magnesium toxicity in the patient. Pulmonary enema can be prevented by restricting the patient's fluid intake to 125 mL/hr. Increasing magnesium toxicity can cause low BP in the patient.

The nurse is caring for a pregnant patient who is receiving antibiotic therapy to treat a urinary tract infection (UTI). Which dietary changes does the nurse suggest for the pregnant patient who is receiving antibiotic therapy for UTI? 1 "Include yogurt, cheese, and milk in your diet." 2 "Avoid folic acid supplements until the end of therapy." 3 "Include vitamins C and E supplementation in your diet." 4 "Reduce your dietary fat intake by 40 to 50 g per day.

1 The antibiotic therapy kills normal flora in the genitourinary tract, as well as pathologic organisms. Therefore the nurse instructs the patient to include yogurt, cheese, and milk in daily diet because they contain active acidophilus cultures. Folic acid should not be avoided, because it may affect the fetal development. Vitamins C and E supplementation is usually included in the diet to treat preeclampsia in a patient. Dietary fat is reduced in patients with cholecystitis or cholelithiasis, because it may cause epigastric pain.

Which instructions does the nurse give to a patient who is prescribed methotrexate therapy for dissolving the tubal pregnancy? 1 "Discontinue folic acid supplements." 2 "Get adequate exposure to sunlight." 3 "Take stronger analgesics for severe pain." 4 "Vaginal intercourse is safe during the therapy."

1 The nurse advises the patient to discontinue folic acid supplements as they interact with methotrexate and may exacerbate ectopic rupture in the patient. Exposure to sunlight is avoided as the therapy makes the patient photosensitive. Analgesics stronger than acetaminophen are avoided, because they may mask symptoms of tubal rupture. Vaginal intercourse is avoided until the pregnancy is dissolved completely.

Which clinical reports does the nurse evaluate to identify ectopic pregnancy in a patient? Select all that apply. 1 Quantitative human chorionic gonadotropin (β-hCG) levels 2 Transvaginal ultrasound 3 Progesterone level 4 Thyroid test reports 5 Kleihauer-Betke (KB) test

1, 2, 3 An ectopic pregnancy is indicated when β-hCG levels are >1500 milli-international units/mL but no intrauterine pregnancy is seen on the transvaginal ultrasound. A transvaginal ultrasound is repeated to verify if the pregnancy is inside the uterus. A progesterone level <5 ng/mL indicates ectopic pregnancy. Thyroid test reports need to be evaluated in case the patient has hyperemesis gravidarum, as hyperthyroidism is associated with this disorder. The KB test is used to determine transplacental hemorrhage.

A pregnant patient in the first trimester reports spotting of blood with the cervical os closed and mild uterine cramping. What does the nurse need to assess? Select all that apply. 1 Progesterone levels 2 Transvaginal ultrasounds 3 Human chorionic gonadotropin (hCG) measurement 4 Blood pressure 5 Kleihauer-Betke (KB) test reports

1, 2, 3 The spotting of blood with the cervical os closed and mild uterine cramping in the first trimester indicates a threatened miscarriage. Therefore the nurse needs to assess progesterone levels, transvaginal ultrasounds, and measurement of hCG to determine whether the fetus is alive and within the uterus. Blood pressure measurements do not help determine the fetal status. KB assay is prescribed to identify fetal-to-maternal bleeding, usually after a trauma.

Which conditions during pregnancy can result in preeclampsia in the patient? Select all that apply. 1 Genetic abnormalities 2 Dietary deficiencies 3 Abnormal trophoblast invasion 4 Cardiovascular changes 5 Maternal hypotension

1, 2, 3, 4 Current theories consider that genetic abnormalities and dietary deficiencies can result in preeclampsia. Abnormal trophoblast invasion causes fetal hypoxia and results in maternal hypertension. Cardiovascular changes stimulate the inflammatory system and result in preeclampsia in the pregnant patient. Maternal hypertension, and not hypotension, after 20 weeks' gestation is known as preeclampsia.

Which hypertensive disorders can occur during pregnancy? Select all that apply. 1 Chronic hypertension 2 Preeclampsia-eclampsia 3 Hyperemesis gravidarum 4 Gestational hypertension 5 Gestational trophoblastic disease

1, 2, 4 Chronic hypertension refers to hypertension that developed in the pregnant patient before 20 weeks' gestation. Preeclampsia refers to hypertension and proteinuria that develops after 20 weeks' gestation. Eclampsia is the onset of seizure activity in a pregnant patient with preeclampsia. Gestational hypertension is the onset of hypertension after 20 weeks' gestation. Gestational trophoblastic disease and hyperemesis gravidarum are not hypertensive disorders. Gestational trophoblastic disease refers to a disorder without a viable fetus that is caused by abnormal fertilization. Hyperemesis gravidarum is excessive vomiting during pregnancy that may result in weight loss and electrolyte imbalance.

What does the nurse assess to detect the presence of a hypertensive disorder in a pregnant patient? Select all that apply. 1 Proteinuria 2 Epigastric pain 3 Placenta previa 4 Presence of edema 5 Blood pressure (BP)

1, 2, 4, 5 Proteinuria indicates hypertension in a pregnant patient. Proteinuria is concentration ≥300 mg/24 hours in a 24-hour urine collection. The nurse needs to assess the patient for epigastric pain because it indicates severe preeclampsia. Hypertension is likely to cause edema or swollen ankles as a result of greater hydrostatic pressure in the lower parts of the body. Therefore the nurse needs to assess the patient for the presence of edema. Accurate measurement of BP will help detect the presence of any hypertensive disorder. A systolic BP greater than 140 mm Hg or a diastolic BP greater than 90 mm Hg will indicate hypertension. Placenta previa is a condition wherein the placenta is implanted in the lower uterine segment covering the cervix, which causes bleeding when the cervix dilates.

A 24-year-old primipara, who is 18 weeks pregnant, has been having increasing vomiting since she was 8 weeks pregnant. Upon arrival at the emergency department, her skin turgor is diminished, temperature is 99.2F (o), pulse is 102, respiration is 18, blood pressure is 102/68, and she has deep furrows on her tongue. What would the nurse expect to do to care for her? Select all that apply. 1 Start an intravenous infusion. 2 Check her urine for ketones 3 Cross match blood for a transfusion. 4 Obtain a complete history. 5 Obtain blood for a complete blood count

1, 2, 4, 5 Whenever a pregnant woman has nausea and vomiting, the first priority is a thorough assessment to determine the severity of the problem. In most cases the woman should be told to come immediately to the health care provider's office or the emergency department because the severity of the illness often is difficult to determine by telephone conversation. The assessment should include frequency, severity, and duration of episodes of nausea and vomiting. If the woman reports vomiting, the assessment also should include the approximate amount and color of the vomitus. The woman is asked to report any precipitating factors relating to the onset of her symptoms. Any pharmacologic or nonpharmacologic treatment measures used should be recorded. Prepregnancy weight and documented weight gain or loss during pregnancy are important to note. The woman's weight and vital signs are measured, and a complete physical examination is performed, with attention to signs of fluid and electrolyte imbalance and nutritional status. The most important initial laboratory test to be obtained is a determination of ketonuria. Other laboratory tests that may be prescribed are a urinalysis, a complete blood cell count, electrolytes, liver enzymes, and bilirubin levels. At this time, there is no supportive evidence that a blood transfusion is required. Based on provided objective data that the patient has deep furrows on her tongue, this may suggest a vitamin B deficiency which should be investigated further.

The nurse is caring for a woman who is at 24 weeks of gestation with suspected severe preeclampsia. Which signs and symptoms should the nurse expect to observe? Select all that apply. 1 Decreased urinary output and irritability 2 Transient headache and +1 proteinuria 3 Ankle clonus and epigastric pain 4 Platelet count of less than 100,000/mm3 and visual problems 5 Seizure activity and hypotension

1, 3, 4 Decreased urinary output and irritability are signs of severe eclampsia. Ankle clonus and epigastric pain are signs of severe eclampsia. Platelet count of less than 100,000/mm3 and visual problems are signs of severe preeclampsia. A transient headache and +1 proteinuria are signs of preeclampsia and should be monitored.Seizure activity and hyperreflexia are signs of eclampsia. Test-Taking Tip: Do not worry if you select the same numbered answer repeatedly, because there usually is no pattern to the answers.

MATCHING Because most pregnant women continue their usual activities, trauma remains a common complication during pregnancy. Approximately 1 in 12 pregnancies in the United States are complicated by trauma each year. As a result of the physiologic alterations that accompany pregnancy, special considerations for the mother and fetus are necessary when trauma occurs. Match the maternal system adaptation in pregnancy with the clinical response to trauma. a. Increased oxygen consumption b. Increased heart rate c. Decreased gastric motility d. Displacement of abdominal viscera e. Increase in clotting factors 1. Decreased placental perfusion in the supine position 2. Increased risk of thrombus formation 3. Altered pain referral 4. Increased risk of acidosis 5. Increased risk of aspiration

1. ANS: B DIF: Cognitive Level: Apply REF: p. 732 2. ANS: E DIF: Cognitive Level: Apply REF: p. 732 3. ANS: D DIF: Cognitive Level: Apply REF: p. 732 4. ANS: A DIF: Cognitive Level: Apply REF: p. 732 5. ANS: C DIF: Cognitive Level: Apply REF: p. 732

The nurse is preparing to teach an antepartum client with GDM the correct method of administering an intermediate-acting insulin, such as neutral protamine Hagedorn (NPH), with a short-acting insulin (regular). In the correct order from 1 through 6, match the step number with the action needed to teach the client self-administration of this combination of insulin. a.Without adding air, withdraw the correct dose of NPH insulin. b.Gently rotate the insulin to mix it, and wipe the stopper. c.Inject air equal to the dose of NPH insulin into the vial, and remove the syringe. d.Inject air equal to the dose of regular insulin into the vial, and withdraw the medication. e.Check the insulin bottles for the expiration date. f.Wash hands. 1. Step 1 2. Step 2 3. Step 3 4. Step 4 5. Step 5 6. Step 6

1. ANS: F DIF: Cognitive Level: Apply REF: p. 694 TOP: Nursing Process: Implementation MSC: Client Needs: Safe and Effective Care Environment NOT: Regular insulin is always drawn up first when combining insulin. Other steps include ensuring that the insulin syringe corresponds to the concentration of insulin that is being used. The bottle should be checked before withdrawing the medication to be certain that it is the appropriate type. 2. ANS: E DIF: Cognitive Level: Apply REF: p. 694 TOP: Nursing Process: Implementation MSC: Client Needs: Safe and Effective Care Environment NOT: Regular insulin is always drawn up first when combining insulin. Other steps include ensuring that the insulin syringe corresponds to the concentration of insulin that is being used. The bottle should be checked before withdrawing the medication to be certain that it is the appropriate type. 3. ANS: B DIF: Cognitive Level: Apply REF: p. 694 TOP: Nursing Process: Implementation MSC: Client Needs: Safe and Effective Care Environment NOT: Regular insulin is always drawn up first when combining insulin. Other steps include ensuring that the insulin syringe corresponds to the concentration of insulin that is being used. The bottle should be checked before withdrawing the medication to be certain that it is the appropriate type. 4. ANS: C DIF: Cognitive Level: Apply REF: p. 694 TOP: Nursing Process: Implementation MSC: Client Needs: Safe and Effective Care Environment NOT: Regular insulin is always drawn up first when combining insulin. Other steps include ensuring that the insulin syringe corresponds to the concentration of insulin that is being used. The bottle should be checked before withdrawing the medication to be certain that it is the appropriate type. 5. ANS: D DIF: Cognitive Level: Apply REF: p. 694

What action does the nurse take to relieve choking in a pregnant patient who is in the third trimester? 1 Administering anesthesia 2 Administering chest thrusts 3 Placing a towel under the hips 4 Positioning the patient onto one side

2 Choking is often relieved in patients by administering abdominal thrusts. However if the patient is in the third trimester of pregnancy, chest thrusts are administered to prevent injury to the uterus. Administering anesthesia or positioning the patient onto one side will not help dislodge the object and relieve choking. The nurse needs to place a towel under the hips to displace the uterus while administering cardiopulmonary resuscitation (CPR).

The nurse observes that a pregnant patient with gestational hypertension who is on magnesium sulfate therapy is prescribed nifedipine (Adalat). What action does the nurse take? 1 Evaluates the patient's renal function test 2 Obtains a prescription for a change of drug 3 Reduces the nifedipine (Adalat) dose by 50% 4 Administers both medications simultaneously

2 Concurrent use of nifedipine (Adalat) and magnesium sulfate can result in skeletal muscle blockade in the patient. Therefore the nurse needs to report immediately to the primary health care provider (PHP) and obtain a prescription for a change of drug. The nurse assesses the patient's renal function to determine the risk for toxicity after administering any drug. However, it is not a priority in this case. Reducing the nifedipine (Adalat) dose is not likely to prevent the drug interaction in the patient. The nurse does not administer both drugs simultaneously because it may be harmful for the patient.

Which is an important nursing intervention when a patient has an incomplete miscarriage with heavy bleeding? 1 Initiate expectant management at once. 2 Prepare the patient for dilation and curettage. 3 Administer the prescribed oxytocin (Pitocin). 4 Obtain a prescription for ergonovine (Methergine).

2 In the case of an incomplete miscarriage, sometimes there is heavy bleeding and excessive cramping and some part of fetal tissue remains in the uterus. Therefore the nurse needs to prepare the patient for dilation and curettage for the removal of the fetal tissue. Expectant management is initiated if the pregnancy continues after a threatened miscarriage. Oxytocin (Pitocin) is administered to prevent hemorrhage after evacuation of the uterus. Ergonovine (Methergine) is administered to contract the uterus.

A woman with severe preeclampsia is being treated with an intravenous infusion of magnesium sulfate. This treatment is considered successful if: 1 blood pressure is reduced to prepregnant baseline. 2 seizures do not occur. 3 deep tendon reflexes become hypotonic. 4 diuresis reduces fluid retention

2 Magnesium sulfate is a central nervous system (CNS) depressant given primarily to prevent seizures . A temporary decrease in blood pressure can occur; however, this is not the purpose of administering this medication. Hypotonia is a sign of an excessive serum level of magnesium. It is critical that calcium gluconate be on hand to counteract the depressant effects of magnesium toxicity. Diuresis is not an expected outcome of magnesium sulfate administration.

The most prevalent clinical manifestation of abruptio placentae (as opposed to placenta previa) is: 1 bleeding. 2 intense abdominal pain. 3 uterine activity. 4 cramping

2 Pain is absent with placenta previa but may be agonizing with abruptio placentae. Bleeding may be present in varying degrees for both placental conditions. Uterine activity may be present with both placental conditions. Cramping is a form of uterine activity that may be present in both placental conditions.

Which intervention does the nurse implement for a patient immediately after a severe abdominal trauma? 1 Prep the patient for cesarean birth. 2 Send the patient for pelvic computed tomography (CT) scanning. 3 Provide fluids to the patient as part of the protocol for ultrasound examination. 4 Prepare to administer Rho(D) immunoglobulin.

2 Pelvic CT scanning helps visualize extraperitoneal and retroperitoneal structures and the genitourinary tract. The nurse needs to prepare the patient for cesarean birth if there is no evidence of a maternal pulse. Ultrasound examination is not as effective as electronic fetal monitoring for determining placental abruption in the patient after the trauma. Therefore the nurse prepares the patient for a CT scan after a severe abdominal trauma. The nurse needs to administer Rho(D) immunoglobulin in an Rh-negative pregnant trauma patient. This helps protect the patient from isoimmunization.

A pregnant patient has a systolic blood pressure that exceeds 160 mm Hg. Which action should the nurse take for this patient? 1 Administer magnesium sulfate intravenously. 2 Obtain a prescription for antihypertensive medications. 3 Restrict intravenous and oral fluids to 125 mL/hr. 4 Monitor fetal heart rate (FHR) and uterine contractions (UCs).

2 Systolic blood pressure exceeding 160 mm Hg indicates severe hypertension in the patient. The nurse should alert the health care provider and obtain a prescription for antihypertensive medications, such as nifedipine (Adalat) and labetalol hydrochloride (Normodyne). Magnesium sulfate would be administered if the patient was experiencing eclamptic seizures. Oral and intravenous fluids are restricted when the patient is at risk for pulmonary edema. Monitoring FHR and UCs is a priority when the patient experiences a trauma so that any complications can be addressed immediately.

A pregnant patient is at risk for cardiac arrest as a result of profound hypovolemia after a trauma. Which action does the nurse take? The nurse: 1 Assesses airway, breathing, and pulse rate. 2 Administers warmed crystalloid solutions. 3 Administers calcium gluconate intravenously. 4 Obtains a prescription for magnesium sulfate.

2 The nurse administers warmed crystalloid solutions for massive fluid resuscitation in the patient who has profound hypovolemia after a trauma. The nurse needs to assess the airway, breathing, and pulse in a patient after a convulsion so that prompt actions can be taken to stabilize the patient. The nurse administers calcium gluconate as an antidote to a patient who has magnesium toxicity. The nurse may administer magnesium sulfate for the treatment of eclamptic seizures in a patient with preeclampsia.

A 24-year-old primipara, 10 weeks pregnant, who has been experiencing vomiting every morning for the past few weeks, asks the nurse at her check-up how long this "morning sickness" will continue. Which statement by the nurse is most accurate? 1 "It will end by the 15th week of pregnancy." 2 "It usually subsides by the 20th week of pregnancy." 3 "It's a very common but not serious problem." 4 "In some women, it can last throughout the pregnancy and become serious."

2 This discomfort of pregnancy usually subsides by the 20th week of pregnancy. An absolute definite end of vomiting during pregnancy can never be stated. Test-Taking Tip: Identifying content and what is being asked about that content is critical to your choosing the correct response. Be alert for words in the stem of the item that are the same or similar in nature to those in one or two of the options. Example: If the item relates to and identifies stroke rehabilitation as its focus and only one of the options contains the word stroke in relation to rehabilitation, you are safe in identifying this choice as the correct response.

A pregnant patient with severe preeclampsia who is being transported to a tertiary care center needs to be administered magnesium sulfate injection for seizure activity. What actions does the nurse take when administering the drug? Select all that apply. 1 A 10-g dose is administered in the buttock. 2 A local anesthetic is added to the solution. 3 The Z-track technique is used to inject the drug. 4 The injection site is massaged after the injection. 5 The subcutaneous route is used to inject the drug.

2, 3, 4 The nurse adds a local anesthetic to the solution to reduce pain that is caused by the injection. The Z-track technique is used to inject the drug so that the drug is injected in the intramuscular (IM) tissue safely. The nurse gently massages the site after administering the injection to reduce pain. The nurse administers two separate injections of 5 g in each buttock. Magnesium sulfate injections are administered in the IM layer and not the subcutaneous layer.

What are the possible causes of miscarriage during early pregnancy? Select all that apply. 1 Premature dilation of cervix 2 Chromosomal abnormalities 3 Endocrine imbalance 4 Hypothyroidism 5 Antiphospholipid antibodies

2, 3, 4, 5 Chromosomal abnormalities account for 50% of all early pregnancy losses. Endocrine imbalance is caused by luteal phase defects, hypothyroidism, and diabetes mellitus in pregnant patients and results in miscarriage. Antiphospholipid antibodies also increase the chances of miscarriage in pregnant patients. Premature dilation of the cervix may cause a second-trimester loss and is usually seen in patients between 12 and 20 weeks' gestation.

What does the nurse include in the plan of care of a pregnant patient with mild preeclampsia? Select all that apply. 1 Ensure prolonged bed rest. 2 Provide diversionary activities. 3 Encourage the intake of more fluids. 4 Restrict sodium and zinc in the diet. 5 Refer to Internet-based support group

2, 3, 5 Activity is restricted in patients with preeclampsia, so it is necessary to provide diversionary activities to such patients to prevent boredom. The nurse encourages the patient to increase fluid intake to enhance renal perfusion and bowel function. The nurse can suggest Internet-based support groups to reduce boredom and stress in the patient. Patients need to restrict activity, but complete bed rest is not advised because it may cause cardiovascular deconditioning, muscle atrophy, and psychological stress. The patient needs to include adequate zinc and sodium in the diet for proper fetal development.

A pregnant woman presents to the emergency department complaining of persistent nausea and vomiting. She is diagnosed with hyperemesis gravidarum. The nurse should include which information when teaching about diet for hyperemesis? Select all that apply. 1 Eat three larger meals a day. 2 Eat a high-protein snack at bedtime. 3 Ice cream may stay down better than other foods. 4 Avoid ginger tea or sweet drinks. 5 Eat what sounds good to you even if your meals are not well-balanced.

2, 3, 5 The diet for hyperemesis includes: (1) Avoid an empty stomach. Eat frequently, at least every 2 to 3 hours. Separate liquids from solids and alternate every 2 to 3 hours. (2) Eat a high-protein snack at bedtime. (3) Eat dry, bland, low-fat, and high-protein foods. Cold foods may be better tolerated than those served at a warm temperature. (4) In general eat what sounds good to you rather than trying to balance your meals. (5) Follow the salty and sweet approach; even so-called junk foods are okay. (6) Eat protein after sweets. (7) Dairy products may stay down more easily than other foods. (8) If you vomit even when your stomach is empty, try sucking on a Popsicle. (9) Try ginger tea. Peel and finely dice a knuckle-sized piece of ginger and place it in a mug of boiling water. Steep for 5 to 8 minutes and add brown sugar to taste. (10) Try warm ginger ale (with sugar, not artificial sweetener) or water with a slice of lemon. (11) Drink liquids from a cup with a lid.

A pregnant patient with chronic hypertension is at risk for placental abruption. Which symptoms of abruption does the nurse instruct the patient to be alert for? Select all that apply. 1 Weight loss 2 Abdominal pain 3 Vaginal bleeding 4 Shortness of breath 5 Uterine tenderness

2, 3, 5 The nurse instructs the pregnant patient to be alert for abdominal pain, vaginal bleeding, and uterine tenderness as these indicates placental abruption. Weight loss indicates fluid and electrolyte loss and not placental abruption. Shortness of breath indicates inadequate oxygen, which is usually seen in a patient who is having cardiac arrest.

The nurse is preparing to discharge a 30-year-old woman who has experienced a miscarriage at 10 weeks of gestation. Which statement by the woman indicates a correct understanding of the discharge instructions? 1 "I will not experience mood swings since I was only at 10 weeks of gestation." 2 "I will avoid sexual intercourse for 6 weeks and pregnancy for 6 months." 3 "I should eat foods that are high in iron and protein to help my body heal." 4 "I should expect the bleeding to be heavy and bright red for at least 1 week."

3 A woman who has experienced a miscarriage should be advised to eat foods that are high in iron and protein to help replenish her body after the loss. After a miscarriage, a woman may experience mood swings and depression from the reduction of hormones and the grieving process. Sexual intercourse should be avoided for 2 weeks or until the bleeding has stopped and should avoid pregnancy for 2 months. The woman should not experience bright red, heavy, profuse bleeding; this should be reported to the health care provider.

The nurse observes that maternal hypotension has decreased uterine and fetal perfusion in a pregnant patient. What does the nurse need to assess further to understand the maternal status? 1 D-dimer blood test 2 Kleihauer-Betke (KB) test 3 Electronic fetal monitoring 4 Electrocardiogram reading

3 Electronic fetal monitoring reflects fetal cardiac responses to hypoxia and hypoperfusion and helps to assess maternal status after a trauma. The D-dimer blood test is used to rule out the presence of a thrombus. The KB test is used to evaluate transplacental hemorrhage. Electrocardiogram reading is more useful to assess the cardiac functions in nonpregnant cardiac patients.

At 37 weeks of gestation, the patient is in a severe automobile crash where her abdomen was hit by the steering wheel and her seat belt. What actions would the emergency room nurse expect to perform upon the patient's arrival at the hospital? 1 Stay with the patient, assure a patent airway is present, and keep the patient as calm as possible. 2 Move the patient's skirt to determine if any vaginal bleeding is present, find out who to call, and monitor the level of consciousness. 3 Assess the patient's vital signs, determine location and severity of pain, and establish continual fetal heart rate monitoring. 4 Obtain arterial blood gases, obtain a hemoglobin and hematocrit, and oxygen saturation rate.

3 Full assessment of the patient and her fetus are essential and include vital signs, continual fetal heart rate monitoring, determining the location and severity of pain, whether any vaginal bleeding is dark red or bright red, and the status of the abdomen, which would be expected to be rigid or "board like." Staying with the patient, assuring a patent airway is present, and keeping the patient as calm as possible would be appropriate at the crash site before the arrival of emergency medical services (EMS). The current status of the patient and fetus are thepriority. The health care provider would prescribe the arterial blood gases and other laboratory work after the patient is assessed and stabilized.

What does the nurse advise a pregnant patient who is prescribed phenazopyridine (Pyridium) for cystitis? 1 "Avoid sweet foods in diet." 2 "Limit exposure to sunlight." 3 "Do not wear contact lenses." 4 "Restrict oral fluids to 125 mL per hour."

3 Phenazopyridine (Pyridium) colors the tears orange. Therefore the nurse instructs the patient to avoid wearing contact lenses. Sweet foods are avoided in patients with diabetes mellitus, because they can cause fluctuating glucose levels, which may harm the fetus. Exposure to sunlight is avoided when the patient is receiving methotrexate therapy, because it causes photosensitivity. Oral fluids are restricted in patients who are at risk for pulmonary edema.

What instruction does the nurse provide to a pregnant patient with mild preeclampsia? 1 "You need to be hospitalized for fetal evaluation." 2 "Nonstress testing can be done once every month." 3 "Fetal movement counts need to be evaluated daily." 4 "Take complete bed rest during the entire pregnancy."

3 Preeclampsia can affect the fetus and may cause fetal growth restrictions, decreased amniotic fluid volume, abnormal fetal oxygenation, low birth weight, and preterm birth. Therefore the fetal movements need to be evaluated daily. Patients with mild preeclampsia can be managed at home effectively and need not be hospitalized. Nonstress testing is performed once or twice per week to determine fetal well-being. Patients need to restrict activity, but complete bed rest is not advised because it may cause cardiovascular deconditioning, muscle atrophy, and psychological stress.

Which intervention will help prevent the risk of pulmonary edema in a pregnant patient with severe preeclampsia? 1 Assess fetal heart rate (FHR) abnormalities regularly. 2 Place the patient on bed rest in a darkened environment. 3 Restrict total intravenous (I.V.) and oral fluids to 125 mL/hr. 4 Ensure that magnesium sulfate is administered as prescribed.

3 Pulmonary edema may be seen in patients with severe preeclampsia. Therefore the nurse needs to restrict total intravenous (I.V.) and oral fluids to 125 mL/hr. FHR monitoring helps assess any fetal complications. The patient is placed on bed rest in a darkened environment to prevent stress. Magnesium sulfate is administered to prevent eclamptic seizures.

Signs of a threatened abortion (miscarriage) are noted in a woman at 8 weeks of gestation. What is an appropriate management approach for this type of abortion? 1 Prepare the woman for a dilation and curettage (D&C). 2 Place the woman on bed rest for at least 1 week and reevaluate. 3 Prepare the woman for an ultrasound and bloodwork. 4 Comfort the woman by telling her that if she loses this baby, she may attempt to get pregnant again in 1 month.

3 Repetitive transvaginal ultrasounds and measurement of human chorionic gonadotropin (hCG) and progesterone levels may be performed to determine if the fetus is alive and within the uterus. If the pregnancy is lost , the woman should be guided through the grieving process. D&C is not considered until signs of the progress to an inevitable abortion are noted or the contents are expelled and incomplete. Bed rest is recommended for 48 hours initially. Telling the woman that she can get pregnant again soon is not a therapeutic response because it discounts the importance of this pregnancy.

A labor and delivery nurse is in the process of admitting a patient who is 39 and at 5 weeks' gestation with a diagnosis of preeclampsia. The nurse has evaluated vital signs, weight, and deep tendon reflexes. Although the presence of edema is no longer included in the definition of preeclampsia, it is an important component of the nurse's evaluation. Edema is assessed for distribution, degree and pitting. Although the amount of edema is difficult to quantify, it is important to record the relative degrees of edema formation. From the graphic below, please select the illustration that best displays +3 edema. 1. A 2. B 3. C 4. D

3 The graphic illustrates a depth of 6 mm when the nurse applies finger pressure to the swollen area. This measurement indicates a +3 measurement for edema. Upon applying finger pressure, a 2 mm depression would be the equivalent of a +1, a 4 mm depression, a +2, and an 8 mm depression a +4.

A blunt abdominal trauma causes fetal hemorrhage in a pregnant patient. The nurse finds that the patient is Rh negative. What action does the nurse take? 1 Initiate magnesium sulfate per protocol. 2 Administer oxytocin (pitocin). 3 Administer prescribed Rho (D) immunoglobulin. 4 Prepare the patient for magnetic resonance imaging (MRI).

3 The nurse administers the prescribed Rho(D) immunoglobulin to the patient to protect the patient from isoimmunization. The nurse needs to obtain a prescription for magnesium sulfate if there are eclamptic seizures in a patient with preeclampsia. Oxytocin (Pitocin) is administered to prevent bleeding after birth or the evacuation of the uterus. Magnetic resonance imaging (MRI) is used to assess injuries in a patient after trauma.

What does the nurse administer to a patient if there is excessive bleeding after suction curettage? 1 Nifedipine (Procardia) 2 Methyldopa (Aldomet) 3 Hydralazine (Apresoline) 4 Ergonovine (Methergine)

4 Ergonovine (Methergine) is an ergot product, which is administered to contract the uterus when there is excessive bleeding after suction curettage. Nifedipine (Procardia) is prescribed for gestational hypertension or severe preeclampsia. Methyldopa (Aldomet) is an antihypertensive medication indicated for pregnant patients with hypertension. Hydralazine (Apresoline) is also an antihypertensive medication used for treating hypertension intrapartum.

A patient reports excessive vomiting in the first trimester of the pregnancy, which has resulted in nutritional deficiency and weight loss. The urinalysis report of the patient indicates ketonuria. Which disorder does the patient have? 1 Preeclampsia 2 Hyperthyroid disorder 3 Gestational hypertension 4 Hyperemesis gravidarum

4 Hyperemesis gravidarum is characterized by excessive vomiting during pregnancy, which causes nutritional deficiency and weight loss. The presence of ketonuria is another indication of this disorder. Preeclampsia refers to hypertension and proteinuria in patients after 20 weeks' gestation. Hyperthyroid disorder may be one of the causes of hyperemesis gravidarum. Gestational hypertension also develops after 20 weeks' gestation.

Which fetal risk is associated with an ectopic pregnancy? 1 Miscarriage 2 Fetal anemia 3 Preterm birth 4 Fetal deformity

4 In an ectopic pregnancy, the risk for fetal deformity is high because of the pressure deformities caused by oligohydramnios. There may be facial or cranial asymmetry, various joint deformities, limb deficiency, and central nervous system (CNS) anomalies. Miscarriage is not likely to happen in an ectopic pregnancy. Instead, the patient is at risk for pregnancy-related death resulting from ectopic rupture. Fetal anemia is a risk associated with placenta previa. Preterm birth is not possible because the pregnancy is dissolved when it is diagnosed or a surgery is performed to remove the fetus.

Which is a priority nursing action when a pregnant patient with severe gestational hypertension is admitted to the health care facility? 1 Prepare the patient for cesarean delivery. 2 Administer intravenous (I.V.) and oral fluids. 3 Provide diversionary activities during bed rest. 4 Administer the prescribed magnesium sulfate.

4 The nurse administers the prescribed magnesium sulfate to the patient to prevent eclamptic seizures. I.V. oral fluids are indicated when there is severe dehydration in the patient. It is important to provide diversionary activities during bed rest, but it is secondary in this case. A patient who has experienced a multisystem trauma is prepared for cesarean delivery if there is no evidence of a maternal pulse, which increases the chance of maternal survival.

2. Which condition would require prophylaxis to prevent subacute bacterial endocarditis (SBE) both antepartum and intrapartum? a. Valvular heart disease b. Congestive heart disease c. Arrhythmias d. Postmyocardial infarction

: A Prophylaxis for intrapartum endocarditis and pulmonary infection may be provided for women who have mitral valve prolapse. Prophylaxis for intrapartum endocarditis is not indicated for a client with congestive heart disease, underlying arrhythmias, or postmyocardial infarction. DIF: Cognitive Level: Understand REF: p. 712 TOP: Nursing Process: Implementation

MULTIPLE RESPONSE 1. Which congenital anomalies can occur as a result of the use of antiepileptic drugs (AEDs) in pregnancy? (Select all that apply.) a. Cleft lip b. Congenital heart disease c. Neural tube defects d. Gastroschisis e. Diaphragmatic hernia

: A, B, C Congenital anomalies that can occur with AEDs include cleft lip or palate, congenital heart disease, urogenital defects, and neural tube defects. Carbamazepine and valproate should be avoided if all possible; they may cause neural tube defects. Congenital anomalies of gastroschisis and diaphragmatic hernia are not associated with the use of AEDs. DIF: Cognitive Level: Understand REF: p. 725 TOP: Nursing Process: Planning

3. In caring for a pregnant woman with sickle cell anemia, the nurse must be aware of the signs and symptoms of a sickle cell crisis. What do these include? (Select all that apply.) a. Fever b. Endometritis c. Abdominal pain d. Joint pain e. Urinary tract infection (UTI)

: A, C, D Women with sickle cell anemia have recurrent attacks (crises) of fever and pain, most often in the abdomen, joints, and extremities. These attacks are attributed to vascular occlusion when red blood cells (RBCs) assume the characteristic sickled shape. Crises are usually triggered by dehydration, hypoxia, or acidosis. Women with the sickle cell trait are usually at a greater risk for postpartum endometritis (uterine wall infection); however, this development is not likely to occur during the pregnancy and is not a sign for the disorder. Although women with sickle cell anemia are at an increased risk for UTIs, these infections are not an indication of a sickle cell crisis. DIF: Cognitive Level: Understand REF: p. 721 TOP: Nursing Process: Assessment

2. A lupus flare-up during pregnancy or early postpartum occurs in 15% to 60% of women with this disorder. Which conditions associated with systemic lupus erythematosus (SLE) are maternal risks? (Select all that apply.) a. Miscarriage b. Intrauterine growth restriction (IUGR) c. Nephritis d. Preeclampsia e. Cesarean birth

: A, C, D, E Maternal risks associated with SLE include miscarriage, nephritis, preeclampsia, and cesarean birth. IUGR is a fetal risk related to SLE. Other fetal risks include stillbirth and prematurity. DIF: Cognitive Level: Understand REF: p. 727 TOP: Nursing Process: Assessment

3. Which information should the nurse take into consideration when planning care for a postpartum client with cardiac disease? a. The plan of care for a postpartum client is the same as the plan for any pregnant woman. b. The plan of care includes rest, stool softeners, and monitoring of the effect of activity. c. The plan of care includes frequent ambulating, alternating with active range-of-motion exercises. d. The plan of care includes limiting visits with the infant to once per day.

: B Bed rest may be ordered, with or without bathroom privileges. Bowel movements without stress or strain for the woman are promoted with stool softeners, diet, and fluids. Care of the woman with cardiac disease in the postpartum period is tailored to the woman's functional capacity. The woman will be on bed rest to conserve energy and to reduce the strain on the heart. Although the woman may need help caring for the infant, breastfeeding and infant visits are not contraindicated. DIF: Cognitive Level: Understand REF: pp. 718-719 TOP: Nursing Process: Planning

9. Which information regarding the care of antepartum women with cardiac conditions is mostimportant for the nurse to understand? a. Stress on the heart is greatest in the first trimester and the last 2 weeks before labor. b. Women with class II cardiac disease should avoid heavy exertion and any activity that causes even minor symptoms. c. Women with class III cardiac disease should get 8 to 10 hours of sleep every day and limit housework, shopping, and exercise. d. Women with class I cardiac disease need bed rest through most of the pregnancy and face the possibility of hospitalization near term.

: B Class II cardiac disease is symptomatic with ordinary activity. Women in this category need to avoid heavy exertion and limit regular activities as symptoms dictate. Stress is greatest between weeks 28 and 32 of gestation, when hemodynamic changes reach their maximum. Class III cardiac disease is symptomatic with less-than-ordinary activity. These women need bed rest most of the day and face the possibility of hospitalization near term. Class I cardiac disease is asymptomatic at normal levels of activity. These women can perform limited normal activities with discretion, although they still need a good amount of sleep. DIF: Cognitive Level: Understand REF: p. 711 TOP: Nursing Process: Planning

12. The client makes an appointment for preconception counseling. The woman has a known heart condition and is unsure if she should become pregnant. Which is the only cardiac condition that would cause concern? a. Marfan syndrome b. Eisenmenger syndrome c. Heart transplant d. Ventricular septal defect (VSD)

: B Pregnancy is contraindicated in clients with Eisenmenger syndrome. Women who have had heart transplants are successfully having babies. However, conception should be postponed for at least 1 year after transplantation. Management of the client with Marfan syndrome during pregnancy includes bed rest, beta-blockers, and surgery before conception. VSD is usually corrected early in life and is therefore not a contraindication to pregnancy. DIF: Cognitive Level: Understand REF: p. 714 TOP: Nursing Process: Assessment

15. Bell palsy is an acute idiopathic facial paralysis, the cause for which remains unknown. Which statement regarding this condition is correct? a. Bell palsy is the sudden development of bilateral facial weakness. b. Women with Bell palsy have an increased risk for hypertension. c. Pregnant women are affected twice as often as nonpregnant women. d. Bell palsy occurs most frequently in the first trimester.

: B The clinical manifestations of Bell palsy include the development of unilateral facial weakness, pain surrounding the ears, difficulty closing the eye, and hyperacusis. The cause is unknown; however, Bell palsy may be related to a viral infection. Pregnant women are affected at a rate of three to five times that of nonpregnant women. The incidence rate peaks during the third trimester and puerperium. Women who develop Bell palsy in pregnancy have an increased risk for hypertension. DIF: Cognitive Level: Understand REF: p. 726 TOP: Nursing Process: Assessment

18. It is extremely rare for a woman to die in childbirth; however, it can happen. In the United States, the annual occurrence of maternal death is 12 per 100,000 cases of live birth. What are the leading causes of maternal death? a. Embolism and preeclampsia b. Trauma and motor vehicle accidents (MVAs) c. Hemorrhage and infection d. Underlying chronic conditions

: B Trauma is the leading cause of obstetric death in women of childbearing age. Most maternal injuries are the result of MVAs and falls. Although preeclampsia and embolism are significant contributors to perinatal morbidity, these are not the leading cause of maternal mortality. Maternal death caused by trauma may occur as the result of hemorrhagic shock or abruptio placentae. In these cases, the hemorrhage is the result of trauma, not childbirth. The wish to become a parent is not eliminated by a chronic health problem, and many women each year risk their lives to have a baby. Because of advanced pediatric care, many women are surviving childhood illnesses and reaching adulthood with chronic health problems such as cystic fibrosis, diabetes, and pulmonary disorders. DIF: Cognitive Level: Understand REF: p. 731 TOP: Nursing Process: Assessment

4. Autoimmune disorders often occur during pregnancy because a large percentage of women with an autoimmune disorder are of childbearing age. Which disorders fall into the category of collagen vascular disease? (Select all that apply.) a. Multiple sclerosis b. SLE c. Antiphospholipid syndrome d. Rheumatoid arthritis e. Myasthenia gravis

: B, C, D, E Multiple sclerosis is not an autoimmune disorder. This patchy demyelination of the spinal cord may be a viral disorder. Autoimmune disorders (collagen vascular disease) make up a large group of conditions that disrupt the function of the immune system of the body. These disorders include those listed, as well as systemic sclerosis. DIF: Cognitive Level: Comprehend REF: p. 726 TOP: Nursing Process: Assessment

6. Which important component of nutritional counseling should the nurse include in health teaching for a pregnant woman who is experiencing cholecystitis? a. Assess the woman's dietary history for adequate calories and proteins. b. Teach the woman that the bulk of calories should come from proteins. c. Instruct the woman to eat a low-fat diet and to avoid fried foods. d. Instruct the woman to eat a low-cholesterol, low-salt diet.

: C Eating a low-fat diet and avoiding fried foods is appropriate nutritional counseling for this client. Caloric and protein intake do not predispose a woman to the development of cholecystitis. The woman should be instructed to limit protein intake and choose foods that are high in carbohydrates. A low-cholesterol diet may be the result of limiting fats. However, a low-salt diet is not indicated. DIF: Cognitive Level: Apply REF: p. 728 TOP: Nursing Process: Implementation

10. A woman at 28 weeks of gestation experiences blunt abdominal trauma as the result of a fall. The nurse must closely observe the client for what? a. Alteration in maternal vital signs, especially blood pressure b. Complaints of abdominal pain c. Placental absorption d. Hemorrhage

: C Electronic fetal monitoring (EFM) tracings can help evaluate maternal status after trauma and can reflect fetal cardiac responses to hypoxia and hypoperfusion. Signs and symptoms of placental absorption include uterine irritability, contractions, vaginal bleeding, and changes in FHR characteristics. Hypoperfusion may be present in the pregnant woman before the onset of clinical signs of shock. EFM tracings show the first signs of maternal compromise, such as when the maternal heart rate, blood pressure, and color appear normal, yet the EFM printout shows signs of fetal hypoxia. Abdominal pain, in and of itself, is not the most important symptom. However, if it is accompanied by contractions, changes in the FHR, rupture of membranes, or vaginal bleeding, then the client should be evaluated for abruptio placentae. Clinical signs of hemorrhage do not appear until after a 30% loss of circulating volume occurs. Careful monitoring of fetal status significantly assists in maternal assessment, because the fetal monitor tracing works as an oximeter of internal well-being. DIF: Cognitive Level: Apply REF: p. 732 TOP: Nursing Process: Assessment

16. A pregnant woman at term is transported to the emergency department (ED) after a severe vehicular accident. The obstetric nurse responds and rushes to the ED with a fetal monitor. Cardiopulmonary arrest occurs as the obstetric nurse arrives. What is the highest priority for the trauma team? a. Obtaining IV access, and starting aggressive fluid resuscitation b. Quickly applying the fetal monitor to determine whether the fetus viability c. Starting cardiopulmonary resuscitation (CPR) d. Transferring the woman to the surgical unit for an emergency cesarean delivery in case the fetus is still alive

: C In a situation of severe maternal trauma, the systematic evaluation begins with a primary survey and the initial ABCs (airway, breathing, and circulation) of resuscitation. CPR is initiated first, followed by intravenous (IV) replacement fluid. After immediate resuscitation and successful stabilization measures, a more detailed secondary survey of the mother and fetus should be accomplished. Attempts at maternal resuscitation are made, followed by a secondary survey of the fetus. In the presence of multisystem trauma, a cesarean delivery may be indicated to increase the chance for maternal survival. DIF: Cognitive Level: Apply REF: p. 734 TOP: Nursing Process: Implementation

5. A woman with asthma is experiencing a postpartum hemorrhage. Which drug should be avoided when treating postpartum bleeding to avoid exacerbating asthma? a. Oxytocin (Pitocin) b. Nonsteroidal antiinflammatory drugs (NSAIDs) c. Hemabate d. Fentanyl

: C Prostaglandin derivatives should not be used to treat women with asthma, because they may exacerbate symptoms. Oxytocin is the drug of choice to treat this woman's bleeding; it will not exacerbate her asthma. NSAIDs are not used to treat bleeding. Fentanyl is used to treat pain, not bleeding. DIF: Cognitive Level: Analyze REF: p. 722 TOP: Nursing Process: Planning

4. A woman has experienced iron deficiency anemia during her pregnancy. She had been taking iron for 3 months before the birth. The client gave birth by cesarean 2 days earlier and has been having problems with constipation. After assisting her back to bed from the bathroom, the nurse notes that the woman's stools are dark (greenish-black). What should the nurse's initial action be? a. Perform a guaiac test, and record the results. b. Recognize the finding as abnormal, and report it to the primary health care provider. c. Recognize the finding as a normal result of iron therapy. d. Check the woman's next stool to validate the observation.

: C The nurse should recognize that dark stools are a common side effect in clients who are taking iron replacement therapy. A guaiac test would be indicated if gastrointestinal (GI) bleeding was suspected. GI irritation, including dark stools, is also a common side effect of iron therapy. Observation of stool formation is a normal nursing activity. DIF: Cognitive Level: Apply REF: p. 716 TOP: Nursing Process: Evaluation

11. Which neurologic condition would require preconception counseling, if at all possible? a. Eclampsia b. Bell palsy c. Epilepsy d. Multiple sclerosis

: C Women with epilepsy should receive preconception counseling, if at all possible. Achieving seizure control before becoming pregnant is a desirable state. Medication should also be carefully reviewed. Eclampsia may sometimes be confused with epilepsy, and Bell palsy is a form of facial paralysis; preconception counseling for either condition is not essential to care. Multiple sclerosis is a patchy demyelination of the spinal cord that does not affect the normal course of pregnancy or birth. DIF: Cognitive Level: Understand REF: p. 725 TOP: Nursing Process: Planning

7. Postoperative care of the pregnant woman who requires abdominal surgery for appendicitis includes which additional assessment? a. Intake and output (I&O) and intravenous (IV) site b. Signs and symptoms of infection c. Vital signs and incision d. Fetal heart rate (FHR) and uterine activity

: D Care of a pregnant woman undergoing surgery for appendicitis differs from that for a nonpregnant woman in one significant aspect: the presence of the fetus. Continuous fetal and uterine monitoring should take place. An assessment of I&O levels, along with an assessment of the IV site, are normal postoperative care procedures. Evaluating the client for signs and symptoms of infection is also part of routine postoperative care. Routine vital signs and evaluation of the incision site are expected components of postoperative care. DIF: Cognitive Level: Apply REF: p. 730 TOP: Nursing Process: Assessment

8. Since the gene for cystic fibrosis was identified in 1989, data can be collected for the purposes of genetic counseling for couples regarding carrier status. According to the most recent statistics, how often does cystic fibrosis occur in Caucasian live births? a. 1 in 100 b. 1 in 1000 c. 1 in 2000 d. 1 in 3200

: D Cystic fibrosis occurs in approximately 1 in 3200 Caucasian live births. 1 in 100, 1 in 1000, and 1 in 2000 occurrences of cystic fibrosis in live births are all too frequent rates. DIF: Cognitive Level: Remember REF: p. 722 TOP: Nursing Process: Assessment

13. What form of heart disease in women of childbearing years generally has a benign effect on pregnancy? a. Cardiomyopathy b. Rheumatic heart disease c. Congenital heart disease d. Mitral valve prolapse

: D Mitral valve prolapse is a benign condition that is usually asymptomatic. Cardiomyopathy produces congestive heart failure during pregnancy. Rheumatic heart disease can lead to heart failure during pregnancy. Some congenital heart diseases produce pulmonary hypertension or endocarditis during pregnancy. DIF: Cognitive Level: Remember REF: p. 713 TOP: Nursing Process: Assessment

14. A pregnant woman at 33 weeks of gestation is brought to the birthing unit after a minor automobile accident. The client is experiencing no pain and no vaginal bleeding, her vital signs are stable, and the FHR is 132 beats per minute with variability. What is the nurse's highestpriority? a. Monitoring the woman for a ruptured spleen b. Obtaining a physician's order to discharge her home c. Monitoring her for 24 hours d. Using continuous EFM for a minimum of 4 hours

: D Monitoring the external FHR and contractions is recommended after blunt trauma in a viable gestation for a minimum of 4 hours, regardless of injury severity. Fetal monitoring should be initiated as soon as the woman is stable. In this scenario, no clinical findings indicate the possibility of a ruptured spleen. If the maternal and fetal findings are normal, then EFM should continue for a minimum of 4 hours after a minor trauma or a minor automobile accident. Once the monitoring has been completed and the health care provider is reassured of fetal well-being, the client may be discharged home. Monitoring for 24 hours is unnecessary unless the ERM strip is abnormal or nonreassuring. DIF: Cognitive Level: Apply REF: p. 732 TOP: Nursing Process: Planning

17. Another common pregnancy-specific condition is pruritic urticarial papules and plaques of pregnancy (PUPPP). A client asks the nurse why she has developed this condition and what can be done. What is the nurse's bestresponse? a. PUPPP is associated with decreased maternal weight gain. b. The rate of hypertension decreases with PUPPP. c. This common pregnancy-specific condition is associated with a poor fetal outcome. d. The goal of therapy is to relieve discomfort.

: D PUPPP is associated with increased maternal weight gain, increased rate of twin gestation, and hypertension. It is not, however, associated with poor maternal or fetal outcomes. The goal of therapy is simply to relieve discomfort. Antipruritic topical medications, topical steroids, and antihistamines usually provide relief. PUPPP usually resolves before childbirth or shortly thereafter. DIF: Cognitive Level: Apply REF: p. 724 TOP: Nursing Process: Planning

1. When caring for a pregnant woman with cardiac problems, the nurse must be alert for the signs and symptoms of cardiac decompensation. Which critical findings would the nurse find on assessment of the client experiencing this condition? a. Regular heart rate and hypertension b. Increased urinary output, tachycardia, and dry cough c. Shortness of breath, bradycardia, and hypertension d. Dyspnea, crackles, and an irregular, weak pulse

: D Signs of cardiac decompensation include dyspnea; crackles; an irregular, weak, and rapid pulse; rapid respirations; a moist and frequent cough; generalized edema; increasing fatigue; and cyanosis of the lips and nailbeds. A regular heart rate and hypertension are not generally associated with cardiac decompensation. Of the symptoms of increased urinary output, tachycardia, and dry cough, only tachycardia is indicative of cardiac decompensation. Of the symptoms of shortness of breath, bradycardia, and hypertension, only dyspnea is indicative of cardiac decompensation. DIF: Cognitive Level: Understand REF: p. 716 TOP: Nursing Process: Assessment

Screening questions for alcohol and drug abuse should be included in the overall assessment during the first prenatal visit for all women. The 4 Ps-Plus is a screening tool designed specifically to identify when there is a need for a more in-depth assessment. Which of the following is not included in the 4 Ps-Plus screening tool? a. Present b. Partner c. Past d. Pregnancy

A

28. What is the correct placement of the tocotransducer for effective EFM? a. Over the uterine fundus b. On the fetal scalp c. Inside the uterus d. Over the mothers lower abdomen

A (The tocotransducer monitors uterine activity and should be placed over the fundus, where the most intensive uterine contractions occur. The tocotransducer is for external use.)

24. Which of these correctly describes the average length of pregnancy? A. 38 weeks B. 9 lunar months C. 280 days from the last day of the last menstrual period D. 280 days from the first day of the last menstrual period

ANS: D The average length of pregnancy is 280 days from the first day of the last menstrual period, which is equal to 40 weeks, 10 lunar months, or roughly 9 calendar months

15. With one exception, the safest pregnancy is one during which the woman is drug and alcohol free. What is the optimal treatment for women addicted to opioids? a. Methadone maintenance treatment (MMT) b. Detoxification c. Smoking cessation d. 4 Ps Plus

A MMT is currently considered the standard of care for pregnant women who are dependent on heroin or other narcotics. Buprenorphine is another medication approved for the treatment of opioid addiction that is increasingly being used during pregnancy. Opioid replacement therapy has been shown to decrease opioid and other drug use, reduce criminal activity, improve individual functioning, and decrease the rates of infections such as hepatitis B and C, human immunodeficiency virus (HIV), and other STIs. Detoxification is the treatment used for alcohol addiction. Pregnant women requiring withdrawal from alcohol should be admitted for inpatient management. Women are more likely to stop smoking during pregnancy than at any other time in their lives. A smoking cessation program can assist in achieving this goal. The 4 Ps Plus is a screening tool specifically designed to identify pregnant women who need in-depth assessment related to substance abuse.

4. Despite warnings, prenatal exposure to alcohol continues to far exceed exposure to illicit drugs. Which condition is rarely associated with fetal alcohol syndrome (FAS)? a. Respiratory conditions b. Intellectual impairment c. Neural development disorder d. Alcohol-related birth defects (ARBDs)

A Respiratory difficulties are not attributed to exposure to alcohol in utero. Other abnormalities related to FAS include mental retardation, neurodevelopment disorders, and ARBDs.

Which opiate causes euphoria, relaxation, drowsiness, and detachment from reality and has possible effects on the pregnancy, including preeclampsia, intrauterine growth restriction, and premature rupture of membranes? a. Heroin b. Alcohol c. Phencyclidine palmitate (PCP) d. Cocaine

A The opiates include opium, heroin, meperidine, morphine, codeine, and methadone. The signs and symptoms of heroin use are euphoria, relaxation, relief from pain, detachment from reality, impaired judgment, drowsiness, constricted pupils, nausea, constipation, slurred speech, and respiratory depression. Possible effects on pregnancy include preeclampsia, intrauterine growth restriction, miscarriage, premature rupture of membranes, infections, breech presentation, and preterm labor. Alcohol is not an opiate. PCP is not an opiate. Cocaine is not an opiate.

6. Which alteration in the FHR pattern would indicate the potential need for an amnioinfusion? a. Variable decelerations b. Late decelerations c. Fetal bradycardia d. Fetal tachycardia

A (Amnioinfusion is used during labor to either dilute meconium-stained amniotic fluid or supplement the amount of amniotic fluid to reduce the severity of variable FHR decelerations caused by cord compression. Late decelerations are unresponsive to amnioinfusion. Amnioinfusion is not appropriate for the treatment of fetal bradycardia and has no bearing on fetal tachycardia.)

1. What is the most likely cause for early decelerations in the fetal heart rate (FHR) pattern? a. Altered fetal cerebral blood flow b. Umbilical cord compression c. Uteroplacental insufficiency d. Spontaneous rupture of membranes

A (Early decelerations are the fetus response to fetal head compression; these are considered benign, and interventions are not necessary. Variable decelerations are associated with umbilical cord compression. Late decelerations are associated with uteroplacental insufficiency. Spontaneous rupture of membranes has no bearing on the FHR unless the umbilical cord prolapses, which would result in variable or prolonged bradycardia.)

2. Which clinical finding or intervention might be considered the rationale for fetal tachycardia to occur? a. Maternal fever b. Umbilical cord prolapse c. Regional anesthesia d. Magnesium sulfate administration

A (Fetal tachycardia can be considered an early sign of fetal hypoxemia and may also result from maternal or fetal infection. Umbilical cord prolapse, regional anesthesia, and the administration of magnesium sulfate will each more likely result in fetal bradycardia, not tachycardia.)

16. Which client would not be a suitable candidate for internal EFM? a. Client who still has intact membranes b. Woman whose fetus is well engaged in the pelvis c. Pregnant woman who has a comorbidity of obesity d. Client whose cervix is dilated to 4 to 5 cm

A (For internal EFM, the membranes must have ruptured and the cervix must be dilated at least 2 to 3 cm. The presenting part must be low enough to allow placement of the spiral electrode necessary for internal EFM. The accuracy of EFM is not affected by maternal size. However, evaluating fetal well-being using external EFM may be more difficult on an obese client. The client whose cervix is dilated to 4 to 5 cm is indeed a candidate for internal monitoring.)

3. While evaluating an external monitor tracing of a woman in active labor, the nurse notes that the FHR for five sequential contractions begins to decelerate late in the contraction, with the nadir of the decelerations occurring after the peak of the contraction. What is the nurses first priority? a. Change the womans position. b. Notify the health care provider. c. Assist with amnioinfusion d. Insert a scalp electrode.

A (Late FHR decelerations may be caused by maternal supine hypotension syndrome. These decelerations are usually corrected when the woman turns onto her side to displace the weight of the gravid uterus from the vena cava. If the fetus does not respond to primary nursing interventions for late decelerations, then the nurse should continue with subsequent intrauterine resuscitation measures and notify the health care provider. An amnioinfusion may be used to relieve pressure on an umbilical cord that has not prolapsed. The FHR pattern associated with this situation most likely will reveal variable decelerations. Although a fetal scalp electrode will provide accurate data for evaluating the well-being of the fetus, it is not a nursing intervention that will alleviate late decelerations nor is it the nurses first priority.)

25. Which nursing intervention would result in an increase in maternal cardiac output? a. Change in position b. Oxytocin administration c. Regional anesthesia d. IV analgesic

A (Maternal supine hypotension syndrome is caused by the weight and pressure of the gravid uterus on the ascending vena cava when the woman is in a supine position. This position reduces venous return to the womans heart, as well as cardiac output, and subsequently reduces her blood pressure. The nurse can encourage the woman to change positions and to avoid the supine position. Oxytocin administration, regional anesthesia, and IV analgesic may reduce maternal cardiac output.)

10. What are the legal responsibilities of the perinatal nurses? a. Correctly interpreting FHR patterns, initiating appropriate nursing interventions, and documenting the outcomes b. Greeting the client on arrival, assessing her status, and starting an IV line c. Applying the external fetal monitor and notifying the health care provider d. Ensuring that the woman is comfortable

A (Nurses who care for women during childbirth are legally responsible for correctly interpreting FHR patterns, initiating appropriate nursing interventions based on those patterns, and documenting the outcomes of those interventions. Greeting the client on arrival, assessing her, and starting an IV line are activities that should be performed when any client arrives to the maternity unit. The nurse is not the only one legally responsible for performing these functions. Applying the external fetal monitor and notifying the health care provider is a nursing function that is part of the standard of care for all obstetric clients and falls within the registered nurses scope of practice. Everyone caring for the pregnant woman should ensure that both she and her support partner are comfortable.)

13. Which statement best describes a normal uterine activity pattern in labor? a. Contractions every 2 to 5 minutes b. Contractions lasting approximately 2 minutes c. Contractions approximately 1 minute apart d. Contraction intensity of approximately 500 mm Hg with relaxation at 50 mm Hg

A (Overall contraction frequency generally ranges from two to five contractions per 10 minutes of labor, with lower frequencies during the first stage and higher frequencies observed during the second stage. Contraction duration remains fairly stable throughout the first and second stages, ranging from 45 to 80 seconds, generally not exceeding 90 seconds. Contractions 1 minute apart are occurring too often and would be considered an abnormal labor pattern. The intensity of uterine contractions generally ranges from 25 to 50 mm Hg in the first stage of labor and may rise to more than 80 mm Hg in the second stage.)

23. Part of the nurses role is assisting with pushing and positioning. Which guidance should the nurse provide to her client in active labor? a. Encourage the womans cooperation in avoiding the supine position. b. Advise the woman to avoid the semi-Fowler position. c. Encourage the woman to hold her breath and tighten her abdominal muscles to produce a vaginal response. d. Instruct the woman to open her mouth and close her glottis, letting air escape after the push.

A (The woman should maintain a side-lying position. The semi-Fowler position is the recommended side-lying position with a lateral tilt to the uterus. Encouraging the woman to hold her breath and tighten her abdominal muscles is the Valsalva maneuver, which should be avoided. Both the mouth and glottis should be open, allowing air to escape during the push.)

30. The client has delivered by urgent caesarean birth for fetal compromise. Umbilical cord gases were obtained for acid-base determination. The pH is 6.9, partial pressure of carbon dioxide (PCO2) is elevated, and the base deficit is 11 mmol/L. What type of acidemia is displayed by the infant? a. Respiratory b. Metabolic c. Mixed d. Turbulent

A (These findings are evidence of respiratory acidemia. Metabolic acidemia is expressed by a pH <7.20, normal carbon dioxide pressure, and a base excess of 12 mmol/L. Mixed acidemia is evidenced by a pH <7.20, elevated carbon dioxide pressure, and a base excess of 12 mmol/L. There is no such finding as turbulent acidemia.)

18. A nurse caring for a woman in labor should understand that absent or minimal variability is classified as either abnormal or indeterminate. Which condition related to decreased variability is considered benign? a. Periodic fetal sleep state b. Extreme prematurity c. Fetal hypoxemia d. Preexisting neurologic injury

A (When the fetus is temporarily in a sleep state, minimal variability is present. Periodic fetal sleep states usually last no longer than 30 minutes. A woman in labor with extreme prematurity may display a FHR pattern of minimal or absent variability. Abnormal variability may also be related to fetal hypoxemia and metabolic acidemia. Congenital anomalies or a preexisting neurologic injury may also result in absent or minimal variability. Other possible causes might be central nervous system (CNS) depressant medications, narcotics, or general anesthesia.)

The nurse should tell a primigravida that the definitive sign indicating that labor has begun is: A. progressive uterine contractions with cervical change. B. lightening. C. rupture of membranes. D. passage of the mucous plug (operculum).

A Regular, progressive uterine contractions that increase in intensity and frequency are the definitive sign of true labor along with cervical change. Lightening is a premonitory sign indicating that the onset of labor is getting closer. Rupture of membranes usually occurs during labor itself. Passage of the mucous plug is a premonitory sign indicating that the onset of labor is getting closer

On completion of a vaginal examination on a laboring woman, the nurse records: 50%, 6 cm, -1. What is a correct interpretation of the data? A. The fetal presenting part is 1 cm above the ischial spines. B. Effacement is 4 cm from completion. C. Dilation is 50% completed. D. The fetus has achieved passage through the ischial spines.

A Station of -1 indicates that the fetal presenting part is above the ischial spines and has not yet passed through the pelvic inlet. Progress of effacement is referred to by percentages, with 100% indicating full effacement and dilation by centimeters, with 10 cm indicating full dilation. Progress of effacement is referred to by percentages, with 100% indicating full effacement and dilation by centimeters, with 10 cm indicating full dilation. Passage through the ischial spines with internal rotation would be indicated by a plus station such as +1.

The nurse is caring for a pregnant client who is in the second stage of labor. The nurse instructs the client not to hold her breath or tighten the abdominal muscles while having intense labor pain. What is the rationale for this instruction? To prevent: A. The onset of fetal hypoxia. B. Maternal hypotension. C. increased fetal heart rate. D. Hemorrhoids in the client.

A While caring for a client who is in the second stage of the labor, the nurse should instruct the client to refrain from performing the Valsalva maneuver. During the Valsalva maneuver, the client holds her breath and tightens the abdominal muscles, which may reduce the oxygen content in the blood and cause fetal hypoxia. Tightening of the abdominal muscles increases the intrathoracic pressure and may cause hypertension in the client, but not hypotension. Due to reduced oxygen supply, the pulse rate may decrease and cause fetal bradycardia, but not tachycardia. The pressure exerted by the fetus on the vaginal wall during the delivery causes hemorrhoids, but these are not associated with the Valsalva maneuver.

Achieving and maintaining euglycemia are the primary goals of medical therapy for the pregnant woman with diabetes. These goals are achieved through a combination of diet, insulin, exercise, and blood glucose monitoring. The target blood glucose levels 1 hour after a meal should be _____________.

ANS: 110 to 129 mg/dl Target levels of blood glucose during pregnancy are lower than nonpregnant values. Accepted fasting levels are between 60 and 99 mg/dl, and 1-hour postmeal levels should be between 110 to 129 mg/dl. Two-hour postmeal levels should be 120 mg/dl or less.

1. In assessing the immediate condition of the newborn after birth, a sample of cord blood may be a useful adjunct to the Apgar score. Cord blood is then tested for pH, carbon dioxide, oxygen, and base deficit or excess. Which clinical situation warrants this additional testing? (Select all that apply.) a. Low 5-minute Apgar score b. Intrauterine growth restriction (IUGR) c. Maternal thyroid disease d. Intrapartum fever e. Vacuum extraction

A, B, C, D (The American College of Obstetricians and Gynecologists (ACOG) suggests obtaining cord blood values in all of these clinical situations except for vacuum extractions deliveries. Cord blood gases should also be performed for multifetal pregnancies or abnormal FHR tracings. Samples can be drawn from both the umbilical artery and the umbilical vein. Results may indicate that fetal compromise has occurred.)

26. Intrauterine growth restriction (IUGR) is associated with numerous pregnancy-related risk factors (Select all that apply). a. Poor nutrition b. Maternal collagen disease c. Gestational hypertension d. Premature rupture of membranes e. Smoking

A, B, C, E

27. Transvaginal ultrasonography is often performed during the first trimester. While preparing your 6-week gestation patient for this procedure, she expresses concerns over the necessity for this test. The nurse should explain that this diagnostic test may be indicated for a number of situations (Select all that apply). a. Multifetal gestation b. Obesity c. Fetal abnormalities d. Amniotic fluid volume e. Ectopic pregnancy

A, B, C, E

The charge nurse on the maternity unit is orienting a new nurse to the unit and explains that the 5 Ps of labor and birth are: Select all that apply. A. passenger. B. placenta. C. passageway. D. psychological response. E. powers. F. position.

A, C, D, E, F At least five factors affect the process of labor and birth. These are easily remembered as the five Ps: passenger (fetus and placenta), passageway (birth canal), powers (contractions), position of the mother, and psychological response.

What are the factors that speed up the dilation of the cervix? Select all that apply. A. Strong uterine contractions B. Scarring of the cervix C. Pressure by amniotic fluid D. Prior infection of the cervix E. Force by fetal presenting part

A, C, E Dilation of the cervix occurs by the drawing upward of the musculofibrous components of the cervix, which are, in turn, caused by strong uterine contractions. Pressure exerted by the amniotic fluid while the membranes are intact or by the force applied by the presenting part can promote cervical dilation. Scarring of the cervix may occur following a surgery. Prior infection or surgery may slow cervical dilation.

The 21-year-old female is affected with human papillopmavirus (HPV) following unprotected sexual intercourse with a male she recently met. She is now at higher risk for developing which of the following cancers? a. cervical b. Ovarian c. Endometrial d vulvar

A. Cervical HPV is associated with cervical cancer HPV 16 accounts for about 60% of cervical cancer cases and HBP 18 for about another 10%, other types contributes less than 5% of cases.

A 35-year-old female is diagnosed with ovarian cancer. CT scan reveals that the cancer is limited to the ovaries. It would be classified as stage: a. I b. II c. III d. IV

A. I In stage I, growth is limited to the ovaries.In stage II, growth involves one or both ovaries and involvement of other organs.In III, cancer involves one or both ovaries, and one of both of the following: (i) cancer has spread beyond the pelvis to the lining of the abdomen and (ii) cancer has spread to lymph nodes. In stage IV, growth involves one or both ovaries with distant methestases to lungs, liver, or other organs outside the peritoneal cavity.

2. Screening questions for alcohol and drug abuse should be included in the overall assessment during the first prenatal visit for all women. The 4 Ps Plus is a screening tool specifically designed to identify the need for a more in-depth assessment. Which are the correct components of the 4 Ps Plus? (Select all that apply.) a. Parents b. Partner c. Present d. Past e. Pregnancy

ABDE The 4Ps Plus is a screening tool designed specifically to identify pregnant women who need in-depth assessment. The 4Ps Plus tool includes: Past: Have you ever had any beer or wine or liquor? Partner: Does your partner have a problem with alcohol or drugs? Parents: Did either of your parents ever have a problem with alcohol or drugs? Pregnancy: In the month before you knew you were pregnant, how many cigarettes did you smoke? How much beer, wine, or liquor did you drink?

A 35-year-old nonpregnant female presents with breast discharge and is diagnosed with galactorrhea. The condition is most likely caused by: a. Infection b. Prolactin imbalance c. Tissue injury d. Cancer

B. Prolactin imbalance the most common cause of galactorrhea is nonpuerperal hyperprolactinemia, and excessive amounts of prolactin in the blood not related to pregnancy or childbirth.

12. When is a prophylactic cerclage for an incompetent cervix usually placed (in weeks of gestation)? a. 12 to 14 b. 6 to 8 c. 23 to 24 d. After 24

ANS: A A prophylactic cerclage is usually placed at 12 to 14 weeks of gestation. The cerclage is electively removed when the woman reaches 37 weeks of gestation or when her labor begins. Six to 8 weeks of gestation is too early to place the cerclage. Cerclage placement is offered if the cervical length falls to less than 20 to 25 mm before 23 to 24 weeks. Although no consensus has been reached, 24 weeks is used as the upper gestational age limit for cerclage placement. DIF: Cognitive Level: Apply REF: p. 674 TOP: Nursing Process: Planning MSC: Client Needs: Health Promotion and Maintenance

A client at 39 weeks of gestation has been admitted for an external version. Which intervention would the nurse anticipate the provider to order? a.Tocolytic drug b.Contraction stress test (CST) c.Local anesthetic d.Foley catheter

ANS: A A tocolytic drug will relax the uterus before and during the version, thus making manipulation easier. CST is used to determine the fetal response to stress. A local anesthetic is not used with external version. Although the bladder should be emptied, catheterization is not necessary.

Which information regarding the procedures and criteria for admitting a woman to the hospital labor unit is important for the nurse to understand? a.Client is considered to be in active labor when she arrives at the facility with contractions. b.Client can have only her male partner or predesignated doula with her at assessment. c.Children are not allowed on the labor unit. d.Non-English speaking client must bring someone to translate.

ANS: A According to the Emergency Medical Treatment and Active Labor Act (EMTALA), a woman is entitled to active labor care and is presumed to be in true labor until a qualified health care provider certifies otherwise. A woman may have anyone she wishes present for her support. An interpreter must be provided by the hospital, either in person or by a telephonic service. Siblings of the new infant may be allowed at the delivery, depending on hospital policy and adequate preparation and supervision.

Nursing care measures are commonly offered to women in labor. Which nursing measure reflects the application of the gate-control theory? a.Massage the woman's back. b.Change the woman's position. c.Give the prescribed medication. d.Encourage the woman to rest between contractions.

ANS: A According to the gate-control theory, pain sensations travel along sensory nerve pathways to the brain, but only a limited number of sensations, or messages, can travel through these nerve pathways at one time. Distraction techniques, such as massage or stroking, music, focal points, and imagery, reduce or completely block the capacity of the nerve pathways to transmit pain. These distractions are thought to work by closing down a hypothetic gate in the spinal cord, thus preventing pain signals from reaching the brain. The perception of pain is thereby diminished. Changing the woman's position, administering pain medication, and resting between contractions do not reduce or block the capacity of the nerve pathways to transmit pain using the gate-control theory.

18. What is the correct definition of a spontaneous termination of a pregnancy (abortion)? a. Pregnancy is less than 20 weeks. b. Fetus weighs less than 1000 g. c. Products of conception are passed intact. d. No evidence exists of intrauterine infection.

ANS: A An abortion is the termination of pregnancy before the age of viability (20 weeks). The weight of the fetus is not considered because some older fetuses may have a low birth weight. A spontaneous abortion may be complete or incomplete and may be caused by many problems, one being intrauterine infection. DIF: Cognitive Level: Remember REF: p. 669 TOP: Nursing Process: Assessment MSC: Client Needs: Health Promotion and Maintenance

A woman in preterm labor at 30 weeks of gestation receives two 12-mg intramuscular (IM) doses of betamethasone. What is the purpose of this pharmacologic intervention? a.To stimulate fetal surfactant production b.To reduce maternal and fetal tachycardia associated with ritodrine administration c.To suppress uterine contractions d.To maintain adequate maternal respiratory effort and ventilation during magnesium sulfate therapy

ANS: A Antenatal glucocorticoids administered as IM injections to the mother accelerate fetal lung maturity. Propranolol (Inderal) is given to reduce the effects of ritodrine administration. Betamethasone has no effect on uterine contractions. Calcium gluconate is given to reverse the respiratory depressive effects of magnesium sulfate therapy.

21. What is the highest priority nursing intervention when admitting a pregnant woman who has experienced a bleeding episode in late pregnancy? a. Assessing FHR and maternal vital signs b. Performing a venipuncture for hemoglobin and hematocrit levels c. Placing clean disposable pads to collect any drainage d. Monitoring uterine contractions

ANS: A Assessment of the FHR and maternal vital signs will assist the nurse in determining the degree of the blood loss and its effect on the mother and fetus. The most important assessment is to check the well-being of both the mother and the fetus. The blood levels can be obtained later. Assessing future bleeding is important; however, the top priority remains mother/fetal well-being. Monitoring uterine contractions is important but not a top priority. DIF: Cognitive Level: Apply REF: p. 681 TOP: Nursing Process: Implementation MSC: Client Needs: Health Promotion and Maintenance

Which description most accurately describes the augmentation of labor? a.Is part of the active management of labor that is instituted when the labor process is unsatisfactory b.Relies on more invasive methods when oxytocin and amniotomy have failed c.Is a modern management term to cover up the negative connotations of forceps-assisted birth d.Uses vacuum cups

ANS: A Augmentation is part of the active management of labor that stimulates uterine contractions after labor has started but is not progressing satisfactorily. Augmentation uses amniotomy and oxytocin infusion, as well as some more gentle, noninvasive methods. Forceps-assisted births are less common than in the past and not considered a method of augmentation. A vacuum-assisted delivery occurs during childbirth if the mother is too exhausted to push. Vacuum extraction is not considered an augmentation methodology.

6. According to research, which risk factor for PPD is likely to have the greatest effect on the client postpartum? a. Prenatal depression b. Single-mother status c. Low socioeconomic status d. Unplanned or unwanted pregnancy

ANS: A Prenatal depression has been found to be a major risk factor for PPD. Single-mother status and low socioeconomic status are both small-relationship predictors for PPD. Although an unwanted pregnancy may contribute to the risk for PPD, it does not pose as great an effect as prenatal depression.

8. The client is being induced in response to worsening preeclampsia. She is also receiving magnesium sulfate. It appears that her labor has not become active, despite several hours of oxytocin administration. She asks the nurse, "Why is this taking so long?" What is the nurse's most appropriate response? a. "The magnesium is relaxing your uterus and competing with the oxytocin. It may increase the duration of your labor." b. "I don't know why it is taking so long." c. "The length of labor varies for different women." d. "Your baby is just being stubborn."

ANS: A Because magnesium sulfate is a tocolytic agent, its use may increase the duration of labor. The amount of oxytocin needed to stimulate labor may be more than that needed for the woman who is not receiving magnesium sulfate. The nurse should explain to the client the effects of magnesium sulfate on the duration of labor. Although the length of labor varies for different women, the most likely reason this woman's labor is protracted is the tocolytic effects of magnesium sulfate. The behavior of the fetus has no bearing on the length of labor. DIF: Cognitive Level: Apply REF: p. 664 TOP: Nursing Process: Planning MSC: Client Needs: Health Promotion and Maintenance

When managing the care of a woman in the second stage of labor, the nurse uses various measures to enhance the progress of fetal descent. Which instruction best describes these measures? a.Encouraging the woman to try various upright positions, including squatting and standing b.Telling the woman to start pushing as soon as her cervix is fully dilated c.Continuing an epidural anesthetic so pain is reduced and the woman can relax d.Coaching the woman to use sustained, 10- to 15-second, closed-glottis bearing-down efforts with each contraction

ANS: A Both upright and squatting positions may enhance the progress of fetal descent. Many factors dictate when a woman should begin pushing. Complete cervical dilation is necessary, but complete dilation is only one factor. If the fetal head is still in a higher pelvic station, then the physician or midwife may allow the woman to "labor down" if the woman is able (allowing more time for fetal descent and thereby reducing the amount of pushing needed). The epidural may mask the sensations and muscle control needed for the woman to push effectively. Closed glottic breathing may trigger the Valsalva maneuver, which increases intrathoracic and cardiovascular pressures, reducing cardiac output and inhibiting perfusion of the uterus and placenta. In addition, holding her breath for longer than 5 to 7 seconds diminishes the perfusion of oxygen across the placenta and results in fetal hypoxia.

11. Which intervention is most important when planning care for a client with severe gestational hypertension? a. Induction of labor is likely, as near term as possible. b. If at home, the woman should be confined to her bed, even with mild gestational hypertension. c. Special diet low in protein and salt should be initiated. d. Vaginal birth is still an option, even in severe cases.

ANS: A By 34 weeks of gestation, the risk of continuing the pregnancy may be considered greater than the risks of a preterm birth. Strict bed rest is controversial for mild cases; some women in the hospital are even allowed to move around. Diet and fluid recommendations are essentially the same as for healthy pregnant women, although some authorities have suggested a diet high in protein. Women with severe gestational hypertension should expect a cesarean delivery. DIF: Cognitive Level: Apply REF: p. 660 TOP: Nursing Process: Planning MSC: Client Needs: Health Promotion and Maintenance

The nurse should be cognizant of which important information regarding nerve block analgesia and anesthesia? a.Most local agents are chemically related to cocaine and end in the suffix -caine. b.Local perineal infiltration anesthesia is effective when epinephrine is added, but it can be injected only once. c.Pudendal nerve block is designed to relieve the pain from uterine contractions. d.Pudendal nerve block, if performed correctly, does not significantly lessen the bearing-down reflex.

ANS: A Common agents include lidocaine and chloroprocaine. Injections can be repeated to prolong the anesthesia. A pudendal nerve block relieves pain in the vagina, vulva, and perineum but not the pain from uterine contractions. A pudendal nerve block lessens or shuts down the bearing-down reflex.

A client is experiencing back labor and complains of intense pain in her lower back. Which measure would best support this woman in labor? a.Counterpressure against the sacrum b.Pant-blow (breaths and puffs) breathing techniques c.Effleurage d.Conscious relaxation or guided imagery

ANS: A Counterpressure is steady pressure applied by a support person to the sacral area with the fist or heel of the hand. This technique helps the woman cope with the sensations of internal pressure and pain in the lower back. The pain management techniques of pant-blow, effleurage, and conscious relaxation or guided imagery are usually helpful for contractions per the gate-control theory

22. Which order should the nurse expect for a client admitted with a threatened abortion? a. Bed rest b. Administration of ritodrine IV c. Nothing by mouth (nil per os [NPO]) d. Narcotic analgesia every 3 hours, as needed

ANS: A Decreasing the woman's activity level may alleviate the bleeding and allow the pregnancy to continue. Ritodrine is not the first drug of choice for tocolytic medications. Having the woman placed on NPO is unnecessary. At times, dehydration may produce contractions; therefore, hydration is important. Narcotic analgesia will not decrease the contractions and may mask the severity of the contractions. DIF: Cognitive Level: Understand REF: pp. 671-672 TOP: Nursing Process: Planning MSC: Client Needs: Health Promotion and Maintenance

The nurse expects which maternal cardiovascular finding during labor? a.Increased cardiac output b.Decreased pulse rate c.Decreased white blood cell (WBC) count d.Decreased blood pressure

ANS: A During each contraction, 400 ml of blood is emptied from the uterus into the maternal vascular system, which increases cardiac output by approximately 10% to 15% during the first stage of labor and by approximately 30% to 50% in the second stage of labor. The heart rate increases slightly during labor. The WBC count can increase during labor. During the first stage of labor, uterine contractions cause systolic readings to increase by approximately 10 mm Hg. During the second stage, contractions may cause systolic pressures to increase by 30 mm Hg and diastolic readings to increase by 25 mm Hg.

31. During auscultation of fetal heart tones (FHTs), the nurse determines that the rate is 136 beats per minute. The nurse's next action should be to: A. Document the results, which are within normal range. B. Take the maternal pulse to verify these findings as the uterine souffle. C. Have the patient change positions and count the FHTs again. D. Notify the physician immediately for possible fetal distress.

ANS: A The normal FHT rate is between 120 and 160 beats per minute. The nurse should document the results as within the normal range. The other options are not correct

Which description of the four stages of labor is correct for both the definition and the duration? a.First stage: onset of regular uterine contractions to full dilation; less than 1 hour to 20 hours b.Second stage: full effacement to 4 to 5 cm; visible presenting part; 1 to 2 hours c.Third stage: active pushing to birth; 20 minutes (multiparous woman), 50 minutes (nulliparous woman) d.Fourth stage: delivery of the placenta to recovery; 30 minutes to 1 hour

ANS: A Full dilation may occur in less than 1 hour, but in first-time pregnancies full dilation can take up to 20 hours. The second stage of labor extends from full dilation to birth and takes an average of 20 to 50 minutes, although 2 hours is still considered normal. The third stage of labor extends from birth to the expulsion of the placenta and usually takes a few minutes. The fourth stage begins after the expulsion of the placenta and lasts until homeostasis is reestablished (approximately 2 hours).

As the United States and Canada continue to become more culturally diverse, recognizing a wide range of varying cultural beliefs and practices is increasingly important for the nursing staff. A client is from which country if she requests to have the baby's father in attendance? a.Mexico b.China c.Iran d.India

ANS: A Hispanic women routinely have fathers and female relatives in attendance during the second stage of labor. The father of the baby is expected to provide encouragement, support, and reassurance that all will be well. In China, fathers are usually not present. The side-lying position is preferred for labor and birth because it is believed that this will reduce trauma to the infant. In China, the client has a stoic response to pain. In Iran, the father will not be present. Female support persons and female health care providers are preferred. For many, a male caregiver is unacceptable. In India, the father is usually not present, but female relatives are usually in attendance. Natural childbirth methods are preferred

17. A woman with severe preeclampsia has been receiving magnesium sulfate by intravenous infusion for 8 hours. The nurse assesses the client and documents the following findings: temperature of 37.1° C, pulse rate of 96 beats per minute, respiratory rate of 24 breaths per minute, BP of 155/112 mm Hg, 3+ DTRs, and no ankle clonus. The nurse calls the provider with an update. The nurse should anticipate an order for which medication? a. Hydralazine b. Magnesium sulfate bolus c. Diazepam d. Calcium gluconate

ANS: A Hydralazine is an antihypertensive medication commonly used to treat hypertension in severe preeclampsia. Typically, it is administered for a systolic BP higher than 160 mm Hg or a diastolic BP higher than 110 mm Hg. An additional bolus of magnesium sulfate may be ordered for increasing signs of CNS irritability related to severe preeclampsia (e.g., clonus) or if eclampsia develops. Diazepam is sometimes used to stop or shorten eclamptic seizures. Calcium gluconate is used as the antidote for magnesium sulfate toxicity. The client is not currently displaying any signs or symptoms of magnesium toxicity. DIF: Cognitive Level: Analyze REF: p. 665 TOP: Nursing Process: Planning MSC: Client Needs: Physiologic Integrity

In recovery, if a woman is asked to either raise her legs (knees extended) off the bed or flex her knees, and then place her feet flat on the bed and raise her buttocks well off the bed, the purpose of this exercise is to assess what? a.Recovery from epidural or spinal anesthesia b.Hidden bleeding underneath her c.Flexibility d.Whether the woman is a candidate to go home after 6 hours

ANS: A If the numb or prickly sensations are gone from her legs after these movements, then she has likely recovered from the epidural or spinal anesthesia. Assessing the client for bleeding beneath her buttocks before discharge from the recovery is always important; however, she should be rolled to her side for this assessment. The nurse is not required to assess the woman for flexibility. This assessment is performed to evaluate whether the client has recovered from spinal anesthesia, not to determine if she is a candidate for early discharge.

Which presentation is accurately described in terms of both the resenting part and the frequency of occurrence? a.Cephalic: occiput, at least 96% b.Breech: sacrum, 10% to 15% c.Shoulder: scapula, 10% to 15% d.Cephalic: cranial, 80% to 85%

ANS: A In cephalic presentations (head first), the presenting part is the occiput; this presentation occurs in 96% of births. In a breech birth, the sacrum emerges first; this presentation occurs in approximately 3% of births. In shoulder presentations, the scapula emerges first; this presentation occurs in only 1% of births. In a cephalic presentation, the part of the head or cranium that emerges first is the occiput; cephalic presentations occur in 96% of births.

The exact cause of preterm labor is unknown but believed to be multifactorial. Infection is thought to be a major factor in many preterm labors. Which type of infection has not been linked to preterm birth? a.Viral b.Periodontal c.Cervical d.Urinary tract

ANS: A Infections that increase the risk of preterm labor and birth are bacterial and include cervical, urinary tract, periodontal, and other bacterial infections. Therefore, early, continual, and comprehensive participation by the client in her prenatal care is important. Recent evidence has shown a link between periodontal infections and preterm labor. Researchers recommend regular dental care before and during pregnancy, oral assessment as a routine part of prenatal care, and scrupulous oral hygiene to prevent periodontal infections.

Which component of the physical examination are Leopold's maneuvers unable to determine? a.Gender of the fetus b.Number of fetuses c.Fetal lie and attitude d.Degree of the presenting part's descent into the pelvis

ANS: A Leopold's maneuvers help identify the number of fetuses, the fetal lie and attitude, and the degree of descent of the presenting part into the pelvis. The gender of the fetus cannot be determined by performing Leopold's maneuvers.

15. Which neonatal complications are associated with hypertension in the mother? a. Intrauterine growth restriction (IUGR) and prematurity b. Seizures and cerebral hemorrhage c. Hepatic or renal dysfunction d. Placental abruption and DIC

ANS: A Neonatal complications are related to placental insufficiency and include IUGR, prematurity, and necrotizing enterocolitis. Seizures and cerebral hemorrhage are maternal complications. Hepatic and renal dysfunction are maternal complications of hypertensive disorders in pregnancy. Placental abruption and DIC are conditions related to maternal morbidity and mortality. DIF: Cognitive Level: Understand REF: p. 667 TOP: Nursing Process: Assessment MSC: Client Needs: Physiologic Integrity

Because of its size and rigidity, the fetal head has a major effect on the birth process. Which bones comprise the structure of the fetal skull? (Select all that apply.) a.Parietal b.Temporal c.Fontanel d.Occipital e.Femoral

ANS: A, B, D The fetal skull has two parietal bones, two temporal bones, an occipital bone, and a frontal bone. The fontanels are membrane-filled spaces.

When assessing a multiparous woman who has just given birth to an 8-pound boy, the nurse notes that the woman's fundus is firm and has become globular in shape. A gush of dark red blood comes from her vagina. What is the nurse's assessment of the situation? a.The placenta has separated. b.A cervical tear occurred during the birth. c.The woman is beginning to hemorrhage. d.Clots have formed in the upper uterine segment.

ANS: A Placental separation is indicated by a firmly contracting uterus, a change in the uterus from a discoid to a globular ovoid shape, a sudden gush of dark red blood from the introitus, an apparent lengthening of the umbilical cord, and a finding of vaginal fullness. Cervical tears that do not extend to the vagina result in minimal blood loss. Signs of hemorrhage are a boggy uterus, bright red vaginal bleeding, alterations in vital signs, pallor, lightheadedness, restlessness, decreased urinary output, and alteration in the level of consciousness. If clots have formed in the upper uterine segment, then the nurse would expect to find the uterus boggy and displaced to the side.

Screening at 24 weeks of gestation reveals that a pregnant woman has gestational diabetes mellitus (GDM). In planning her care, the nurse and the client mutually agree that an expected outcome is to prevent injury to the fetus as a result of GDM. This fetus is at the greatest risk for which condition? a.Macrosomia b.Congenital anomalies of the central nervous system c.Preterm birth d.Low birth weight

ANS: A Poor glycemic control later in pregnancy increases the rate of fetal macrosomia. Poor glycemic control during the preconception time frame and into the early weeks of the pregnancy is associated with congenital anomalies. Preterm labor or birth is more likely to occur with severe diabetes and is the greatest risk in women with pregestational diabetes. Increased weight, or macrosomia, is the greatest risk factor for this fetus.

The nurse should be cognizant of which physiologic effect of pain? a.Predominant pain of the first stage of labor is visceral pain that is located in the lower portion of the abdomen. b.Referred pain is the extreme discomfort experienced between contractions. c.Somatic pain of the second stage of labor is more generalized and related to fatigue. d.Pain during the third stage is a somewhat milder version of the pain experienced during the second stage.

ANS: A Predominant pain comes from cervical changes, the distention of the lower uterine segment, and uterine ischemia. Referred pain occurs when the pain that originates in the uterus radiates to the abdominal wall, lumbosacral area of the back, iliac crests, and gluteal area. Second-stage labor pain is intense, sharp, burning, and localized. Third-stage labor pain is similar to that of the first stage.

19. What is the most common medical complication of pregnancy? a. Hypertension b. Hyperemesis gravidarum c. Hemorrhagic complications d. Infections

ANS: A Preeclampsia and eclampsia are two noted deadly forms of hypertension. A large percentage of pregnant women will have nausea and vomiting, but a relatively few will have the severe form called hyperemesis gravidarum. Hemorrhagic complications are the second most common medical complication of pregnancy; hypertension is the most common. Infection is a risk factor for preeclampsia. DIF: Cognitive Level: Remember REF: p. 653 TOP: Nursing Process: Assessment MSC: Client Needs: Physiologic Integrity

14. In caring for the woman with DIC, which order should the nurse anticipate? a. Administration of blood b. Preparation of the client for invasive hemodynamic monitoring c. Restriction of intravascular fluids d. Administration of steroids

ANS: A Primary medical management in all cases of DIC involves a correction of the underlying cause, volume replacement, blood component therapy, optimization of oxygenation and perfusion status, and continued reassessment of laboratory parameters. Central monitoring would not be initially ordered in a client with DIC because it could contribute to more areas of bleeding. Management of DIC would include volume replacement, not volume restriction. Steroids are not indicated for the management of DIC. DIF: Cognitive Level: Apply REF: pp. 685-686 TOP: Nursing Process: Planning MSC: Client Needs: Physiologic Integrity

Which statement concerning the complication of maternal diabetes is the most accurate? a.Diabetic ketoacidosis (DKA) can lead to fetal death at any time during pregnancy. b.Hydramnios occurs approximately twice as often in diabetic pregnancies than in nondiabetic pregnancies. c.Infections occur about as often and are considered about as serious in both diabetic and nondiabetic pregnancies. d.Even mild-to-moderate hypoglycemic episodes can have significant effects on fetal well-being.

ANS: A Prompt treatment of DKA is necessary to save the fetus and the mother. Hydramnios occurs 10 times more often in diabetic pregnancies. Infections are more common and more serious in pregnant women with diabetes. Mild-to-moderate hypoglycemic episodes do not appear to have significant effects on fetal well-being.

9. What nursing diagnosis is the most appropriate for a woman experiencing severe preeclampsia? a. Risk for injury to mother and fetus, related to central nervous system (CNS) irritability b. Risk for altered gas exchange c. Risk for deficient fluid volume, related to increased sodium retention secondary to the administration of magnesium sulfate d. Risk for increased cardiac output, related to the use of antihypertensive drugs

ANS: A Risk for injury is the most appropriate nursing diagnosis for this client scenario. Gas exchange is more likely to become impaired, attributable to pulmonary edema. A risk for excess, not deficient, fluid volume, related to increased sodium retention, is increased, and a risk for decreased, not increased, cardiac output, related to the use of antihypertensive drugs, also is increased. DIF: Cognitive Level: Apply REF: p. 660 TOP: Nursing Process: Diagnosis MSC: Client Needs: Physiologic Integrity

Which alterations in the perception of pain by a laboring client should the nurse understand? a.Sensory pain for nulliparous women is often greater than for multiparous women during early labor. b.Affective pain for nulliparous women is usually less than for multiparous women throughout the first stage of labor. c.Women with a history of substance abuse experience more pain during labor. d.Multiparous women have more fatigue from labor and therefore experience more pain.

ANS: A Sensory pain is greater for nulliparous women because their reproductive tract structures are less supple. Affective pain is greater for nulliparous women during the first stage but decreases for both nulliparous and multiparous during the second stage. Women with a history of substance abuse experience the same amount of pain as those without such a history. Nulliparous women have longer labors and therefore experience more fatigue.

9. With regard to hemorrhagic complications that may occur during pregnancy, what information is most accurate? a. An incompetent cervix is usually not diagnosed until the woman has lost one or two pregnancies. b. Incidences of ectopic pregnancy are declining as a result of improved diagnostic techniques. c. One ectopic pregnancy does not affect a woman's fertility or her likelihood of having a normal pregnancy the next time. d. Gestational trophoblastic neoplasia (GTN) is one of the persistently incurable gynecologic malignancies.

ANS: A Short labors and recurring losses of pregnancy at progressively earlier gestational ages are characteristics of reduced cervical competence. Because diagnostic technology is improving, more ectopic pregnancies are being diagnosed. One ectopic pregnancy places the woman at increased risk for another one. Ectopic pregnancy is a leading cause of infertility. Once invariably fatal, GTN now is the most curable gynecologic malignancy. DIF: Cognitive Level: Understand REF: p. 675 TOP: Nursing Process: Assessment MSC: Client Needs: Health Promotion and Maintenance

Breathing patterns are taught to laboring women. Which breathing pattern should the nurse support for the woman and her coach during the latent phase of the first stage of labor if the couple has attended childbirth preparation classes? a.Slow-paced breathing b.Deep abdominal breathing c.Modified-paced breathing d.Patterned-paced breathing

ANS: A Slow-paced breathing is approximately one half the woman's normal breathing rate and is used during the early stages of labor when a woman can no longer walk or talk through her contractions. No such pattern called deep abdominal breathing exists in childbirth preparation. Modified-paced breathing is shallow breathing that is twice the woman's normal breathing rate. It is used when labor progresses and the woman can no longer maintain relaxation through paced breathing. Patterned-pace breathing is a fast, 4:1 breathe, breathe, breathe, blow pattern that is used during the transitional phase of labor just before pushing and delivery.

Which technique is an adequate means of controlling the birth of the fetal head during delivery in a vertex presentation? a.Ritgen maneuver b.Fundal pressure c.Lithotomy position d.De Lee apparatus

ANS: A The Ritgen maneuver extends the head during the actual birth and protects the perineum. Gentle, steady pressure against the fundus of the uterus facilitates vaginal birth. The lithotomy position has been commonly used in Western cultures, partly because it is convenient for the health care provider. The De Lee apparatus is used to suction fluid from the infant's mouth.

What is the most critical nursing action in caring for the newborn immediately after the birth? a.Keeping the airway clear b.Fostering parent-newborn attachment c.Drying the newborn and wrapping the infant in a blanket d.Administering eye drops and vitamin K

ANS: A The care given immediately after the birth focuses on assessing and stabilizing the newborn. Although fostering parent-newborn attachment is an important task for the nurse, it is not the most critical nursing action in caring for the newborn immediately after birth. The care given immediately after birth focuses on assessing and stabilizing the newborn. The nursing activities are (in order of importance) to maintain a patent airway, to support respiratory effort, and to prevent cold stress by drying the newborn and covering him or her with a warmed blanket or placing the newborn under a radiant warmer. After the newborn has been stabilized, the nurse assesses the newborn's physical condition, weighs and measures the newborn, administers prophylactic eye ointment and a vitamin K injection, affixes an identification bracelet, wraps the newborn in warm blankets, and then gives the newborn to the partner or to the mother of the infant.

13. In caring for an immediate postpartum client, the nurse notes petechiae and oozing from her intravenous (IV) site. The client would be closely monitored for which clotting disorder? a. DIC b. Amniotic fluid embolism (AFE) c. Hemorrhage d. HELLP syndrome

ANS: A The diagnosis of DIC is made according to clinical findings and laboratory markers. A physical examination reveals unusual bleeding. Petechiae may appear around a blood pressure cuff on the woman's arm. Excessive bleeding may occur from the site of slight trauma such as venipuncture sites. These symptoms are not associated with AFE, nor is AFE a bleeding disorder. Hemorrhage occurs for a variety of reasons in the postpartum client. These symptoms are associated with DIC. Hemorrhage would be a finding associated with DIC and is not a clotting disorder in and of itself. HELLP syndrome is not a clotting disorder, but it may contribute to the clotting disorder DIC. DIF: Cognitive Level: Understand REF: p. 685 TOP: Nursing Process: Planning MSC: Client Needs: Physiologic Integrity

Which stage of labor varies the most in length? a.First b.Second c.Third d.Fourth

ANS: A The first stage of labor is considered to last from the onset of regular uterine contractions to the full dilation of the cervix. The first stage is significantly longer than the second and third stages combined. In a first-time pregnancy, the first stage of labor can take up to 20 hours. The second stage of labor lasts from the time the cervix is fully dilated to the birth of the fetus. The average length is 20 minutes for a multiparous woman and 50 minutes for a nulliparous woman. The third stage of labor lasts from the birth of the fetus until the placenta is delivered. This stage may be as short as 3 minutes or as long as 1 hour. The fourth stage of labor, recovery, lasts approximately 2 hours after the delivery of the placenta.

Under which circumstance should the nurse assist the laboring woman into a hands-and-knees position? a.Occiput of the fetus is in a posterior position. b.Fetus is at or above the ischial spines. c.Fetus is in a vertex presentation. d.Membranes have ruptured.

ANS: A The hands-and-knees position is effective in helping to rotate the fetus from a posterior to an anterior position. Many women experience the irresistible urge to push when the fetus is at the level of the ischial spines. In some cases, this urge is felt before the woman is fully dilated. The woman should be instructed not to push until complete cervical dilation has occurred. No one position is correct for childbirth. The two most common positions assumed by women are the sitting and side-lying positions. The woman may be encouraged into a hands-and-knees position if the umbilical cord prolapsed when the membranes ruptured.

Which nursing intervention should be immediately performed after the forceps-assisted birth of an infant? a.Assessing the infant for signs of trauma b.Administering prophylactic antibiotic agents to the infant c.Applying a cold pack to the infant's scalp d.Measuring the circumference of the infant's head

ANS: A The infant should be assessed for bruising or abrasions at the site of application, facial palsy, and subdural hematoma. Prophylactic antibiotics are not necessary with a forceps delivery. A cold pack would place the infant at risk for cold stress and is contraindicated. Measuring the circumference of the head is part of the initial nursing assessment.

Which major neonatal complication is carefully monitored after the birth of the infant of a diabetic mother? a.Hypoglycemia b.Hypercalcemia c.Hypobilirubinemia d.Hypoinsulinemia

ANS: A The neonate is at highest risk for hypoglycemia because fetal insulin production is accelerated during pregnancy to metabolize excessive glucose from the mother. At birth, the maternal glucose supply stops and the neonatal insulin exceeds the available glucose, thus leading to hypoglycemia. Hypocalcemia is associated with preterm birth, birth trauma, and asphyxia, all common problems of the infant of a diabetic mother. Excess erythrocytes are broken down after birth, and large amounts of bilirubin are released into the neonate's circulation, with resulting hyperbilirubinemia. Because fetal insulin production is accelerated during pregnancy, hyperinsulinemia develops in the neonate.

What is the nurse's understanding of the appropriate role of primary and secondary powers? a.Primary powers are responsible for the effacement and dilation of the cervix. b.Effacement is generally well ahead of dilation in women giving birth for the first time; they are closer together in subsequent pregnancies. c.Scarring of the cervix caused by a previous infection or surgery may make the delivery a bit more painful, but it should not slow or inhibit dilation. d.Pushing in the second stage of labor is more effective if the woman can breathe deeply and control some of her involuntary needs to push, as the nurse directs.

ANS: A The primary powers are responsible for dilation and effacement; secondary powers are concerned with expulsion of the fetus. Effacement is generally well ahead of dilation in first-time pregnancies; they are closer together in subsequent pregnancies. Scarring of the cervix may slow dilation. Pushing is more effective and less fatiguing when the woman begins to push only after she has the urge to do so.

When assessing a woman in the first stage of labor, which clinical finding will alert the nurse that uterine contractions are effective? a.Dilation of the cervix b.Descent of the fetus to -2 station c.Rupture of the amniotic membranes d.Increase in bloody show

ANS: A The vaginal examination reveals whether the woman is in true labor. Cervical change, especially dilation, in the presence of adequate labor, indicates that the woman is in true labor. Engagement and descent of the fetus are not synonymous and may occur before labor. ROM may occur with or without the presence of labor. Bloody show may indicate a slow, progressive cervical change (e.g., effacement) in both true and false labor.

A pregnant woman's amniotic membranes have ruptured. A prolapsed umbilical cord is suspected. What intervention would be the nurse's highest priority? a.Placing the woman in the knee-chest position b.Covering the cord in sterile gauze soaked in saline c.Preparing the woman for a cesarean birth d.Starting oxygen by face mask

ANS: A The woman is assisted into a modified Sims position, Trendelenburg position, or the knee-chest position in which gravity keeps the pressure of the presenting part off the cord. Although covering the cord in sterile gauze soaked saline, preparing the woman for a cesarean, and starting oxygen by face mark are appropriate nursing interventions in the event of a prolapsed cord, the intervention of top priority would be positioning the mother to relieve cord compression.

A laboring woman is reclining in the supine position. What is the most appropriate nursing action at this time? a.Ask her to turn to one side. b.Elevate her feet and legs. c.Take her blood pressure. d.Determine whether fetal tachycardia is present.

ANS: A The woman's supine position may cause the heavy uterus to compress her inferior vena cava, thus reducing blood return to her heart and reducing placental blood flow. Elevating her legs will not relieve the pressure from the inferior vena cava. If the woman is allowed to stay in the supine position and blood flow to the placental is reduced significantly, then fetal tachycardia may occur. The most appropriate nursing action is to prevent this from occurring by turning the woman to her side. Blood pressure readings may be obtained when the client is in the appropriate and safest position.

The nurse is caring for a client in early labor. Membranes ruptured approximately 2 hours earlier. This client is at increased risk for which complication? a.Intrauterine infection b.Hemorrhage c.Precipitous labor d.Supine hypotension

ANS: A When the membranes rupture, microorganisms from the vagina can ascend into the amniotic sac, causing chorioamnionitis and placentitis. ROM is not associated with fetal or maternal bleeding. Although ROM may increase the intensity of the contractions and facilitate active labor, it does not result in precipitous labor. ROM has no correlation with supine hypotension.

Conscious relaxation is associated with which method of childbirth preparation? a.Grantly Dick-Read childbirth method b.Lamaze method c.Bradley method d.Psychoprophylactic method

ANS: A With the Grantly Dick-Read method, women are taught to consciously and progressively relax different muscle groups throughout the body until a high degree of skill at relaxation is achieved. The Lamaze method combines controlled muscular relaxation with breathing techniques. The Bradley method advocates natural labor, without any form of anesthesia or analgesia, assisted by a husband-coach and using breathing techniques for labor. The psychoprophylactic method is another name for the Lamaze method.

The nurse should be aware of which information related to a woman's intake and output during labor? a.Traditionally, restricting the laboring woman to clear liquids and ice chips is being challenged because regional anesthesia is used more often than general anesthesia. b.Intravenous (IV) fluids are usually necessary to ensure that the laboring woman stays hydrated. c.Routine use of an enema empties the rectum and is very helpful for producing a clean, clear delivery. d.When a nulliparous woman experiences the urge to defecate, it often means birth will quickly follow.

ANS: A Women are awake with regional anesthesia and are able to protect their own airway, which reduces the worry over aspiration. Routine IV fluids during labor are unlikely to be beneficial and may be harmful. The routine use of an enema is, at best, ineffective and may be harmful. Having the urge to defecate followed by the birth of her fetus is true for a multiparous woman but not for a nulliparous woman.

13. The nurse knows that the best time to assess a woman's blood pressure during an initial prenatal visit is: A. At the end of the examination when she will be the most relaxed. B. At the beginning of the interview as a non threatening method of gaining rapport. C. During the middle of the physical examination when she is the most comfortable. D. Before beginning the pelvic examination because her blood pressure will be higher after the pelvic examination.

ANS: A Assess the woman's blood pressure at the end of the examination, when it is hoped that she will be most relaxed. The other options are not correct.

18. When performing an examination of a woman who is 34 weeks pregnant, the nurse notices that as the woman raises her head and shoulders off of the bed there is a midline linear protrusion in the abdomen over the area of the rectus abdominis muscles. Which response by the nurse is correct? A. Document the presence of diastasis recti. B. Discuss this condition with the physician because it will most likely need to be surgically repaired. C. Suspect that the woman has a hernia from the increased pressure within the abdomen from pregnancy. D. Tell the woman that she may have a difficult time with delivery because of the weakness in her abdominal muscles.

ANS: A The separation of the abdominal muscles is called diastasis recti. It frequently occurs during pregnancy. The rectus abdominis muscles will return together after pregnancy with abdominal exercise. This condition is not a true hernia

15. Which of these is considered a normal and expected finding when the nurse is performing a physical examination on a pregnant woman? A. A palpable, full thyroid B. Edema in one lower leg C. Significant diffuse enlargement of the thyroid D. Pale mucous membranes of the mouth

ANS: A The thyroid may be palpable during pregnancy. It should feel full, but smooth. Significant diffuseenlargement occurs with hyperthyroidism, thyroiditis, and hypothyroidism. Pale mucous membranes may indicate anemia. Bilateral lower extremity edema is common in pregnancy, but edema with pain in only one leg occurs with deep vein thrombosis

Women who have participated in childbirth education classes often bring a birth plan with them to the hospital. Which items might this plan include? (Select all that apply.) a.Presence of companions b.Clothing to be worn c.Care and handling of the newborn d.Medical interventions e.Date of delivery

ANS: A, B, C, D The presence of companions, clothing to be worn, care and handling of the newborn, medical interventions, and environmental modifications all might be included in the couple's birth plan. Other items include the presence of nonessential medical personnel (students), labor activities such as the tub or ambulation, preferred comfort and relaxation methods, and any cultural or religious requirements. The expected date of delivery would not be part of a birth plan unless the client is scheduled for an elective cesarean birth.

What are the complications and risks associated with cesarean births? (Select all that apply.) a.Pulmonary edema b.Wound dehiscence c.Hemorrhage d.Urinary tract infections e.Fetal injuries

ANS: A, B, C, D, E Pulmonary edema, wound dehiscence, hemorrhage, urinary tract infections, and fetal injuries are possible complications and risks associated with cesarean births.

Women who are obese are at risk for several complications during pregnancy and birth. Which of these would the nurse anticipate with an obese client? (Select all that apply.) a.Thromboembolism b.Cesarean birth c.Wound infection d.Breech presentation e.Hypertension

ANS: A, B, C, E A breech presentation is not a complication of pregnancy or birth for the client who is obese. Venous thromboembolism is a known risk for obese women. Therefore, the use of thromboembolism-deterrent (TED) hose and sequential compression devices may help decrease the chance for clot formation. Women should also be encouraged to ambulate as soon as possible. In addition to having an increased risk for complications with a cesarean birth, in general, obese women are also more likely to require an emergency cesarean birth. Many obese women have a pannus (i.e., large roll of abdominal fat) that overlies a lower transverse incision made just above the pubic area. The pannus causes the area to remain moist, which encourages the development of infection. Obese women are more likely to begin pregnancy with comorbidities such as hypertension and type 2 diabetes.

Emergency conditions during labor that would require immediate nursing intervention can arise with startling speed. Which situations are examples of such an emergency? (Select all that apply.) a.Nonreassuring or abnormal FHR pattern b.Inadequate uterine relaxation c.Vaginal bleeding d.Prolonged second stage e.Prolapse of the cord

ANS: A, B, C, E A nonreassuring or abnormal FHR pattern, inadequate uterine relaxation, vaginal bleeding, infection, and cord prolapse all constitute an emergency during labor that requires immediate nursing intervention. A prolonged second stage of labor after the upper limits for duration is reached. This is 3 hours for nulliparous women and 2 hours for multiparous women.

Which alternative approaches to relaxation have proven successful when working with the client in labor? (Select all that apply.) a.Aromatherapy b.Massage c.Hypnosis d.Cesarean birth e.Biofeedback

ANS: A, B, C, E Approaches to relaxation can include neuromuscular relaxation, aromatherapy, music, massage, imagery, hypnosis, or touch relaxation. Cesarean birth is a method of delivery, not a method of relaxation.

While developing an intrapartum care plan for the client in early labor, which psychosocial factors would the nurse recognize upon the client's pain experience? (Select all that apply.) a.Culture b.Anxiety and fear c.Previous experiences with pain d.Intervention of caregivers e.Support systems

ANS: A, B, C, E Culture: A woman's sociocultural roots influence how she perceives, interprets, and responds to pain during childbirth. Some cultures encourage loud and vigorous expressions of pain, whereas others value self-control. The nurse should avoid praising some behaviors (stoicism) while belittling others (noisy expression). Anxiety and fear: Extreme anxiety and fear magnify the sensitivity to pain and impair a woman's ability to tolerate it. Anxiety and fear increase muscle tension in the pelvic area, which counters the expulsive forces of uterine contractions and pushing efforts. Previous experiences with pain: Fear and withdrawal are a natural response to pain during labor. Learning about these normal sensations ahead of time helps a woman suppress her natural reactions of fear regarding the impending birth. If a woman previously had a long and difficult labor, she is likely to be anxious. She may also have learned ways to cope and may use these skills to adapt to the present labor experience. Support systems: An anxious partner is less able to provide help and support to a woman during labor. A woman's family and friends can be an important source of support if they convey realistic and positive information about labor and delivery. Although the intervention of caregivers may be necessary for the well-being of the woman and her fetus, some interventions add discomfort to the natural pain of labor (i.e., fetal monitor straps, IV lines).

At least five factors affect the process of labor and birth. These are easily remembered as the five Ps. Which factors are included in this process? (Select all that apply.) a.Passenger b.Passageway c.Powers d.Pressure e.Psychologic response

ANS: A, B, C, E The five Ps are passenger (fetus and placenta), passageway (birth canal), powers (contractions), position of the mother, and psychologic response. Pressure is not one of the five Ps.

A woman has requested an epidural block for her pain. She is 5 cm dilated and 100% effaced. The baby is in a vertex position and is engaged. The nurse increases the woman's IV fluid for a preprocedural bolus. Before the initiation of the epidural, the woman should be informed regarding the disadvantages of an epidural block. Which concerns should the nurse share with this client? (Select all that apply.) a.Ability to move freely is limited. b.Orthostatic hypotension and dizziness may occur. c.Gastric emptying is not delayed. d.Higher body temperature may occur. e.Blood loss is not excessive.

ANS: A, B, D The woman's ability to move freely and to maintain control of her labor is limited, related to the use of numerous medical interventions (IV lines and electronic fetal monitoring [EFM]). Significant disadvantages of an epidural block include the occurrence of orthostatic hypotension, dizziness, sedation, and leg weakness. Women who receive an epidural block have a higher body temperature (38° C or higher), especially when labor lasts longer than 12 hours, and may result in an unnecessary neonatal workup for sepsis. An advantage of an epidural block is that blood loss is not excessive. Other advantages include the following: the woman remains alert and able to participate, good relaxation is achieved, airway reflexes remain intact, and only partial motor paralysis develops.

3. The reported incidence of ectopic pregnancy has steadily risen over the past 2 decades. Causes include the increase in sexually transmitted infections (STIs) accompanied by tubal infection and damage. The popularity of contraceptive devices such as the IUD has also increased the risk for ectopic pregnancy. The nurse suspects that a client has early signs of ectopic pregnancy. The nurse should be observing the client for which signs or symptoms? (Select all that apply.) a. Pelvic pain b. Abdominal pain c. Unanticipated heavy bleeding d. Vaginal spotting or light bleeding e. Missed period

ANS: A, B, D, E A missed period or spotting can be easily mistaken by the client as an early sign of pregnancy. More subtle signs depend on exactly where the implantation occurs. The nurse must be thorough in her assessment because pain is not a normal symptom of early pregnancy. As the fallopian tube tears open and the embryo is expelled, the client often exhibits severe pain accompanied by intraabdominal hemorrhage, which may progress to hypovolemic shock with minimal or even no external bleeding. In approximately one half of women, shoulder and neck pain results from irritation of the diaphragm from the hemorrhage. DIF: Cognitive Level: Apply REF: p. 676 TOP: Nursing Process: Assessment MSC: Client Needs: Physiologic Integrity

Which factors influence cervical dilation? (Select all that apply.) a.Strong uterine contractions b.Force of the presenting fetal part against the cervix c.Size of the woman d.Pressure applied by the amniotic sac e.Scarring of the cervix

ANS: A, B, D, E Dilation of the cervix occurs by the drawing upward of the musculofibrous components of the cervix, which is caused by strong uterine contractions. Pressure exerted by the amniotic fluid while the membranes are intact or by the force applied by the presenting part can also promote cervical dilation. Scarring of the cervix as a result of a previous infection or surgery may slow cervical dilation. Pelvic size or the size of the woman does not affect cervical dilation.

Diabetes refers to a group of metabolic diseases characterized by hyperglycemia resulting from defects in insulin action, insulin secretion, or both. Over time, diabetes causes significant changes in the microvascular and macrovascular circulations. What do these complications include? (Select all that apply.) a.Atherosclerosis b.Retinopathy c.Intrauterine fetal death (IUFD) d.Nephropathy e.Neuropathy f.Autonomic neuropathy

ANS: A, B, D, E These structural changes will most likely affect a variety of systems, including the heart, eyes, kidneys, and nerves. IUFD (stillbirth) remains a major complication of diabetes in pregnancy; however, this is a fetal complication.

2. One of the most important components of the physical assessment of the pregnant client is the determination of BP. Consistency in measurement techniques must be maintained to ensure that the nuances in the variations of the BP readings are not the result of provider error. Which techniques are important in obtaining accurate BP readings? (Select all that apply.) a. The client should be seated. b. The client's arm should be placed at the level of the heart. c. An electronic BP device should be used. d. The cuff should cover a minimum of 60% of the upper arm. e. The same arm should be used for every reading.

ANS: A, B, E BP readings are easily affected by maternal position. Ideally, the client should be seated. An alternative position is left lateral recumbent with the arm at the level of the heart. The arm should always be held in a horizontal position at approximately the level of the heart. The same arm should be used at every visit. The manual sphygmomanometer is the most accurate device. If manual and electronic devices are used in the care setting, then the nurse must use caution when interpreting the readings. A proper size cuff should cover at least 80% of the upper arm or be approximately 1.5 times the length of the upper arm. DIF: Cognitive Level: Apply REF: p. 658 TOP: Nursing Process: Assessment MSC: Client Needs: Physiologic Integrity

A serious but uncommon complication of undiagnosed or partially treated hyperthyroidism is a thyroid storm, which may occur in response to stress such as infection, birth, or surgery. What are the signs and symptoms of this emergency disorder? (Select all that apply.) a.Fever b.Hypothermia c.Restlessness d.Bradycardia e.Hypertension

ANS: A, C Fever, restlessness, tachycardia, vomiting, hypotension, and stupor are symptoms of a thyroid storm. Fever, not hypothermia; tachycardia, not bradycardia; and hypotension, not hypertension, are symptoms of thyroid storm.

Which changes take place in the woman's reproductive system, days or even weeks before the commencement of labor? (Select all that apply.) a.Lightening b.Exhaustion c.Bloody show d.Rupture of membranes e.Decreased fetal movement

ANS: A, C, D Signs that precede labor may include lightening, urinary frequency, backache, weight loss, surge of energy, bloody show, and rupture of membranes. Many women experience a burst of energy before labor. A decrease in fetal movement is an ominous sign that does not always correlate with labor.

18. The client being cared for has severe preeclampsia and is receiving a magnesium sulfate infusion. Which new finding would give the nurse cause for concern? a. Sleepy, sedated affect b. Respiratory rate of 10 breaths per minute c. DTRs of 2 d. Absent ankle clonus

ANS: B A respiratory rate of 10 breaths per minute indicates the client is experiencing respiratory depression from magnesium toxicity. Because magnesium sulfate is a CNS depressant, the client will most likely become sedated when the infusion is initiated. DTRs of 2 and absent ankle clonus are normal findings. DIF: Cognitive Level: Understand REF: p. 664 TOP: Nursing Process: Diagnosis MSC: Client Needs: Physiologic Integrity

35. A woman in her second trimester of pregnancy states, "I'm so glad the morning sickness is gone,but I feel so tired!" What action by the nurse is appropriate at this time? Select all that apply. A. Order a test of the woman's vitamin D level. B. Instruct her to get more rest during the day. C. Order a complete blood count. D. Order thyroid function studies. E. Offer HIV screening.

ANS: A, C, D Vitamin D is essential for maternal response to the calcium demands of the fetus for growth and bone development. Maternal anorexia and malaise are often associated with Vitamin D deficiency. In addition, iron deficiency is a common cause of anemia in pregnancy; a complete blood count will indicate whether this is present. Thyroid function should be done to check for maternal hypothyroidism, which can cause symptoms of lethargy in the mother and also have severe effects on the developing fetus. HIV screening should have been offered at the onset of her pregnancy. Instructing her to get more rest will not help in finding a possible cause of her tiredness.

2. Approximately 10% to 15% of all clinically recognized pregnancies end in miscarriage. What are possible causes of early miscarriage? (Select all that apply.) a. Chromosomal abnormalities b. Infections c. Endocrine imbalance d. Systemic disorders e. Varicella

ANS: A, C, D, E Infections are not a common cause of early miscarriage. At least 50% of pregnancy losses result from chromosomal abnormalities. Endocrine imbalances such as hypothyroidism or diabetes are also possible causes for early pregnancy loss. Other systemic disorders that may contribute to pregnancy loss include lupus and genetic conditions. Although infections are not a common cause of early miscarriage, varicella infection in the first trimester has been associated with pregnancy loss. DIF: Cognitive Level: Remember REF: p. 669 TOP: Nursing Process: Assessment MSC: Client Needs: Health Promotion and Maintenance

MULTIPLE RESPONSE 1. Which adverse prenatal outcomes are associated with the HELLP syndrome? (Select all that apply.) a. Placental abruption b. Placenta previa c. Renal failure d. Cirrhosis e. Maternal and fetal death

ANS: A, C, E The HELLP syndrome is associated with an increased risk for adverse perinatal outcomes, including placental abruption, acute renal failure, subcapsular hepatic hematoma, hepatic rupture, recurrent preeclampsia, preterm birth, and fetal and maternal death. The HELLP syndrome is associated with an increased risk for placental abruption, not placenta previa. It is also associated with an increased risk for hepatic hematoma, not cirrhosis. DIF: Cognitive Level: Analyze REF: p. 658 TOP: Nursing Process: Assessment MSC: Client Needs: Physiologic Integrity

The class of drugs known as opioid analgesics (butorphanol, nalbuphine) is not suitable for administration to women with known opioid dependence. The antagonistic activity could precipitate withdrawal symptoms (abstinence syndrome) in both mothers and newborns. Which signs would indicate opioid or narcotic withdrawal in the mother? (Select all that apply.) a.Yawning, runny nose b.Increase in appetite c.Chills or hot flashes d.Constipation e.Irritability, restlessness

ANS: A, C, E The woman experiencing maternal opioid withdrawal syndrome will exhibit yawning, runny nose, sneezing, anorexia, chills or hot flashes, vomiting, diarrhea, abdominal pain, irritability, restlessness, muscle spasms, weakness, and drowsiness. Assessing both the mother and the newborn and planning the care accordingly are important steps for the nurse to take

MULTIPLE RESPONSE 1. A client who has undergone a D&C for early pregnancy loss is likely to be discharged the same day. The nurse must ensure that her vital signs are stable, that bleeding has been controlled, and that the woman has adequately recovered from the administration of anesthesia. To promote an optimal recovery, what information should discharge teaching include? (Select all that apply.) a. Iron supplementation b. Resumption of intercourse at 6 weeks postprocedure c. Referral to a support group, if necessary d. Expectation of heavy bleeding for at least 2 weeks e. Emphasizing the need for rest

ANS: A, C, E The woman should be advised to consume a diet high in iron and protein. For many women, iron supplementation also is necessary. The nurse should acknowledge that the client has experienced a loss, however early. She can be taught to expect mood swings and possibly depression. Referral to a support group, clergy, or professional counseling may be necessary. Discharge teaching should emphasize the need for rest. Nothing should be placed in the vagina for 2 weeks after the procedure, including tampons and vaginal intercourse. The purpose of this recommendation is to prevent infection. Should infection occur, antibiotics may be prescribed. The client should expect a scant, dark discharge for 1 to 2 weeks. Should heavy, profuse, or bright bleeding occur, she should be instructed to contact her health care provider. DIF: Cognitive Level: Apply REF: p. 672 TOP: Nursing Process: Implementation MSC: Client Needs: Physiologic Integrity

A woman who is 39 weeks pregnant expresses fear about her impending labor and how she will manage. What is the nurse's ideal response? a."Don't worry about it. You'll do fine." b."It's normal to be anxious about labor. Let's discuss what makes you afraid." c."Labor is scary to think about, but the actual experience isn't." d."You can have an epidural. You won't feel anything."

ANS: B "It's normal to be anxious about labor. Let's discuss what makes you afraid" is a statement that allows the woman to share her concerns with the nurse and is a therapeutic communication tool. "Don't worry about it. You'll do fine" negates the woman's fears and is not therapeutic. "Labor is scary to think about, but the actual experience isn't" negates the woman's fears and offers a false sense of security. To suggest that every woman can have an epidural is untrue. A number of criteria must be met before an epidural is considered. Furthermore, many women still experience the feeling of pressure with an epidural.

What is a maternal indication for the use of vacuum-assisted birth? a.Wide pelvic outlet b.Maternal exhaustion c.History of rapid deliveries d.Failure to progress past station 0

ANS: B A mother who is exhausted may be unable to assist with the expulsion of the fetus. The client with a wide pelvic outlet will likely not require vacuum extraction. With a rapid delivery, vacuum extraction is not necessary. A station of 0 is too high for a vacuum-assisted birth

What is the correct terminology for the nerve block that provides anesthesia to the lower vagina and perineum? a.Epidural b.Pudendal c.Local d.Spinal block

ANS: B A pudendal block anesthetizes the lower vagina and perineum to provide anesthesia for an episiotomy and the use of low forceps, if needed. An epidural provides anesthesia for the uterus, perineum, and legs. A local provides anesthesia for the perineum at the site of the episiotomy. A spinal block provides anesthesia for the uterus, perineum, and down the legs.

Which statement concerning the third stage of labor is correct? a.The placenta eventually detaches itself from a flaccid uterus. b.An expectant or active approach to managing this stage of labor reduces the risk of complications. c.It is important that the dark, roughened maternal surface of the placenta appears before the shiny fetal surface. d.The major risk for women during the third stage is a rapid heart rate.

ANS: B Active management facilitates placental separation and expulsion, reducing the risk of complications. The placenta cannot detach itself from a flaccid (relaxed) uterus. Which surface of the placenta comes out first is not clinically important. The major risk for women during the third stage of labor is postpartum hemorrhaging.

A woman who is gravida 3 para 2 arrives on the intrapartum unit. What is the most important nursing assessment at this time? a.Contraction pattern, amount of discomfort, and pregnancy history b.FHR, maternal vital signs, and the woman's nearness to birth c.Identification of ruptured membranes, woman's gravida and para, and her support person d.Last food intake, when labor began, and cultural practices the couple desires

ANS: B All options describe relevant intrapartum nursing assessments; however, this focused assessment has a priority. If the maternal and fetal conditions are normal and birth is not imminent, then other assessments can be performed in an unhurried manner; these include: gravida, para, support person, pregnancy history, pain assessment, last food intake, and cultural practices.

Under which circumstance would it be unnecessary for the nurse to perform a vaginal examination? a.Admission to the hospital at the start of labor b.When accelerations of the FHR are noted c.On maternal perception of perineal pressure or the urge to bear down d.When membranes rupture

ANS: B An accelerated FHR is a positive sign; therefore, a vaginal examination would not be necessary. A vaginal examination should be performed when the woman is admitted to the hospital, when she perceives perineal pressure or the urge to bear down, when her membranes rupture, when a significant change in her uterine activity has occurred, or when variable decelerations of the FHR are noted.

Anxiety is commonly associated with pain during labor. Which statement regarding anxiety is correct? a.Even mild anxiety must be treated. b.Severe anxiety increases tension, increases pain, and then, in turn, increases fear and anxiety, and so on. c.Anxiety may increase the perception of pain, but it does not affect the mechanism of labor. d.Women who have had a painful labor will have learned from the experience and have less anxiety the second time because of increased familiarity.

ANS: B Anxiety and pain reinforce each other in a negative cycle that will slow the progress of labor. Mild anxiety is normal for a woman in labor and likely needs no special treatment other than the standard reassurances. Anxiety increases muscle tension and ultimately can sufficiently build to slow the progress of labor. Unfortunately, an anxious, painful first labor is likely to carry over, through expectations and memories, into an anxious and painful experience in the second pregnancy.

A number of methods can be used for inducing labor. Which cervical ripening method falls under the category of mechanical or physical? a.Prostaglandins are used to soften and thin the cervix. b.Labor can sometimes be induced with balloon catheters or laminaria tents. c.Oxytocin is less expensive and more effective than prostaglandins but creates greater health risks. d.Amniotomy can be used to make the cervix more favorable for labor.

ANS: B Balloon catheters or laminaria tents are mechanical means of ripening the cervix. Ripening the cervix, making it softer and thinner, increases the success rate of induced labor. Prostaglandin E1 is less expensive and more effective than oxytocin but carries a greater risk. Amniotomy is the artificial rupture of membranes, which is used to induce labor only when the cervix is already ripe.

Developing a realistic birth plan with the pregnant woman regarding her care is important for the nurse. How would the nurse explain the major advantage of nonpharmacologic pain management? a.Greater and more complete pain relief is possible. b.No side effects or risks to the fetus are involved. c.The woman will remain fully alert at all times. d.Labor will likely be more rapid.

ANS: B Because nonpharmacologic pain management does not include analgesics, adjunct drugs, or anesthesia, it is harmless to the mother and the fetus. However, pain relief is lessened with nonpharmacologic pain management during childbirth. Although the woman's alertness is not altered by medication, the increase in pain may decrease alertness. Pain management may or may not alter the length of labor. At times when pain is decreased, the mother relaxes and labor progresses at a quicker pace.

The nurse who elects to work in the specialty of obstetric care must have the ability to distinguish between preterm birth, preterm labor, and low birth weight. Which statement regarding this terminology is correct? a.Terms preterm birth and low birth weight can be used interchangeably. b.Preterm labor is defined as cervical changes and uterine contractions occurring between 20 and 37 weeks of gestation. c.Low birth weight is a newborn who weighs below 3.7 pounds. d.Preterm birth rate in the United States continues to increase.

ANS: B Before 20 weeks of gestation, the fetus is not viable (miscarriage); after 37 weeks, the fetus can be considered term. Although these terms are used interchangeably, they have different meanings: preterm birth describes the length of gestation (before 37 weeks), regardless of the newborn's weight; low birth weight describes only the infant's weight at the time of birth (2500 g or less), whenever it occurs. Low birth weight is anything below 2500 g or approximately pounds. In 2011, the preterm birth rate in the United States was 11.7 %; it has dropped every year since 2008.

The nurse who is caring for a woman hospitalized for hyperemesis gravidarum would expect the initial treatment to involve what? a.Corticosteroids to reduce inflammation b.Intravenous (IV) therapy to correct fluid and electrolyte imbalances c.Antiemetic medication, such as pyridoxine, to control nausea and vomiting d.Enteral nutrition to correct nutritional deficits

ANS: B Initially, the woman who is unable to down clear liquids by mouth requires IV therapy to correct fluid and electrolyte imbalances. Corticosteroids have been successfully used to treat refractory hyperemesis gravidarum, but they are not the expected initial treatment for this disorder. Pyridoxine is vitamin B6, not an antiemetic medication. Promethazine, a common antiemetic, may be prescribed. In severe cases of hyperemesis gravidarum, enteral nutrition via a feeding tube may be necessary to correct maternal nutritional deprivation but is not the initial treatment for this client.

An 18-year-old client who has reached 16 weeks of gestation was recently diagnosed with pregestational diabetes. She attends her centering appointment accompanied by one of her girlfriends. This young woman appears more concerned about how her pregnancy will affect her social life than her recent diagnosis of diabetes. A number of nursing diagnoses are applicable to assist in planning adequate care. What is the most appropriate diagnosis at this time? a.Risk for injury, to the fetus related to birth trauma b.Deficient knowledge, related to diabetic pregnancy management c.Deficient knowledge, related to insulin administration d.Risk for injury, to the mother related to hypoglycemia or hyperglycemia

ANS: B Before a treatment plan is developed or goals for the outcome of care are outlined, this client must come to an understanding of diabetes and the potential effects on her pregnancy. She appears more concerned about changes to her social life than adopting a new self-care regimen. Risk for injury to the fetus related to either placental insufficiency or birth trauma may come later in the pregnancy. At this time, the client is having difficulty acknowledging the adjustments that she needs to make to her lifestyle to care for herself during pregnancy. The client may not yet be on insulin. Insulin requirements increase with gestation. The importance of glycemic control must be part of health teaching for this client. However, she has not yet acknowledged that changes to her lifestyle need to be made and may not participate in the plan of care until understanding takes place.

2. A perinatal nurse is giving discharge instructions to a woman, status postsuction, and curettage secondary to a hydatidiform mole. The woman asks why she must take oral contraceptives for the next 12 months. What is the bestresponse by the nurse? a. "If you get pregnant within 1 year, the chance of a successful pregnancy is very small. Therefore, if you desire a future pregnancy, it would be better for you to use the most reliable method of contraception available." b. "The major risk to you after a molar pregnancy is a type of cancer that can be diagnosed only by measuring the same hormone that your body produces during pregnancy. If you were to get pregnant, then it would make the diagnosis of this cancer more difficult." c. "If you can avoid a pregnancy for the next year, the chance of developing a second molar pregnancy is rare. Therefore, to improve your chance of a successful pregnancy, not getting pregnant at this time is best." d. "Oral contraceptives are the only form of birth control that will prevent a recurrence of a molar pregnancy."

ANS: B Beta-human chorionic gonadotropin (beta-hCG) hormone levels are drawn for 1 year to ensure that the mole is completely gone. The chance of developing choriocarcinoma after the development of a hydatidiform mole is increased. Therefore, the goal is to achieve a zero human chorionic gonadotropin (hCG) level. If the woman were to become pregnant, then it may obscure the presence of the potentially carcinogenic cells. Women should be instructed to use birth control for 1 year after treatment for a hydatidiform mole. The rationale for avoiding pregnancy for 1 year is to ensure that carcinogenic cells are not present. Any contraceptive method except an intrauterine device (IUD) is acceptable. DIF: Cognitive Level: Apply REF: p. 679 TOP: Nursing Process: Planning | Nursing Process: Implementation MSC: Client Needs: Physiologic Integrity

A client with maternal phenylketonuria (PKU) has come to the obstetrical clinic to begin prenatal care. Why would this preexisting condition result in the need for closer monitoring during pregnancy? a.PKU is a recognized cause of preterm labor. b.The fetus may develop neurologic problems. c.A pregnant woman is more likely to die without strict dietary control. d.Women with PKU are usually mentally handicapped and should not reproduce

ANS: B Children born to women with untreated PKU are more likely to be born with mental retardation, microcephaly, congenital heart disease, and low birth weight. Maternal PKU has no effect on labor. Women without dietary control of PKU are more likely to miscarry or bear a child with congenital anomalies. Screening for undiagnosed maternal PKU at the first prenatal visit may be warranted, especially in individuals with a family history of the disorder, with low intelligence of an uncertain cause, or who have given birth to microcephalic infants.

11. Which laboratory marker is indicative of DIC? a. Bleeding time of 10 minutes b. Presence of fibrin split products c. Thrombocytopenia d. Hypofibrinogenemia

ANS: B Degradation of fibrin leads to the accumulation of multiple fibrin clots throughout the body's vasculature. Bleeding time in DIC is normal. Low platelets may occur but are not indicative of DIC because they may be the result from other coagulopathies. Hypofibrinogenemia occurs with DIC. DIF: Cognitive Level: Remember REF: p. 684 TOP: Nursing Process: Assessment MSC: Client Needs: Physiologic Integrity

Which statement is the best rationale for assessing the maternal vital signs between uterine contractions? a.During a contraction, assessing the fetal heart rate is the priority. b.Maternal circulating blood volume temporarily increases during contractions. c.Maternal blood flow to the heart is reduced during contractions. d.Vital signs taken during contractions are not accurate.

ANS: B During uterine contractions, blood flow to the placenta temporarily stops, causing a relative increase in the mother's blood volume, which, in turn, temporarily increases blood pressure and slows the pulse. Monitoring fetal responses to the contractions is important; however, this question concerns the maternal vital signs. Maternal blood flow is increased during a contraction. Vital signs are altered by contractions but are considered accurate for that period.

What is the primary difference between the labor of a nullipara and that of a multipara? a.Amount of cervical dilation b.Total duration of labor c.Level of pain experienced d.Sequence of labor mechanisms

ANS: B In a first-time pregnancy, the descent is usually slow but steady; in subsequent pregnancies, the descent is more rapid, resulting in a shorter duration of labor. Cervical dilation is the same for all labors. The level of pain is individual to the woman, not to the number of labors she has experienced. The sequence of labor mechanisms is the same with all labors.

17. Which maternal condition always necessitates delivery by cesarean birth? a. Marginal placenta previa b. Complete placenta previa c. Ectopic pregnancy d. Eclampsia

ANS: B In complete placenta previa, the placenta completely covers the cervical os. A cesarean birth is the acceptable method of delivery. The risk of fetal death occurring is due to preterm birth. If the previa is marginal (i.e., 2 cm or greater away from the cervical os), then labor can be attempted. A cesarean birth is not indicated for an ectopic pregnancy. Labor can be safely induced if the eclampsia is under control. DIF: Cognitive Level: Understand REF: p. 681 TOP: Nursing Process: Assessment MSC: Client Needs: Physiologic Integrity

19. What is the correct terminology for an abortion in which the fetus dies but is retained within the uterus? a. Inevitable abortion b. Missed abortion c. Incomplete abortion d. Threatened abortion

ANS: B Missed abortion refers to the retention of a dead fetus in the uterus. An inevitable abortion means that the cervix is dilating with the contractions. An incomplete abortion means that not all of the products of conception were expelled. With a threatened abortion, the woman has cramping and bleeding but no cervical dilation. DIF: Cognitive Level: Remember REF: p. 670 TOP: Nursing Process: Assessment MSC: Client Needs: Physiologic Integrity

What is the correct term describing the slight overlapping of cranial bones or shaping of the fetal head during labor? a.Lightening b.Molding c.Ferguson reflex d.Valsalva maneuver

ANS: B Molding also permits adaptation to various diameters of the maternal pelvis. Lightening is the mother's sensation of decreased abdominal distention, which usually occurs the week before labor. The Ferguson reflex is the contraction urge of the uterus after the stimulation of the cervix. The Valsalva maneuver describes conscious pushing during the second stage of labor.

What is the role of the nurse as it applies to informed consent? a.Inform the client about the procedure, and ask her to sign the consent form. b.Act as a client advocate, and help clarify the procedure and the options. c.Call the physician to see the client. d.Witness the signing of the consent form.

ANS: B Nurses play a part in the informed consent process by clarifying and describing procedures or by acting as the woman's advocate and asking the primary health care provider for further explanations. The physician is responsible for informing the woman of her options, explaining the procedure, and advising the client about potential risk factors. The physician must be present to explain the procedure to the client. However, the nurse's responsibilities go further than simply asking the physician to see the client. The nurse may witness the signing of the consent form. However, depending on the state's guidelines, the woman's husband or another hospital health care employee may sign as a witness.

1. A pregnant woman is being discharged from the hospital after the placement of a cervical cerclage because of a history of recurrent pregnancy loss, secondary to an incompetent cervix. Which information regarding postprocedural care should the nurse emphasize in the discharge teaching? a. Any vaginal discharge should be immediately reported to her health care provider. b. The presence of any contractions, rupture of membranes (ROM), or severe perineal pressure should be reported. c. The client will need to make arrangements for care at home, because her activity level will be restricted. d. The client will be scheduled for a cesarean birth.

ANS: B Nursing care should stress the importance of monitoring for the signs and symptoms of preterm labor. Vaginal bleeding needs to be reported to her primary health care provider. Bed rest is an element of care. However, the woman may stand for periods of up to 90 minutes, which allows her the freedom to see her physician. Home uterine activity monitoring may be used to limit the woman's need for visits and to monitor her status safely at home. The cerclage can be removed at 37 weeks of gestation (to prepare for a vaginal birth), or a cesarean birth can be planned. DIF: Cognitive Level: Apply REF: p. 675 TOP: Nursing Process: Planning | Nursing Process: Implementation MSC: Client Needs: Health Promotion and Maintenance

According to professional standards (the Association of Women's Health, Obstetric and Neonatal Nurses [AWHONN], 2007), which action cannot be performed by the nonanesthetist registered nurse who is caring for a woman with epidural anesthesia? a.Monitoring the status of the woman and fetus b.Initiating epidural anesthesia c.Replacing empty infusion bags with the same medication and concentrate d.Stopping the infusion, and initiating emergency measures

ANS: B Only qualified, licensed anesthesia care providers are permitted to insert a catheter, initiate epidural anesthesia, verify catheter placement, inject medication through the catheter, or alter the medication or medications including type, amount, or rate of infusion. The nonanesthetist nurse is permitted to monitor the status of the woman, the fetus, and the progress of labor. Replacement of the empty infusion bags or syringes with the same medication and concentration is permitted. If the need arises, the nurse may stop the infusion, initiate emergency measures, and remove the catheter if properly educated to do so. Complications can require immediate interventions. Nurses must be prepared to provide safe and effective care during an emergency situation.

What should the laboring client who receives an opioid antagonist be told to expect? a.Her pain will decrease. b.Her pain will return. c.She will feel less anxious. d.She will no longer feel the urge to push.

ANS: B Opioid antagonists such as naloxone (Narcan) promptly reverse the CNS-depressant effects of opioids. In addition, the antagonist counters the effect of the stress-induced levels of endorphins. An opioid antagonist is especially valuable if the labor is more rapid than expected and the birth is anticipated when the opioid is at its peak effect. The woman should be told that the pain that was relieved by the opioid analgesic will return with the administration of the opioid antagonist. Her pain level will increase rather than decrease. Opioid antagonists have no effect on anxiety levels. They are primarily administered to reverse the excessive CNS depression in the mother, newborn, or both. An opioid antagonist (e.g., naloxone) has no effect on the mother's urge or ability to push. The practice of giving lower doses of IV opioids has reduced the incidence and severity of opioid-induced CNS depression; therefore, opioid antagonists are used less frequently.

What is the rationale for the administration of an oxytocic (e.g., Pitocin, Methergine) after expulsion of the placenta? a.To relieve pain b.To stimulate uterine contraction c.To prevent infection d.To facilitate rest and relaxation

ANS: B Oxytocics stimulate uterine contractions, which reduce blood loss after the third stage of labor. Oxytocics are not used to treat pain, do not prevent infection, and do not facilitate rest and relaxation.

A new mother with a thyroid disorder has come for a lactation follow-up appointment. Which thyroid disorder is a contraindication for breastfeeding? a.Hyperthyroidism b.PKU c.Hypothyroidism d.Thyroid storm

ANS: B PKU is a cause of mental retardation in infants; mothers with PKU pass on phenylalanine and therefore should elect not to breastfeed. A woman with either hyperthyroidism or hypothyroidism would have no particular reason not to breastfeed. A thyroid storm is a complication of hyperthyroidism and is not a contraindication to breastfeeding.

16. In contrast to placenta previa, what is the most prevalent clinical manifestation of abruptio placentae? a. Bleeding b. Intense abdominal pain c. Uterine activity d. Cramping

ANS: B Pain is absent with placenta previa and may be agonizing with abruptio placentae. Bleeding may be present in varying degrees for both placental conditions. Uterine activity and cramping may be present with both placental conditions. DIF: Cognitive Level: Understand REF: p. 683 TOP: Nursing Process: Diagnosis MSC: Client Needs: Physiologic Integrity

Preconception counseling is critical in the safe management of diabetic pregnancies. Which complication is commonly associated with poor glycemic control before and during early pregnancy? a.Frequent episodes of maternal hypoglycemia b.Congenital anomalies in the fetus c.Hydramnios d.Hyperemesis gravidarum

ANS: B Preconception counseling is particularly important since strict metabolic control before conception and in the early weeks of gestation is instrumental in decreasing the risk of congenital anomalies. Frequent episodes of maternal hypoglycemia may occur during the first trimester (not before conception) as a result of hormonal changes and the effects on insulin production and use. Hydramnios occurs approximately 10 times more often in diabetic pregnancies than in nondiabetic pregnancies. Typically, it is observed in the third trimester of pregnancy. Hyperemesis gravidarum may exacerbate hypoglycemic events because the decreased food intake by the mother and glucose transfer to the fetus contribute to hypoglycemia.

A first-time mother is concerned about the type of medications she will receive during labor. The client is in a fair amount of pain and is nauseated. In addition, she appears to be very anxious. The nurse explains that opioid analgesics are often used along with sedatives. How should the nurse phrase the rationale for this medication combination? a."The two medications, together, reduce complications." b."Sedatives enhance the effect of the pain medication." c."The two medications work better together, enabling you to sleep until you have the baby." d."This is what your physician has ordered for you."

ANS: B Sedatives may be used to reduce the nausea and vomiting that often accompany opioid use. In addition, some ataractic drugs reduce anxiety and apprehension and potentiate the opioid analgesic affects. A potentiator may cause two drugs to work together more effectively, but it does not ensure zero maternal or fetal complications. Sedation may be a related effect of some ataractic drugs; however, sedation is not the goal. Furthermore, a woman is unlikely to be able to sleep through transitional labor and birth. Although the physician may have ordered the medication, "This is what your physician has ordered for you" is not an acceptable comment for the nurse to make.

After an emergency birth, the nurse encourages the woman to breastfeed her newborn. What is the primary purpose of this activity? a.To facilitate maternal-newborn interaction b.To stimulate the uterus to contract c.To prevent neonatal hypoglycemia d.To initiate the lactation cycle

ANS: B Stimulation of the nipples through breastfeeding or manual stimulation causes the release of oxytocin and prevents maternal hemorrhage. Breastfeeding facilitates maternal-newborn interaction, but it is not the primary reason a woman is encouraged to breastfeed after an emergency birth. The primary intervention for preventing neonatal hypoglycemia is thermoregulation. Cold stress can result in hypoglycemia. The woman is encouraged to breastfeed after an emergency birth to stimulate the release of oxytocin, which prevents hemorrhaging. Breastfeeding is encouraged to initiate the lactation cycle, but it is not the primary reason for this activity after an emergency birth.

Which nursing intervention is paramount when providing care to a client with preterm labor who has received terbutaline? a.Assess deep tendon reflexes (DTRs). b.Assess for dyspnea and crackles. c.Assess for bradycardia. d.Assess for hypoglycemia.

ANS: B Terbutaline is a beta2-adrenergic agonist that affects the mother's cardiopulmonary and metabolic systems. Signs of cardiopulmonary decompensation include adventitious breath sounds and dyspnea. An assessment for dyspnea and crackles is important for the nurse to perform if the woman is taking magnesium sulfate. Assessing DTRs does not address the possible respiratory side effects of using terbutaline. Since terbutaline is a beta2-adrenergic agonist, it can lead to hyperglycemia, not hypoglycemia. Beta2-adrenergic agonist drugs cause tachycardia, not bradycardia.

Which description of the phases of the first stage of labor is most accurate? a.Latent: mild, regular contractions; no dilation; bloody show b. Active: moderate, regular contractions; 4 to 7 cm dilation c.Lull: no contractions; dilation stable d.Transition: very strong but irregular contractions; 8 to 10 cm dilation

ANS: B The active phase is characterized by moderate and regular contractions, 4 to 7 cm dilation, and duration of 3 to 6 hours. The latent phase is characterized by mild-to-moderate and irregular contractions, dilation up to 3 cm, brownish-to-pale pink mucus, and duration of 6 to 8 hours. No official "lull" phase exists in the first stage. The transition phase is characterized by strong to very strong and regular contractions, 8 to 10 cm dilation, and duration of 20 to 40 minutes.

Nurses can help their clients by keeping them informed about the distinctive stages of labor. Which description of the phases of the first stage of labor is accurate? a.Latent: Mild, regular contractions; no dilation; bloody show; duration of 2 to 4 hours b.Active: Moderate, regular contractions; 4- to 7-cm dilation; duration of 3 to 6 hours c.Lull: No contractions; dilation stable; duration of 20 to 60 minutes d.Transition: Very strong but irregular contractions; 8- to 10-cm dilation; duration of 1 to 2 hours

ANS: B The active phase is characterized by moderate, regular contractions; 4- to 7-cm dilation; and a duration of 3 to 6 hours. The latent phase is characterized by mild-to-moderate and irregular contractions; dilation up to 3 cm; brownish-to-pale pink mucus, and a duration of 6 to 8 hours. No official "lull" phase exists in the first stage. The transition phase is characterized by strong- to-very strong and regular contractions; 8- to 10-cm dilation; and a duration of 20 to 40 minutes.

The nurse should be aware of what important information regarding systemic analgesics administered during labor? a.Systemic analgesics cross the maternal blood-brain barrier as easily as they do the fetal blood-brain barrier. b.Effects on the fetus and newborn can include decreased alertness and delayed sucking. c.Intramuscular (IM) administration is preferred over IV administration. d.IV patient-controlled analgesia (PCA) results in increased use of an analgesic.

ANS: B The effects of analgesics depend on the specific drug administered, the dosage, and the timing. Systemic analgesics cross the fetal blood-brain barrier more readily than the maternal blood-brain barrier. IV administration is preferred over IM administration because the drug acts faster and more predictably. PCA results in a decrease in the use of an analgesic.

A woman who has a history of sexual abuse may have a number of traumatic memories triggered during labor. She may fight the labor process and react with pain or anger. The nurse can implement a number of care measures to help her client view the childbirth experience in a positive manner. Which intervention is key for the nurse to use while providing care? a.Tell the client to relax and that it won't hurt much. b.Limit the number of procedures that invade her body. c.Reassure the client that, as the nurse, you know what is best. d.Allow unlimited care providers to be with the client

ANS: B The number of invasive procedures such as vaginal examinations, internal monitoring, and IV therapy should be limited as much as possible. The nurse should always avoid words and phrases that may result in the client's recalling the phrases of her abuser (i.e., "Relax, this won't hurt" or "Just open your legs"). The woman's sense of control should be maintained at all times. The nurse should explain procedures at the client's pace and wait for permission to proceed. Protecting the client's environment by providing privacy and limiting the number of staff who observe the client will help to make her feel safe.

4. A 26-year-old pregnant woman, gravida 2, para 1-0-0-1, is 28 weeks pregnant when she experiences bright red, painless vaginal bleeding. On her arrival at the hospital, which diagnostic procedure will the client most likely have performed? a. Amniocentesis for fetal lung maturity b. Transvaginal ultrasound for placental location c. Contraction stress test (CST) d. Internal fetal monitoring

ANS: B The presence of painless bleeding should always alert the health care team to the possibility of placenta previa, which can be confirmed through ultrasonography. Amniocentesis is not performed on a woman who is experiencing bleeding. In the event of an imminent delivery, the fetus is presumed to have immature lungs at this gestational age, and the mother is given corticosteroids to aid in fetal lung maturity. A CST is not performed at a preterm gestational age. Furthermore, bleeding is a contraindication to a CST. Internal fetal monitoring is also contraindicated in the presence of bleeding. DIF: Cognitive Level: Apply REF: p. 680 TOP: Nursing Process: Assessment MSC: Client Needs: Health Promotion and Maintenance

Which statement related to fetal positioning during labor is correct and important for the nurse to understand? a.Position is a measure of the degree of descent of the presenting part of the fetus through the birth canal. b.Birth is imminent when the presenting part is at +4 to +5 cm below the spine. c.The largest transverse diameter of the presenting part is the suboccipitobregmatic diameter. d.Engagement is the term used to describe the beginning of labor.

ANS: B The station of the presenting part should be noted at the beginning of labor to determine the rate of descent. Position is the relationship of the presenting part of the fetus to the four quadrants of the mother's pelvis; station is the measure of degree of descent. The largest diameter is usually the biparietal diameter. The suboccipitobregmatic diameter is the smallest, although one of the most critical. Engagement often occurs in the weeks just before labor in nulliparous women and before or during labor in multiparous women.

The nurse has received a report regarding a client in labor. The woman's last vaginal examination was recorded as 3 cm, 30%, and -2. What is the nurse's interpretation of this assessment? a.Cervix is effaced 3 cm and dilated 30%; the presenting part is 2 cm above the ischial spines. b.Cervix is dilated 3 cm and effaced 30%; the presenting part is 2 cm above the ischial spines. c.Cervix is effaced 3 cm and dilated 30%; the presenting part is 2 cm below the ischial spines. d.Cervix is dilated 3 cm and effaced 30%; the presenting part is 2 cm below the ischial spines.

ANS: B The sterile vaginal examination is recorded as centimeters of cervical dilation, percentage of cervical dilation, and the relationship of the presenting part to the ischial spines (either above or below). For this woman, the cervix is dilated 3 cm and effaced 30%, and the presenting part is 2 cm above the ischial spines. The first interpretation of this vaginal examination is incorrect; the cervix is dilated 3 cm and is 30% effaced. However, the presenting part is correct at 2 cm above the ischial spines. The remaining two interpretations of this vaginal examination are incorrect. Although the dilation and effacement are correct at 3 cm and 30%, the presenting part is actually 2 cm above the ischial spines.

What is the rationale for the use of a blood patch after spinal anesthesia? a.Hypotension b.Headache c.Neonatal respiratory depression d.Loss of movement

ANS: B The subarachnoid block may cause a postspinal headache resulting from the loss of cerebrospinal fluid from the puncture in the dura. When blood is injected into the epidural space in the area of the dural puncture, it forms a seal over the hole to stop the leaking of cerebrospinal fluid. Hypotension is prevented by increasing fluid volume before the procedure. Neonatal respiratory depression is not an expected outcome with spinal anesthesia. Loss of movement is an expected outcome of spinal anesthesia.

A 26-year-old primigravida has come to the clinic for her regular prenatal visit at 12 weeks. She appears thin and somewhat nervous. She reports that she eats a well-balanced diet, although her weight is 5 pounds less than it was at her last visit. The results of laboratory studies confirm that she has a hyperthyroid condition. Based on the available data, the nurse formulates a plan of care. Which nursing diagnosis is most appropriate for the client at this time? a.Deficient fluid volume b.Imbalanced nutrition: less than body requirements c.Imbalanced nutrition: more than body requirements d.Disturbed sleep pattern

ANS: B This client's clinical cues include weight loss, which supports a nursing diagnosis of "Imbalanced nutrition: less than body requirements." No clinical signs or symptoms support a nursing diagnosis of deficient fluid volume. This client reports weight loss, not weight gain. Although the client reports nervousness, the most appropriate nursing diagnosis, based on the client's other clinical symptoms, is "Imbalanced nutrition: less than body requirements."

A pregnant woman at 29 weeks of gestation has been diagnosed with preterm labor. Her labor is being controlled with tocolytic medications. She asks when she might be able to go home. Which response by the nurse is most accurate? a."After the baby is born." b."When we can stabilize your preterm labor and arrange home health visits." c."Whenever your physician says that it is okay." d."It depends on what kind of insurance coverage you have."

ANS: B This client's preterm labor is being controlled with tocolytics. Once she is stable, home care may be a viable option for this type of client. Care of a client with preterm labor is multidisciplinary and multifactorial; the goal is to prevent delivery. In many cases, this goal may be achieved at home. Managed care may dictate an earlier hospital discharge or a shift from hospital to home care. Insurance coverage may be one factor in client care, but ultimately, client safety remains the most important factor.

Through a vaginal examination, the nurse determines that a woman is 4 cm dilated. The external fetal monitor shows uterine contractions every to 4 minutes. The nurse reports this as what stage of labor? a.First stage, latent phase b.First stage, active phase c.First stage, transition phase d.Second stage, latent phase

ANS: B This maternal progress indicates that the woman is in the active phase of the first stage of labor. During the latent phase of the first stage of labor, the expected maternal progress is 0 to 3 cm dilation with contractions every 5 to 30 minutes. During the transition phase of the first stage of labor, the expected maternal progress is 8 to 10 cm dilation with contractions every 2 to 3 minutes. During the latent phase of the second stage of labor, the woman is completely dilated and experiences a restful period of "laboring down."

In terms of the incidence and classification of diabetes, which information should the nurse keep in mind when evaluating clients during their ongoing prenatal appointments? a.Type 1 diabetes is most common. b.Type 2 diabetes often goes undiagnosed. c.GDM means that the woman will receive insulin treatment until 6 weeks after birth. d.Type 1 diabetes may become type 2 during pregnancy.

ANS: B Type 2 diabetes often goes undiagnosed because hyperglycemia gradually develops and is often not severe. Type 2, sometimes called adult-onset diabetes, is the most common type of diabetes. GDM refers to any degree of glucose intolerance first recognized during pregnancy; insulin may or may not be needed. People do not go back and forth between type 1 and type 2 diabetes.

5. A laboring woman with no known risk factors suddenly experiences spontaneous ROM. The fluid consists of bright red blood. Her contractions are consistent with her current stage of labor. No change in uterine resting tone has occurred. The fetal heart rate (FHR) begins to decline rapidly after the ROM. The nurse should suspect the possibility of what condition? a. Placenta previa b. Vasa previa c. Severe abruptio placentae d. Disseminated intravascular coagulation (DIC)

ANS: B Vasa previa is the result of a velamentous insertion of the umbilical cord. The umbilical vessels are not surrounded by Wharton jelly and have no supportive tissue. The umbilical blood vessels thus are at risk for laceration at any time, but laceration occurs most frequently during ROM. The sudden appearance of bright red blood at the time of ROM and a sudden change in the FHR without other known risk factors should immediately alert the nurse to the possibility of vasa previa. The presence of placenta previa most likely would be ascertained before labor and is considered a risk factor for this pregnancy. In addition, if the woman had a placenta previa, it is unlikely that she would be allowed to pursue labor and a vaginal birth. With the presence of severe abruptio placentae, the uterine tonicity typically is tetanus (i.e., a boardlike uterus). DIC is a pathologic form of diffuse clotting that consumes large amounts of clotting factors, causing widespread external bleeding, internal bleeding, or both. DIC is always a secondary diagnosis, often associated with obstetric risk factors such as the hemolysis, elevated liver enzyme levels, and low platelet levels (HELLP) syndrome. This woman did not have any prior risk factors. DIF: Cognitive Level: Analyze REF: p. 684 TOP: Nursing Process: Diagnosis MSC: Client Needs: Physiologic Integrity

21. During a prenatal visit at 30 weeks of gestation, the patient mentions to the nurse, "The baby has not moved very much since yesterday." The nurse notices that fetal movement is not as active as it was the last visit, and notifies the physician. What should the nurse prepare to do next? A. Admit the woman to a hospital. B. Administer a nonstress test for fetal monitoring. C. Start an intravenous infusion. D. Have the patient walk around the hallways and then check fetal movement again.

ANS: B A decrease in fetal movement should be reported and followed by a nonstress test (fetal monitoring). Cessation of fetal movement may be indicative of fetal distress or impending fetal death

29. A 25-year-old woman is in the clinic for her first prenatal visit. The nurse will prepare to obtain which laboratory screening test at this time? A. Urine toxicology B. Complete blood cell count C. Alpha-fetoprotein D. Carrier screening for cystic fibrosis

ANS: B At the onset of pregnancy, a routine prenatal panel usually includes a complete blood cell count, serology, rubella antibodies, hepatitis B screening, blood type and Rhesus factor, and antibody screen. A clean-catch urine sample is collected for urinalysis to rule out cystitis. Urine toxicologyis beneficial for women if active substance abuse is suspected or known, but it not routinely done. In the second trimester, maternal serum is analyzed for alpha-fetoprotein. Carrier screeningfor cystic fibrosis is offered to check whether a person carries the abnormal gene that causes cystic fibrosis but is not part of routine testing

14. When examining the face of a woman who is 28 weeks pregnant, the nurse notices the presence of a butterfly-shaped increase in pigmentation on the face. The proper term for this finding in the documentation is: A. Striae. B. Chloasma. C. Linea nigra. D. The mask of pregnancy

ANS: B Chloasma is a butterfly-shaped increase in pigmentation on the face. It is known as the mask of pregnancy, but when documenting, the nurse should use the correct medical term, "chloasma." "Striae" is the term for stretch marks. The linea nigra is a hyperpigmented line that begins at the sternal notch and extends down the abdomen through the umbilicus to the pubis

30. A woman in her 25th week of gestation comes to the clinic for her prenatal visit. The nurse notices that her face and lower extremities are swollen, and her blood pressure is 154/94 mm Hg. She states that she has had headaches and blurry vision but thought she was just tired. What should the nurse suspect? A. Eclampsia B. Preeclampsia C. Diabetes type 1 D. Preterm labor

ANS: B Classic symptoms of preeclampsia include elevated blood pressure (greater than 140 systolic or 90 diastolic mm Hg in a woman with previously normal blood pressure) and proteinuria. Onset and worsening symptoms may be sudden, and subjective signs include headaches and visual changes. Eclampsia is manifested by generalized tonic-clonic seizures. These symptoms are not indicative of diabetes mellitus (type 1 or 2) or preterm labor.

5. A patient is being seen at the clinic for her 10-week prenatal visit. She asks when she will be able to hear the baby's heartbeat. The nurse should reply: A. "The baby's heartbeat is not usually heard until the second trimester." B. "The baby's heartbeat may be heard anywhere from the ninth to the twelfth week." C."It is often difficult to hear the heartbeat at this point, but we can try." D. "It is normal to hear the heartbeat at six weeks. We may be able to hear it today."

ANS: B Fetal heart tones can be heard by use of the Doppler device between 9 and 12 weeks. The other responses are incorrect.

27. A patient who is 24 weeks pregnant asks about wearing a seatbelt while driving. Which response by the nurse is correct? A. "Seat belts should not be worn during pregnancy." B. "Place the lap belt below the uterus and use the shoulder strap at the same time." C. "Place the lap belt below the uterus but omit the shoulder strap during pregnancy." D. "Place the lap belt at your waist above the uterus and use the shoulder strap at the same time."

ANS: B For maternal and fetal safety, the nurse should instruct the woman to place the lap belt below the uterus and to use the shoulder strap. The other instructions are incorrect.

17. When the nurse is assessing the deep tendon reflexes (DTRs) on a woman who is 32 weeks pregnant, which of these would be considered a normal finding on a 0 to 4+ scale? A. Absent DTRs B. 2+ C. 4+ D. Brisk reflexes and the presence of clonus

ANS: B Normally during pregnancy the DTRs are 1+ to 2+ and equal bilaterally. Brisk or greater than 2+DTRs and the presence of clonus are abnormal and may be associated with an elevated blood pressure and cerebral edema in the preeclamptic woman

16. When auscultating the anterior thorax of a pregnant woman, the nurse notices the presence of a murmur over the second, third, and fourth intercostal spaces. It is continuous but can be obliterated by pressure with the stethoscope or finger on the thorax just lateral to the murmur. The nurse interprets this finding to be: A. The murmur of aortic stenosis. B. Most likely a mammary souffle. C. Associated with aortic insufficiency. D. An indication of a patent ductus arteriosus.

ANS: B Often blood flow through the blood vessels, specifically the internal mammary artery, can be heard over the second, third, and fourth intercostal spaces. This is called a mammary souffle, but it may be mistaken for a cardiac murmur. The other options are incorrect

12. When assessing a woman who is in her third trimester of pregnancy, the nurse looks for the classic symptoms associated with preeclampsia, which include: A. Edema, headaches, and seizures. B. Elevated blood pressure and proteinuria. C. Elevated liver enzymes and high platelet counts. D. Decreased blood pressure and edema.

ANS: B The classic symptoms of preeclampsia are hypertension and proteinuria. Headaches may occur with worsening symptoms, and seizures may occur if preeclampsia is untreated and leads to eclampsia. A serious variant of preeclampsia, the HELLP syndrome, involves hemolysis, elevated liver enzymes, and low platelet counts, and it is an ominous picture. Edema is a common occurrence in pregnancy.

28. During a history interview, a 38-year-old woman shares that she is thinking about having another baby. The nurse knows that which statement is true regarding pregnancy after age 35 years? A. Fertility does not start to decline until age 40 years. B. The occurrence of Down syndrome is much more frequent after age 35 years. C. Genetic counseling and prenatal screening are not routine until after age 40 years. D. Women older than 35 years who are pregnant have the same rate of pregnancy-related complications as those who are younger than 35 years.

ANS: B The risk of Down syndrome increases as the woman ages, from about 1 in 1250 at age 25 to 1 in 400 at age 35. Fertility declines with advancing maternal age. Women 35 years and older, or with a history of a genetic abnormality, are offered genetic counseling and the options of prenatal diagnostic screening tests. Because the incidence of chronic diseases increases with age, women older than 35 years who are pregnant more often have medical complications such as diabetes, obesity, and hypertension.

Maternal hypotension is a potential side effect of regional anesthesia and analgesia. What nursing interventions could the nurse use to increase the client's blood pressure? (Select all that apply.) a.Place the woman in a supine position. b.Place the woman in a lateral position. c.Increase IV fluids. d.Administer oxygen. e.Perform a vaginal examination.

ANS: B, C, D Nursing interventions for maternal hypotension arising from analgesia or anesthesia include turning the woman to a lateral position, increasing IV fluids, administering oxygen via face mask, elevating the woman's legs, notifying the physician, administering an IV vasopressor, and monitoring the maternal and fetal status at least every 5 minutes until the woman is stable. Placing the client in a supine position causes venous compression, thereby limiting blood flow to and oxygenation of the placenta and fetus. A sterile vaginal examination has no bearing on maternal blood pressure.

Indications for a primary cesarean birth are often nonrecurring. Therefore, a woman who has had a cesarean birth with a low transverse scar may be a candidate for vaginal birth after cesarean (VBAC). Which clients would beless likely to have a successful VBAC? (Select all that apply.) aLengthy interpregnancy interval b.African-American race c.Delivery at a rural hospital d.Estimated fetal weight <4000 g e.Maternal obesity (BMI >30)

ANS: B, C, E Indications for a low success rate for a VBAC delivery include a short interpregnancy interval, non-Caucasian race, gestational age longer than 40 weeks, maternal obesity, preeclampsia, fetal weight greater than 4000 g, and delivery at a rural or private hospital.

The nurse recognizes that uterine hyperstimulation with oxytocin requires emergency interventions. What clinical cues alert the nurse that the woman is experiencing uterine hyperstimulation? (Select all that apply.) a.Uterine contractions lasting <90 seconds and occurring >2 minutes in frequency b.Uterine contractions lasting >90 seconds and occurring <2 minutes in frequency c.Uterine tone <20 mm Hg d.Uterine tone >20 mm Hg e.Increased uterine activity accompanied by a nonreassuring FHR and pattern

ANS: B, D, E Uterine contractions that occur less frequently than 2 minutes apart and last longer than 90 seconds, a uterine tone over 20 mm Hg, and a nonreassuring FHR and pattern are indications of uterine hyperstimulation with oxytocin administration. Uterine contractions that occur more frequently than 2 minutes apart and last less than 90 seconds are the expected goal of oxytocin induction. A uterine tone less than 20 mm Hg is normal.

Which nursing assessment indicates that a woman who is in second-stage labor is almost ready to give birth? a.Fetal head is felt at 0 station during vaginal examination. b.Bloody mucous discharge increases. c.Vulva bulges and encircles the fetal head. d.Membranes rupture during a contraction.

ANS: C A bulging vulva that encircles the fetal head describes crowning, which occurs shortly before birth. Birth of the head occurs when the station is +4. A 0 station indicates engagement. Bloody show occurs throughout the labor process and is not an indication of an imminent birth. ROM can occur at any time during the labor process and does not indicate an imminent birth.

A woman in labor is breathing into a mouthpiece just before the start of her regular contractions. As she inhales, a valve opens and gas is released. She continues to inhale the gas slowly and deeply until the contraction starts to subside. When the inhalation stops, the valve closes. Which statement regarding this procedure is correct? a.The application of nitrous oxide gas is not often used anymore. b.An inhalation of gas is likely to be used in the second stage of labor, not during the first stage. c.An application of nitrous oxide gas is administered for pain relief. d.The application of gas is a prelude to a cesarean birth.

ANS: C A mixture of nitrous oxide with oxygen in a low concentration can be used in combination with other nonpharmacologic and pharmacologic measures for pain relief. This procedure is still commonly used in Canada and in the United Kingdom. Nitrous oxide inhaled in a low concentration will reduce but not eliminate pain during the first and second stages of labor. Nitrous oxide inhalation is not generally used before a caesarean birth. Nitrous oxide does not appear to depress uterine contractions or cause adverse reactions in the newborn.

Which preexisting factor is known to increase the risk of GDM? a.Underweight before pregnancy b.Maternal age younger than 25 years c.Previous birth of large infant d.Previous diagnosis of type 2 diabetes mellitus

ANS: C A previous birth of a large infant suggests GDM. Obesity (body mass index [BMI] of 30 or greater) creates a higher risk for gestational diabetes. A woman younger than 25 years is not generally at risk for GDM. The person with type 2 diabetes mellitus already has diabetes and thus will continue to have it after pregnancy. Insulin may be required during pregnancy because oral hypoglycemia drugs are contraindicated during pregnancy.

15. A woman arrives at the emergency department with complaints of bleeding and cramping. The initial nursing history is significant for a last menstrual period 6 weeks ago. On sterile speculum examination, the primary care provider finds that the cervix is closed. The anticipated plan of care for this woman would be based on a probable diagnosis of which type of spontaneous abortion? a. Incomplete b. Inevitable c. Threatened d. Septic

ANS: C A woman with a threatened abortion has spotting, mild cramps, and no cervical dilation. A woman with an incomplete abortion would have heavy bleeding, mild-to-severe cramping, and cervical dilation. An inevitable abortion demonstrates the same symptoms as an incomplete abortion: heavy bleeding, mild-to-severe cramping, and cervical dilation. A woman with a septic abortion has malodorous bleeding and typically a dilated cervix. DIF: Cognitive Level: Understand REF: p. 670 TOP: Nursing Process: Planning MSC: Client Needs: Physiologic Integrity

Which statement is not an expected outcome for the client who attends a reputable childbirth preparation program? a.Childbirth preparation programs increase the woman's sense of control. b.Childbirth preparation programs prepare a support person to help during labor. c.Childbirth preparation programs guarantee a pain-free childbirth. d.Childbirth preparation programs teach distraction techniques.

ANS: C All methods try to increase a woman's sense of control, prepare a support person, and train the woman in physical conditioning, which includes breathing techniques. These programs cannot, and reputable ones do not, promise a pain-free childbirth. Increasing a woman's sense of control is the goal of all childbirth preparation methods. Preparing a support person to help in labor is a vitally important component of any childbirth education program. The coach may learn how to touch a woman's body to detect tense and contracted muscles. The woman then learns how to relax in response to the gentle stroking by the coach. Distraction techniques are a form of care that are effective to some degree in relieving labor pain and are taught in many childbirth programs. These distractions include imagery, feedback relaxation, and attention-focusing behaviors.

A laboring woman has received meperidine (Demerol) intravenously (IV), 90 minutes before giving birth. Which medication should be available to reduce the postnatal effects of meperidine on the neonate? a.Fentanyl (Sublimaze) b.Promethazine (Phenergan) c.Naloxone (Narcan) d.Nalbuphine (Nubain)

ANS: C An opioid antagonist can be given to the newborn as one part of the treatment for neonatal narcosis, which is a state of central nervous system (CNS) depression in the newborn produced by an opioid. Opioid antagonists, such as naloxone (Narcan), can promptly reverse the CNS depressant effects, especially respiratory depression. Fentanyl (Sublimaze), promethazine (Phenergan), and nalbuphine (Nubain) do not act as opioid antagonists to reduce the postnatal effects of meperidine on the neonate.

6. A woman with worsening preeclampsia is admitted to the hospital's labor and birth unit. The physician explains the plan of care for severe preeclampsia, including the induction of labor, to the woman and her husband. Which statement by the husband leads the nurse to believe that the couple needs further information? a. "I will help my wife use the breathing techniques that we learned in our childbirth classes." b. "I will give my wife ice chips to eat during labor." c. "Since we will be here for a while, I will call my mother so she can bring the two boys—2 years and 4 years of age—to visit their mother." d. "I will stay with my wife during her labor, just as we planned."

ANS: C Arranging a visit with their two children indicates that the husband does not understand the importance of the quiet, subdued environment that is needed to prevent his wife's condition from worsening. Implementing breathing techniques is indicative of adequate knowledge related to pain management during labor. Administering ice chips indicates an understanding of nutritional needs during labor. Staying with his wife during labor demonstrates the husband's support for his wife and is appropriate. DIF: Cognitive Level: Apply REF: p. 662 TOP: Nursing Process: Evaluation MSC: Client Needs: Psychosocial Integrity

The nurse is performing an initial assessment of a client in labor. What is the appropriate terminology for the relationship of the fetal body parts to one another? a.Lie b.Presentation c.Attitude d.Position

ANS: C Attitude is the relationship of the fetal body parts to one another. Lie is the relationship of the long axis (spine) of the fetus to the long axis (spine) of the mother. Presentation refers to the part of the fetus that enters the pelvic inlet first and leads through the birth canal during labor at term. Position is the relationship of the presenting part of the fetus to the four quadrants of the mother's pelvis.

38-year-old female was recently diagnosed with cancer. She learns that her mother almost miscarried while pregnant with her and was given diethylstilbestrol (DES) to prevent it. Which of the following cancers that she most likely have? a. Endometrial b. Vaginal c. Cervical d. Uterine

B. Vaginal exposure in utero to nonsteroidal estrogens (DES) has been identified as a risk factor for vaginal cancer.

What is the primary purpose for the use of tocolytic therapy to suppress uterine activity? a.Drugs can be efficaciously administered up to the designated beginning of term at 37 weeks gestation. b.Tocolytic therapy has no important maternal (as opposed to fetal) contraindications. c.The most important function of tocolytic therapy is to provide the opportunity to administer antenatal glucocorticoids. d.If the client develops pulmonary edema while receiving tocolytic therapy, then intravenous (IV) fluids should be given.

ANS: C Buying time for antenatal glucocorticoids to accelerate fetal lung development may be the best reason to use tocolytic therapy. Once the pregnancy has reached 34 weeks, however, the risks of tocolytic therapy outweigh the benefits. Important maternal contraindications to tocolytic therapy exist. Tocolytic-induced edema can be caused by IV fluids.

20. What condition indicates concealed hemorrhage when the client experiences abruptio placentae? a. Decrease in abdominal pain b. Bradycardia c. Hard, boardlike abdomen d. Decrease in fundal height

ANS: C Concealed hemorrhage occurs when the edges of the placenta do not separate. The formation of a hematoma behind the placenta and subsequent infiltration of the blood into the uterine muscle results in a very firm, boardlike abdomen. Abdominal pain may increase. The client will have shock symptoms that include tachycardia. As bleeding occurs, the fundal height increases. DIF: Cognitive Level: Analyze REF: p. 683 TOP: Nursing Process: Assessment MSC: Client Needs: Physiologic Integrity

6. A woman arrives for evaluation of signs and symptoms that include a missed period, adnexal fullness, tenderness, and dark red vaginal bleeding. On examination, the nurse notices an ecchymotic blueness around the woman's umbilicus. What does this finding indicate? a. Normal integumentary changes associated with pregnancy b. Turner sign associated with appendicitis c. Cullen sign associated with a ruptured ectopic pregnancy d. Chadwick sign associated with early pregnancy

ANS: C Cullen sign, the blue ecchymosis observed in the umbilical area, indicates hematoperitoneum associated with an undiagnosed ruptured intraabdominal ectopic pregnancy. Linea nigra on the abdomen is the normal integumentary change associated with pregnancy and exhibits a brown pigmented, vertical line on the lower abdomen. Turner sign is ecchymosis in the flank area, often associated with pancreatitis. A Chadwick sign is a blue-purple cervix that may be seen during or around the eighth week of pregnancy. DIF: Cognitive Level: Analyze REF: p. 676 TOP: Nursing Process: Assessment MSC: Client Needs: Physiologic Integrity

During a prenatal visit, the nurse is explaining dietary management to a woman with pregestational diabetes. Which statement by the client reassures the nurse that teaching has been effective? a."I will need to eat 600 more calories per day because I am pregnant." b."I can continue with the same diet as before pregnancy as long as it is well balanced." c."Diet and insulin needs change during pregnancy." d."I will plan my diet based on the results of urine glucose testing."

ANS: C Diet and insulin needs change during the pregnancy in direct correlation to hormonal changes and energy needs. In the third trimester, insulin needs may double or even quadruple. The diet is individualized to allow for increased fetal and metabolic requirements, with consideration of such factors as prepregnancy weight and dietary habits, overall health, ethnic background, lifestyle, stage of pregnancy, knowledge of nutrition, and insulin therapy. Energy needs are usually calculated on the basis of 30 to 35 calories per kilogram of ideal body weight. Dietary management during a diabetic pregnancy must be based on blood, not urine, glucose changes.

Which nursing assessment indicates that a woman who is in second-stage labor is almost ready to give birth? a.Fetal head is felt at 0 station during the vaginal examination. b.Bloody mucous discharge increases. c.Vulva bulges and encircles the fetal head. d.Membranes rupture during a contraction.

ANS: C During the active pushing (descent) phase, the woman has strong urges to bear down as the presenting part of the fetus descends and presses on the stretch receptors of the pelvic floor. The vulva stretches and begins to bulge, encircling the fetal head. Birth of the head occurs when the station is +4. A 0 station indicates engagement. Bloody show occurs throughout the labor process and is not an indication of an imminent birth. Rupture of membranes can occur at any time during the labor process and does not indicate an imminent birth.

Which clinical finding indicates that the client has reached the second stage of labor? a.Amniotic membranes rupture. b.Cervix cannot be felt during a vaginal examination. c.Woman experiences a strong urge to bear down. d.Presenting part of the fetus is below the ischial spines.

ANS: C During the descent phase of the second stage of labor, the woman may experience an increase in the urge to bear down. The ROM has no significance in determining the stage of labor. The second stage of labor begins with full cervical dilation. Many women may have an urge to bear down when the presenting fetal part is below the level of the ischial spines. This urge can occur during the first stage of labor, as early as with 5 cm dilation.

When assessing the fetus using Leopold's maneuvers, the nurse feels a round, firm, and movable fetal part in the fundal portion of the uterus and a long, smooth surface in the mother's right side close to midline. What is the position of the fetus? a.ROA b.LSP c.RSA d.LOA

ANS: C Fetal position is denoted with a three-letter abbreviation. The first letter indicates the presenting part in either the right or the left side of the maternal pelvis. The second letter indicates the anatomic presenting part of the fetus. The third letter stands for the location of the presenting part in relationship to the anterior, posterior, or transverse portion of the maternal pelvis. Palpation of a round, firm fetal part in the fundal portion of the uterus would be the fetal head, indicating that the fetus is in a breech position with the sacrum as the presenting part in the maternal pelvis. Palpation of the fetal spine along the mother's right side denotes the location of the presenting part in the mother's pelvis. The ability to palpate the fetal spine indicates that the fetus is anteriorly positioned in the maternal pelvis. This fetus is anteriorly positioned in the right side of the maternal pelvis with the sacrum as the presenting part. RSA is the correct three-letter abbreviation to indicate this fetal position. ROA denotes a fetus that is anteriorly positioned in the right side of the maternal pelvis with the occiput as the presenting part. LSP describes a fetus that is posteriorly positioned in the left side of the pelvis with the sacrum as the presenting part. A fetus that is LOA would be anteriorly positioned in the left side of the pelvis with the occiput as the presenting part.

The nurse would anticipate the treatment for pelvis organ prolapse to be: a. Insertion of the urinary catheter b. A pessary c. Dietary fiber d. Urinary antispasmodics

B. a pessary a common first-line treatment is a pessary, which is a removable mechanical device that holds the uterus in position.

A woman's position is an important component of the labor progress. Which guidance is important for the nurse to provide to the laboring client? a.The supine position, which is commonly used in the United States, increases blood flow. b.The laboring client positioned on her hands and knees ("all fours" position) is hard on the woman's back. c.Frequent changes in position help relieve fatigue and increase the comfort of the laboring client. d.In a sitting or squatting position, abdominal muscles of the laboring client will have to work harder.

ANS: C Frequent position changes relieve fatigue, increase comfort, and improve circulation. Blood flow can be compromised in the supine position; any upright position benefits cardiac output. The "all fours" position is used to relieve backache in certain situations. In a sitting or squatting position, the abdominal muscles work in greater harmony with uterine contractions.

2. The labor of a pregnant woman with preeclampsia is going to be induced. Before initiating the oxytocin (Pitocin) infusion, the nurse reviews the woman's latest laboratory test findings, which reveal a platelet count of 90,000 mm3, an elevated aspartate aminotransaminase (AST) level, and a falling hematocrit. The laboratory results are indicative of which condition? a. Eclampsia b. Disseminated intravascular coagulation (DIC) syndrome c. Hemolysis, elevated liver enzyme levels, and low platelet levels (HELLP) syndrome d. Idiopathic thrombocytopenia

ANS: C HELLP syndrome is a laboratory diagnosis for a variant of severe preeclampsia that involves hepatic dysfunction characterized by hemolysis (H), elevated liver (EL) enzymes, and low platelets (LP). Eclampsia is determined by the presence of seizures. DIC is a potential complication associated with HELLP syndrome. Idiopathic thrombocytopenia is the presence of low platelets of unknown cause and is not associated with preeclampsia. DIF: Cognitive Level: Understand REF: p. 657 TOP: Nursing Process: Diagnosis MSC: Client Needs: Physiologic Integrity

To manage her diabetes appropriately and to ensure a good fetal outcome, how would the pregnant woman with diabetes alter her diet? a.Eat six small equal meals per day. b.Reduce the carbohydrates in her diet. c.Eat her meals and snacks on a fixed schedule. d.Increase her consumption of protein.

ANS: C Having a fixed meal schedule will provide the woman and the fetus with a steady blood sugar level, provide a good balance with insulin administration, and help prevent complications. Having a fixed meal schedule is more important than the equal division of food intake. Approximately 45% of the food eaten should be in the form of carbohydrates.

Which statement regarding the laboratory test for glycosylated hemoglobin Alc is correct? a.The laboratory test for glycosylated hemoglobin Alc is performed for all pregnant women, not only those with or likely to have diabetes. b.This laboratory test is a snapshot of glucose control at the moment. c.This laboratory test measures the levels of hemoglobin Alc, which should remain at less than 7%. d.This laboratory test is performed on the woman's urine, not her blood.

ANS: C Hemoglobin Alc levels greater than 7% indicate an elevated glucose level during the previous 4 to 6 weeks. This extra laboratory test is for diabetic women and defines glycemic control over the previous 4 to 6 weeks. Glycosylated hemoglobin level tests are performed on the blood

Nurses with an understanding of cultural differences regarding likely reactions to pain may be better able to help their clients. Which clients may initially appear very stoic but then become quite vocal as labor progresses until late in labor, when they become more vocal and request pain relief? a.Chinese b.Arab or Middle Eastern c.Hispanic d.African-American

ANS: C Hispanic women may be stoic early in labor but more vocal and ready for medications later. Chinese women may not show reactions to pain. Medical interventions must be offered more than once. Arab or Middle Eastern women may be vocal in response to labor pain from the start; they may prefer pain medications. African-American women may openly express pain; the use of medications for pain is more likely to vary with the individual.

What is the primary rationale for the thorough drying of the infant immediately after birth? a.Stimulates crying and lung expansion b.Removes maternal blood from the skin surface c.Reduces heat loss from evaporation d.Increases blood supply to the hands and feet

ANS: C Infants are wet with amniotic fluid and blood at birth, and this accelerates evaporative heat loss. The primary purpose of drying the infant is to prevent heat loss. Although rubbing the infant stimulates crying, it is not the main reason for drying the infant. This process does not remove all the maternal blood.

12. What is the primary purpose for magnesium sulfate administration for clients with preeclampsia and eclampsia? a. To improve patellar reflexes and increase respiratory efficiency b. To shorten the duration of labor c. To prevent convulsions d. To prevent a boggy uterus and lessen lochial flow

ANS: C Magnesium sulfate is the drug of choice used to prevent convulsions, although it can generate other problems. Loss of patellar reflexes and respiratory depression are signs of magnesium toxicity. Magnesium sulfate can also increase the duration of labor. Women are at risk for a boggy uterus and heavy lochial flow as a result of magnesium sulfate therapy. DIF: Cognitive Level: Understand REF: p. 664 TOP: Nursing Process: Implementation MSC: Client Needs: Physiologic Integrity

3. The nurse is preparing to administer methotrexate to the client. This hazardous drug is most often used for which obstetric complication? a. Complete hydatidiform mole b. Missed abortion c. Unruptured ectopic pregnancy d. Abruptio placentae

ANS: C Methotrexate is an effective nonsurgical treatment option for a hemodynamically stable woman whose ectopic pregnancy is unruptured and measures less than 4 cm in diameter. Methotrexate is not indicated or recommended as a treatment option for a complete hydatidiform mole, for a missed abortion, or for abruptio placentae. DIF: Cognitive Level: Apply REF: p. 677 TOP: Nursing Process: Planning MSC: Client Needs: Physiologic Integrity

The first 1 to 2 hours after birth is sometimes referred to as what? a.Bonding period b.Third stage of labor c.Fourth stage of labor d.Early postpartum period

ANS: C The first 2 hours of the birth are a critical time for the mother and her baby and is often called the fourth stage of labor. Maternal organs undergo their initial readjustment to a nonpregnant state. The third stage of labor lasts from the birth of the baby to the expulsion of the placenta. Bonding will occur over a much longer period, although it may be initiated during the fourth stage of labor.

14. Women with mild gestational hypertension and mild preeclampsia can be safely managed at home with frequent maternal and fetal evaluation. Complete or partial bed rest is still frequently ordered by some providers. Which complication is rarely the result of prolonged bed rest? a. Thrombophlebitis b. Psychologic stress c. Fluid retention d. Cardiovascular deconditioning

ANS: C No evidence has been found that supports the practice of bed rest to improve pregnancy outcome. Fluid retention is not an adverse outcome of prolonged bed rest. The woman is more likely to experience diuresis with accompanying fluid and electrolyte imbalance and weight loss. Prolonged bed rest is known to increase the risk for thrombophlebitis. Psychologic stress is known to begin on the first day of bed rest and continue for the duration of the therapy. Therefore, restricted activity, rather than complete bed rest, is recommended. Cardiovascular deconditioning is a known complication of bed rest. DIF: Cognitive Level: Understand REF: p. 661 TOP: Nursing Process: Diagnosis MSC: Client Needs: Physiologic Integrity

Where is the point of maximal intensity (PMI) of the FHR located? a.Usually directly over the fetal abdomen b.In a vertex position, heard above the mother's umbilicus c.Heard lower and closer to the midline of the mother's abdomen as the fetus descends and internally rotates d.In a breech position, heard below the mother's umbilicus

ANS: C Nurses should be prepared for the shift. The PMI of the FHR is usually directly over the fetal back. In a vertex position, the PMI of the FHR is heard below the mother's umbilicus. In a breech position, it is heard above the mother's umbilicus.

10. The management of the pregnant client who has experienced a pregnancy loss depends on the type of miscarriage and the signs and symptoms. While planning care for a client who desires outpatient management after a first-trimester loss, what would the nurse expect the plan to include? a. Dilation and curettage (D&C) b. Dilation and evacuation (D&E) c. Misoprostol d. Ergot products

ANS: C Outpatient management of a first-trimester loss is safely accomplished by the intravaginal use of misoprostol for up to 2 days. If the bleeding is uncontrollable, vital signs are unstable, or signs of infection are present, then a surgical evacuation should be performed. D&C is a surgical procedure that requires dilation of the cervix and scraping of the uterine walls to remove the contents of pregnancy. This procedure is commonly performed to treat inevitable or incomplete abortion and should be performed in a hospital. D&E is usually performed after 16 weeks of pregnancy. The cervix is widely dilated, followed by removal of the contents of the uterus. Ergot products such as Methergine or Hemabate may be administered for excessive bleeding after miscarriage. DIF: Cognitive Level: Apply REF: p. 672 TOP: Nursing Process: Planning MSC: Client Needs: Physiologic Integrity

A number of metabolic changes occur throughout pregnancy. Which physiologic adaptation of pregnancy will influence the nurse's plan of care? a.Insulin crosses the placenta to the fetus only in the first trimester, after which the fetus secretes its own. b.Women with insulin-dependent diabetes are prone to hyperglycemia during the first trimester because they are consuming more sugar. c.During the second and third trimesters, pregnancy exerts a diabetogenic effect that ensures an abundant supply of glucose for the fetus. d.Maternal insulin requirements steadily decline during pregnancy.

ANS: C Pregnant women develop increased insulin resistance during the second and third trimesters. Insulin never crosses the placenta; the fetus starts making its own around the 10th week. As a result of normal metabolic changes during pregnancy, insulin-dependent women are prone to hypoglycemia (low levels). Maternal insulin requirements may double or quadruple by the end of pregnancy.

Prostaglandin gel has been ordered for a pregnant woman at 43 weeks of gestation. What is the primary purpose of prostaglandin administration? a.To enhance uteroplacental perfusion in an aging placenta b.To increase amniotic fluid volume c.To ripen the cervix in preparation for labor induction d.To stimulate the amniotic membranes to rupture

ANS: C Preparations of prostaglandin E1 and E2 are effective when used before labor induction to ripen (i.e., soften and thin) the cervix. Uteroplacental perfusion is not altered by the use of prostaglandins. The insertion of prostaglandin gel has no effect on the level of amniotic fluid. In some cases, women will spontaneously begin laboring after the administration of prostaglandins, thereby eliminating the need for oxytocin. It is not common for a woman's membranes to rupture as a result of prostaglandin use.

In planning for home care of a woman with preterm labor, which concern should the nurse need to address? a.Nursing assessments are different from those performed in the hospital setting. b.Restricted activity and medications are necessary to prevent a recurrence of preterm labor. c.Prolonged bed rest may cause negative physiologic effects. d.Home health care providers are necessary

ANS: C Prolonged bed rest may cause adverse effects such as weight loss, loss of appetite, muscle wasting, weakness, bone demineralization, decreased cardiac output, risk for thrombophlebitis, alteration in bowel functions, sleep disturbance, and prolonged postpartum recovery. Nursing assessments differ somewhat from those performed in the acute care setting, but this concern does not need to be addressed. Restricted activity and medications may prevent preterm labor but not in all women. In addition, the plan of care is individualized to meet the needs of each client. Many women receive home health nurse visits, but care is individualized for each woman

1. A primigravida is being monitored at the prenatal clinic for preeclampsia. Which finding is of greatest concern to the nurse? a. Blood pressure (BP) increase to 138/86 mm Hg b. Weight gain of 0.5 kg during the past 2 weeks c. Dipstick value of 3+ for protein in her urine d. Pitting pedal edema at the end of the day

ANS: C Proteinuria is defined as a concentration of 1+ or greater via dipstick measurement. A dipstick value of 3+ alerts the nurse that additional testing or assessment should be performed. A 24-hour urine collection is preferred over dipstick testing attributable to accuracy. Generally, hypertension is defined as a BP of 140/90 mm Hg or an increase in systolic pressure of 30 mm Hg or diastolic pressure of 15 mm Hg. Preeclampsia may be demonstrated as a rapid weight gain of more than 2 kg in 1 week. Edema occurs in many normal pregnancies, as well as in women with preeclampsia. Therefore, the presence of edema is no longer considered diagnostic of preeclampsia. DIF: Cognitive Level: Analyze REF: p. 660 TOP: Nursing Process: Diagnosis MSC: Client Needs: Physiologic Integrity

A 30-year-old female with newly diagnosed polycystic syndrome (PCOS) is being counseled by her OB-GYN. The physician indicates the this conditions often results in: a. ovarian cancer b. infertility c. early menopause d. pelvic inflammatory disease

B. infertility Because FSH levels are not totally depressed, new follicular growth is continuously stimulated, but not to full maturation and ovulation.

4. A pregnant woman has been receiving a magnesium sulfate infusion for treatment of severe preeclampsia for 24 hours. On assessment, the nurse finds the following vital signs: temperature 37.3° C, pulse rate 88 beats per minute, respiratory rate 10 breaths per minute, BP 148/90 mm Hg, absent deep tendon reflexes (DTRs), and no ankle clonus. The client complains, "I'm so thirsty and warm." What is the nurse's immediate action? a. To call for an immediate magnesium sulfate level b. To administer oxygen c. To discontinue the magnesium sulfate infusion d. To prepare to administer hydralazine

ANS: C Regardless of the magnesium level, the client is displaying the clinical signs and symptoms of magnesium toxicity. The first action by the nurse should be to discontinue the infusion of magnesium sulfate. In addition, calcium gluconate, the antidote for magnesium, may be administered. Hydralazine is an antihypertensive drug commonly used to treat hypertension in severe preeclampsia. Typically, hydralazine is administered for a systolic BP higher than 160 mm Hg or a diastolic BP higher than 110 mm Hg. DIF: Cognitive Level: Apply REF: p. 664 TOP: Nursing Process: Implementation MSC: Client Needs: Physiologic Integrity

Which statement by the client would lead the nurse to believe that labor has been established? a."I passed some thick, pink mucus when I urinated this morning." b."My bag of waters just broke." c."The contractions in my uterus are getting stronger and closer together." d."My baby dropped, and I have to urinate more frequently now."

ANS: C Regular, strong contractions with the presence of cervical change indicate that the woman is experiencing true labor. Although the loss of the mucous plug (operculum) often occurs during the first stage of labor or before the onset of labor, it is not the indicator of true labor. Spontaneous rupture of membranes often occurs during the first stage of labor; however, it is not an indicator of true labor. The presenting part of the fetus typically becomes engaged in the pelvis at the onset of labor but is not the indicator of true labor.

Which statement by the client will assist the nurse in determining whether she is in true labor as opposed to false labor? a."I passed some thick, pink mucus when I urinated this morning." b."My bag of waters just broke." c."The contractions in my uterus are getting stronger and closer together." d."My baby dropped, and I have to urinate more frequently now."

ANS: C Regular, strong contractions with the presence of cervical change indicate that the woman is experiencing true labor. The loss of the mucous plug (operculum) often occurs during the first stage of labor or before the onset of labor, but it is not the indicator of true labor. Spontaneous rupture of membranes (ROM) often occurs during the first stage of labor, but it is not the indicator of true labor. The presenting part of the fetus typically becomes engaged in the pelvis at the onset of labor, but this is not the indicator of true labor.

The nurse is teaching a client with preterm premature rupture of membranes (PPROM) regarding self-care activities. Which activities should the nurse include in her teaching? a.Report a temperature higher than 40° C. b.Tampons are safe to use to absorb the leaking amniotic fluid. c.Do not engage in sexual activity. d.Taking frequent tub baths is safe.

ANS: C Sexual activity should be avoided because it may induce preterm labor. A temperature higher than 38° C should be reported. To prevent the risk of infection, tub baths should be avoided and nothing should be inserted into the vagina. Further, foul-smelling vaginal fluid, which may be a sign of infection, should be reported.

8. A woman who is 30 weeks of gestation arrives at the hospital with bleeding. Which differential diagnosis would not be applicable for this client? a. Placenta previa b. Abruptio placentae c. Spontaneous abortion d. Cord insertion

ANS: C Spontaneous abortion is another name for miscarriage; it occurs, by definition, early in pregnancy. Placenta previa is a well-known reason for bleeding late in pregnancy. The premature separation of the placenta (abruptio placentae) is a bleeding disorder that can occur late in pregnancy. Cord insertion may cause a bleeding disorder that can also occur late in pregnancy. DIF: Cognitive Level: Understand REF: p. 669 TOP: Nursing Process: Assessment MSC: Client Needs: Physiologic Integrity, Physiologic Adaptation

20. Which statement most accurately describes the HELLP syndrome? a. Mild form of preeclampsia b. Diagnosed by a nurse alert to its symptoms c. Characterized by hemolysis, elevated liver enzymes, and low platelets d. Associated with preterm labor but not perinatal mortality

ANS: C The acronym HELLP stands for hemolysis (H), elevated liver (EL) enzymes, and low platelets (LP). The HELLP syndrome is a variant of severe preeclampsia and is difficult to identify because the symptoms are not often obvious. The HELLP syndrome must be diagnosed in the laboratory. Preterm labor is greatly increased; therefore, so is perinatal mortality. DIF: Cognitive Level: Understand REF: p. 657 TOP: Nursing Process: Diagnosis | Nursing Process: Planning MSC: Client Needs: Physiologic Integrity

A primigravida at 40 weeks of gestation is having uterine contractions every to 2 minutes and states that they are very painful. Her cervix is dilated 2 cm and has not changed in 3 hours. The woman is crying and wants an epidural. What is the likely status of this woman's labor? a.She is exhibiting hypotonic uterine dysfunction. b.She is experiencing a normal latent stage. c.She is exhibiting hypertonic uterine dysfunction. d.She is experiencing precipitous labor.

ANS: C The contraction pattern observed in this woman signifies hypertonic uterine activity. Typically, uterine activity in this phase occurs at 4- to 5-minute intervals lasting 30 to 45 seconds. Women who experience hypertonic uterine dysfunction, or primary dysfunctional labor, are often anxious first-time mothers who are having painful and frequent contractions that are ineffective at causing cervical dilation or effacement to progress. With hypotonic uterine dysfunction, the woman initially makes normal progress into the active stage of labor; then the contractions become weak and inefficient or stop altogether. Precipitous labor is one that lasts less than 3 hours from the onset of contractions until time of birth.

Which description of the phases of the second stage of labor is most accurate? a.Latent phase: Feeling sleepy; fetal station 2+ to 4+; duration of 30 to 45 minutes b.Active phase: Overwhelmingly strong contractions; Ferguson reflux activated; duration of 5 to 15 minutes c.Descent phase: Significant increase in contractions; Ferguson reflux activated; average duration varies d.Transitional phase: Woman "laboring down"; fetal station 0; duration of 15 minutes

ANS: C The descent phase begins with a significant increase in contractions; the Ferguson reflex is activated, and the duration varies, depending on a number of factors. The latent phase is the lull or "laboring down" period at the beginning of the second stage and lasts 10 to 30 minutes on average. The second stage of labor has no active phase. The transition phase is the final phase in the second stage of labor; contractions are strong and painful.

A woman in labor has just received an epidural block. What is the most important nursing intervention at this time? a.Limit parenteral fluids. b.Monitor the fetus for possible tachycardia. c.Monitor the maternal blood pressure for possible hypotension. d.Monitor the maternal pulse for possible bradycardia.

ANS: C The most important nursing intervention for a woman who has received an epidural block is for the nurse to monitor the maternal blood pressure frequently for signs of hypotension. IV fluids are increased for a woman receiving an epidural to prevent hypotension. The nurse also observes for signs of fetal bradycardia and monitors for signs of maternal tachycardia, secondary to hypotension.

A labor and delivery nurse should be cognizant of which information regarding how the fetus moves through the birth canal? a.Fetal attitude describes the angle at which the fetus exits the uterus. b.Of the two primary fetal lies, the horizontal lie is that in which the long axis of the fetus is parallel to the long axis of the mother. c.Normal attitude of the fetus is called general flexion. d.Transverse lie is preferred for vaginal birth.

ANS: C The normal attitude of the fetus is called general flexion. The fetal attitude is the relationship of the fetal body parts to each one another. The horizontal lie is perpendicular to the mother; in the longitudinal (or vertical) lie, the long axes of the fetus and the mother are parallel. Vaginal birth cannot occur if the fetus stays in a transverse lie.

The uterine contractions of a woman early in the active phase of labor are assessed by an internal uterine pressure catheter (IUPC). The uterine contractions occur every 3 to 4 minutes and last an average of 55 to 60 seconds. They are becoming more regular and are moderate to strong. Based on this information, what would a prudent nurse do next? a.Immediately notify the woman's primary health care provider. b.Prepare to administer an oxytocic to stimulate uterine activity. c.Document the findings because they reflect the expected contraction pattern for the active phase of labor. d.Prepare the woman for the onset of the second stage of labor.

ANS: C The nurse is responsible for monitoring the uterine contractions to ascertain whether they are powerful and frequent enough to accomplish the work of expelling the fetus and the placenta. In addition, the nurse documents these findings in the client's medical record. This labor pattern indicates that the client is in the active phase of the first stage of labor. Nothing indicates a need to notify the primary health care provider at this time. Oxytocin augmentation is not needed for this labor pattern; this contraction pattern indicates that the woman is in active labor. Her contractions will eventually become stronger, last longer, and come closer together during the transition phase of the first stage of labor. The transition phase precedes the second stage of labor, or delivery of the fetus.

Which action is correct when palpation is used to assess the characteristics and pattern of uterine contractions? a.Placing the hand on the abdomen below the umbilicus and palpating uterine tone with the fingertips b.Determining the frequency by timing from the end of one contraction to the end of the next contraction c.Evaluating the intensity by pressing the fingertips into the uterine fundus d.Assessing uterine contractions every 30 minutes throughout the first stage of labor

ANS: C The nurse or primary health care provider may assess uterine activity by palpating the fundal section of the uterus using the fingertips. Many women may experience labor pain in the lower segment of the uterus, which may be unrelated to the firmness of the contraction detectable in the uterine fundus. The frequency of uterine contractions is determined by palpating from the beginning of one contraction to the beginning of the next contraction. Assessment of uterine activity is performed in intervals based on the stage of labor. As labor progresses, this assessment is performed more frequently.

A woman has requested an epidural for her pain. She is 5 cm dilated and 100% effaced. The baby is in a vertex position and is engaged. The nurse increases the woman's IV fluid for a preprocedural bolus. The nurse reviews her laboratory values and notes that the woman's hemoglobin is 12 g/dl, hematocrit is 38%, platelets are 67,000, and white blood cells (WBCs) are 12,000/mm3. Which factor would contraindicate an epidural for this woman? a.She is too far dilated. b.She is anemic. c.She has thrombocytopenia. d.She is septic.

ANS: C The platelet count indicates a coagulopathy, specifically, thrombocytopenia (low platelets), which is a contraindication to epidural analgesia and anesthesia. Typically, epidural analgesia and anesthesia are used in the laboring woman when a regular labor pattern has been achieved, as evidenced by progressive cervical change. The laboratory values show that the woman's hemoglobin and hematocrit levels are in the normal range and show a slight increase in the WBC count that is not uncommon in laboring women.

Nurses should be cognizant of what regarding the mechanism of labor? a.Seven critical movements must progress in a more or less orderly sequence. b.Asynclitism is sometimes achieved by means of the Leopold's maneuver. c.Effects of the forces determining descent are modified by the shape of the woman's pelvis and the size of the fetal head. d.At birth, the baby is said to achieve "restitution"; that is, a return to the C-shape of the womb.

ANS: C The size of the maternal pelvis and the ability of the fetal head to mold also affect the process. The seven identifiable movements of the mechanism of labor simultaneously occur in combinations, not in precise sequences. Asynclitism is the deflection of the baby's head; the Leopold's maneuver is a means of judging descent by palpating the mother's abdomen. Restitution is the rotation of the baby's head after the infant is born.

23. Which finding on a prenatal visit at 10 weeks of gestation might suggest a hydatidiform mole? a. Complaint of frequent mild nausea b. Blood pressure of 120/80 mm Hg c. Fundal height measurement of 18 cm d. History of bright red spotting for 1 day, weeks ago

ANS: C The uterus in a hydatidiform molar pregnancy is often larger than would be expected on the basis of the duration of the pregnancy. Nausea increases in a molar pregnancy because of the increased production of hCG. A woman with a molar pregnancy may have early-onset pregnancy-induced hypertension. In the client's history, bleeding is normally described as brownish. DIF: Cognitive Level: Analyze REF: p. 678 TOP: Nursing Process: Assessment MSC: Client Needs: Health Promotion and Maintenance

16. The nurse has evaluated a client with preeclampsia by assessing DTRs. The result is a grade of 3+. Which DTR response most accurately describes this score? a. Sluggish or diminished b. Brisk, hyperactive, with intermittent or transient clonus c. Active or expected response d. More brisk than expected, slightly hyperactive

ANS: D DTRs reflect the balance between the cerebral cortex and the spinal cord. They are evaluated at baseline and to detect changes. A slightly hyperactive and brisk response indicates a grade 3+ response. DIF: Cognitive Level: Apply REF: p. 660 TOP: Nursing Process: Assessment MSC: Client Needs: Physiologic Integrity

A nulliparous woman has just begun the latent phase of the second stage of her labor. The nurse should anticipate which behavior? a.A nulliparous woman will experience a strong urge to bear down. b.Perineal bulging will show. c.A nulliparous woman will remain quiet with her eyes closed between contractions. d.The amount of bright red bloody show will increase

ANS: C The woman is able to relax and close her eyes between contractions as the fetus passively descends. The woman may be very quiet during this phase. During the latent phase of the second stage of labor, the urge to bear down is often absent or only slight during the acme of the contractions. Perineal bulging occurs during the transition phase of the second stage of labor, not at the beginning of the second stage. An increase in bright red bloody show occurs during the descent phase of the second stage of labor.

A 25-year-old gravida 3, para 2 client gave birth to a 9-pound, 7-ounce boy, 4 hours ago after augmentation of labor with oxytocin (Pitocin). She presses her call light, and asks for her nurse right away, stating "I'm bleeding a lot." What is the most likely cause of postpartum hemorrhaging in this client? a.Retained placental fragments b.Unrepaired vaginal lacerations c.Uterine atony d.Puerperal infection

ANS: C This woman gave birth to a macrosomic infant after oxytocin augmentation. Combined with these risk factors, uterine atony is the most likely cause of bleeding 4 hours after delivery. Although retained placental fragments may cause postpartum hemorrhaging, it is typically detected within the first hour after delivery of the placenta and is not the most likely cause of the hemorrhaging in this woman. Although unrepaired vaginal lacerations may also cause bleeding, it typically occurs in the period immediately after birth. Puerperal infection can cause subinvolution and subsequent bleeding that is, however, typically detected 24 hours postpartum.

13. The American College of Obstetricians and Gynecologists (ACOG) has developed a comprehensive list of risk factors associated with the development of preeclampsia. Which client exhibits the greatest number of these risk factors? a. 30-year-old obese Caucasian with her third pregnancy b. 41-year-old Caucasian primigravida c. 19-year-old African American who is pregnant with twins d. 25-year-old Asian American whose pregnancy is the result of donor insemination

ANS: C Three risk factors are present in the 19-year-old African-American client. She has African-American ethnicity, is at the young end of the age distribution, and has a multiple pregnancy. In planning care for this client, the nurse must frequently monitor her BP and teach her to recognize the early warning signs of preeclampsia. The 30-year-old obese Caucasian client has only has one known risk factor: obesity. Age distribution appears to be U-shaped, with women younger than 20 years of age and women older than 40 years of age being at greatest risk. Preeclampsia continues to be more frequently observed in primigravidas; this client is a multigravida woman. Two risk factors are present for the 41-year-old Caucasian primigravida client. Her age and status as a primigravida place her at increased risk for preeclampsia. Caucasian women are at a lower risk than are African-American women. The 25-year-old Asian-American client exhibits only one risk factor. Pregnancies that result from donor insemination, oocyte donation, and embryo donation are at an increased risk of developing preeclampsia. DIF: Cognitive Level: Analyze REF: p. 655 TOP: Nursing Process: Planning MSC: Client Needs: Physiologic Integrity

Which statement related to cephalopelvic disproportion (CPD) is the least accurate? a.CPD can be related to either fetal size or fetal position. b.The fetus cannot be born vaginally. c.CPD can be accurately predicted. d.Causes of CPD may have maternal or fetal origins.

ANS: C Unfortunately, accurately predicting CPD is not possible. Although CPD is often related to excessive fetal size (macrosomia), malposition of the fetal presenting part is the problem in many cases, not true CPD. When CPD is present, the fetus cannot fit through the maternal pelvis to be born vaginally. CPD may be related to either fetal origins such as macrosomia or malposition or maternal origins such as a too small or malformed pelvis.

Which adaptation of the maternal-fetal exchange of oxygen occurs in response to uterine contraction? a.The maternal-fetal exchange of oxygen and waste products continues except when placental functions are reduced. b.This maternal-fetal exchange increases as the blood pressure decreases. c.It diminishes as the spiral arteries are compressed. d.This exchange of oxygen and waste products is not significantly affected by contractions.

ANS: C Uterine contractions during labor tend to decrease circulation through the spiral electrodes and subsequent perfusion through the intervillous space. The maternal blood supply to the placenta gradually stops with contractions. The exchange of oxygen and waste products decreases. The exchange of oxygen and waste products is affected by contractions.

A woman is having her first child. She has been in labor for 15 hours. A vaginal examination performed 2 hours earlier revealed the cervix to be dilated to 5 cm and 100% effaced, and the presenting part of the fetus was at station 0; however, another vaginal examination performed 5 minutes ago indicated no changes. What abnormal labor pattern is associated with this description? a.Prolonged latent phase b.Protracted active phase c.Secondary arrest d.Protracted descent

ANS: C With a secondary arrest of the active phase, the progress of labor has stopped. This client has not had any anticipated cervical change, indicating an arrest of labor. In the nulliparous woman, a prolonged latent phase typically lasts longer than 20 hours. A protracted active phase, the first or second stage of labor, is prolonged (slow dilation). With a protracted descent, the fetus fails to descend at an anticipated rate during the deceleration phase and second stage of labor.

19. The nurse is palpating the fundus of a pregnant woman. Which of these statements about palpation of the fundus is true? A. It should be hard and slightly tender to palpation during the first trimester. B. Fetal movement may not be felt by the examiner until the end of the second trimester. C. After 20 weeks of gestation, the number of centimeters should approximate the number of weeks of gestation. D. Fundal height is usually less than the number of weeks of gestation, unless there isan abnormal condition such as too much amniotic fluid present

ANS: C After 20 weeks, the number of centimeters should approximate the number of weeks of gestation. Also, at 20 weeks of gestation the examiner may feel fetal movement and the head can be balloted

The decent of the bladder and the anterior vaginal wall into the vaginal canal is called a; a. rectocele b. varicocele c. cystocele d. urethrocele

C. cystocele A cystocele is the decent of the bladder into the vaginal canal.

10. The nurse auscultates a functional systolic murmur, grade II/IV, on a woman in week 30 of her pregnancy. The remainder of her physical assessment is within normal limits. The nurse would: A. Consider this an abnormal finding and refer her for additional consultation. B. Ask the woman to run in place briefly and then assess for an increase in intensity of the murmur. C. Know that this is a normal finding resulting from the increase in blood volume during pregnancy. D. Ask the woman to restrict her activities and return to the clinic in 1 week for re-evaluation.

ANS: C Because of the increase in blood volume, a functional systolic murmur, grade II/IV or less, can be heard in 95% of pregnant women. The other actions are not appropriate

3. When performing the examination of a woman who is 8 weeks pregnant, the nurse notices that the cervix is a bluish color. The nurse would document this finding as _____ sign. A. Hegar's B. Homans' C. Chadwick's D. Goodell's

ANS: C During pregnancy, the uterus becomes globular in shape, softens, and flexes over the cervix (Hegar's sign). The cervix softens (Goodell's sign) and becomes bluish or cyanotic in color (Chadwick's sign). Homans' sign is pain in the calf upon dorsiflexion of the foot and is not related to signs of pregnancy

7. During the examination of a woman in her second trimester of pregnancy, the nurse notices the presence of a small amount of yellow drainage from the nipples. The nurse knows that this is: A. An indication that the woman's milk is coming in. B. A sign of possible breast cancer in a pregnant woman. C. Most likely colostrum and considered a normal finding at this stage of the pregnancy. D. Too early in the pregnancy for lactation to begin, and refers the woman to a specialist.

ANS: C During the second trimester, colostrum, the precursor of milk, may be expressed from the nipples. Colostrum is yellow and contains more minerals and protein but less sugar and fat than mature milk. The other options are incorrect.

4. A woman who is 8 weeks pregnant is visiting the clinic for a checkup. Her systolic blood pressure is 30 mm Hg higher than her prepregnancy systolic blood pressure. The nurse should: A. Consider this a normal finding. B. Expect the blood pressure to decrease as the estrogen levels increase throughout the pregnancy. C. Consider this an abnormal finding because blood pressure is typically lower at this point in the pregnancy. D. Recommend that she decrease her salt intake in an attempt to decrease her peripheral vascular resistance

ANS: C During the seventh gestational week, blood pressure begins to drop as a result of falling peripheral vascular resistance. Early in the first trimester, blood pressure values are similar to those of pre pregnancy measurements. In this case, the woman's blood pressure is higher than it should be.

9. A patient who is 20 weeks pregnant tells the nurse that she feels more shortness of breath as her pregnancy progresses. The nurse recognizes that which of these statements is true? A. High levels of estrogen cause shortness of breath. B. Feelings of shortness of breath are abnormal during pregnancy. C. The hormones of pregnancy cause an increased respiratory effort. D. She should get more exercise in an attempt to increase her respiratory reserve.

ANS: C Progesterone and estrogen cause an increased respiratory effort during pregnancy by increasing tidal volume. Increased tidal volume causes a slight drop in partial pressure of arterial carbon dioxide (PaCO2), causing the woman to occasionally have dyspnea. The other options are not correct.

6. A patient who is in her first trimester of pregnancy tells the nurse that she is experiencing significant nausea and vomiting and asks when it will improve. The nurse should reply: A. "Did your mother have significant nausea and vomiting?" B. "Many women experience nausea and vomiting until the third trimester." C. "Usually, by the beginning of the second trimester, the nausea and vomiting improve." D. "At about the time you begin to feel the baby move, the nausea and vomiting will subside."

ANS: C The nausea, vomiting, and fatigue of pregnancy improve by weeks 12 to 16. Quickening, when the mother recognizes fetal movement, occurs at approximately 18 to 20 weeks.

25. A patient's pregnancy test is positive, and she wants to know when the baby is due. The first day of her last menstrual period was June 14, and that period ended June 20. Using Nägele's rule, what is her expected date of delivery? A. March 7 B. March 14 C. March 21 D. March 27

ANS: C Using Nägele's rule, add 7 days to the first day of the last menstrual period and then subtract 3 months. Therefore, adding 7 days to June 14 would be June 21, then subtracting 3 months makes it March 21

34. During a group prenatal teaching session, the nurse teaches Kegel exercises. Which of these statements would be appropriate for this teaching session? Select all that apply. A. "Kegel exercises help to keep your uterus strong during the pregnancy." B. "Kegel exercises should be performed twice a day." C. "Kegel exercises should be performed 50 to 100 times a day." D. "To perform Kegel exercises, squeeze slowly to a peak at the count of eight, and then release slowly to a count of eight." E. "To perform Kegel exercises, perform rapidly alternating squeeze-release exercises up to the count of eight."

ANS: C, D Kegel exercises can be done to prepare for and to recover from birth. The nurse should direct the woman to squeeze slowly to a peak at the count of eight and then to release slowly to the count of eight. The nurse can prescribe this exercise to be performed 50 to 100 times a day.

Hypothyroidism occurs in 2 to 3 pregnancies per 1000. Because severe hypothyroidism is associated with infertility and miscarriage, it is not often seen in pregnancy. Regardless of this fact, the nurse should be aware of the characteristic symptoms of hypothyroidism. Which do they include? (Select all that apply.) a.Hot flashes b.Weight loss c.Lethargy d.Decrease in exercise capacity e.Cold intolerance

ANS: C, D, E Symptoms include weight gain, lethargy, decrease in exercise capacity, and intolerance to cold. Other presentations might include constipation, hoarseness, hair loss, and dry skin. Thyroid supplements are used to treat hyperthyroidism in pregnancy.

Which physiologic alteration of pregnancy most significantly affects glucose metabolism? a.Pancreatic function in the islets of Langerhans is affected by pregnancy. b.Pregnant women use glucose at a more rapid rate than nonpregnant women. c.Pregnant women significantly increase their dietary intake. d.Placental hormones are antagonistic to insulin, thus resulting in insulin resistance.

ANS: D Placental hormones, estrogen, progesterone, and human placental lactogen (HPL) create insulin resistance. Insulin is also broken down more quickly by the enzyme placental insulinase. Pancreatic functioning is not affected by pregnancy. The glucose requirements differ because of the growing fetus. The pregnant woman should increase her intake by 200 calories a day.

A woman with gestational diabetes has had little or no experience reading and interpreting glucose levels. The client shows the nurse her readings for the past few days. Which reading signals the nurse that the client may require an adjustment of insulin or carbohydrates? a.75 mg/dl before lunch. This is low; better eat now. b.115 mg/dl 1 hour after lunch. This is a little high; maybe eat a little less next time. c.115 mg/dl 2 hours after lunch. This is too high; it is time for insulin. d.50 mg/dl just after waking up from a nap. This is too low; maybe eat a snack before going to sleep.

ANS: D 50 mg/dl after waking from a nap is too low. During hours of sleep, glucose levels should not be less than 60 mg/dl. Snacks before sleeping can be helpful. The premeal acceptable range is 60 to 99 mg/dl. The readings 1 hour after a meal should be less than 129 mg/dl. Two hours after eating, the readings should be less than 120 mg/dl.

Which basic type of pelvis includes the correct description and percentage of occurrence in women? a.Gynecoid: classic female pelvis; heart shaped; 75% b.Android: resembling the male pelvis; wide oval; 15% c.Anthropoid: resembling the pelvis of the ape; narrow; 10% d.Platypelloid: flattened, wide, and shallow pelvis; 3%

ANS: D A platypelloid pelvis is flattened, wide, and shallow; approximately 3% of women have this shape. The gynecoid pelvis is the classic female shape, slightly ovoid and rounded; approximately 50% of women have this shape. An android or malelike pelvis is heart shaped; approximately 23% of women have this shape. An anthropoid or apelike pelvis is oval and wide; approximately 24% of women have this shape.

The obstetric nurse is preparing the client for an emergency cesarean birth, with no time to administer spinal anesthesia. The nurse is aware of and prepared for the greatest risk of administering general anesthesia to the client. What is this risk? a.Respiratory depression b.Uterine relaxation c.Inadequate muscle relaxation d.Aspiration of stomach contents

ANS: D Aspiration of acidic gastric contents with possible airway obstruction is a potentially fatal complication of general anesthesia. Respirations can be altered during general anesthesia, and the anesthesiologist will take precautions to maintain proper oxygenation. Uterine relaxation can occur with some anesthesia but can be monitored and prevented. Inadequate muscle relaxation can be improved with medication.

When a nulliparous woman telephones the hospital to report that she is in labor, what guidance should the nurse provide or information should the nurse obtain? a.Tell the woman to stay home until her membranes rupture. b.Emphasize that food and fluid intake should stop. c.Arrange for the woman to come to the hospital for labor evaluation. d.Ask the woman to describe why she believes she is in labor.

ANS: D Assessment begins at the first contact with the woman, whether by telephone or in person. By asking the woman to describe her signs and symptoms, the nurse can begin her assessment and gather data. The initial nursing activity should be to gather data about the woman's status. The amniotic membranes may or may not spontaneously rupture during labor. The client may be instructed to stay home until the uterine contractions become strong and regular. Before instructing the woman to come to the hospital, the nurse should initiate her assessment during the telephone interview. After this assessment has been made, the nurse may want to discuss the appropriate oral intake for early labor, such as light foods or clear liquids, depending on the preference of the client or her primary health care provider.

7. The nurse who elects to practice in the area of women's health must have a thorough understanding of miscarriage. Which statement regarding this condition is most accurate? a. A miscarriage is a natural pregnancy loss before labor begins. b. It occurs in fewer than 5% of all clinically recognized pregnancies. c. Careless maternal behavior, such as poor nutrition or excessive exercise, can be a factor in causing a miscarriage. d. If a miscarriage occurs before the 12th week of pregnancy, then it may be observed only as moderate discomfort and blood loss.

ANS: D Before the sixth week, the only evidence might be a heavy menstrual flow. After the 12th week, more severe pain, similar to that of labor, is likely. Miscarriage is a natural pregnancy loss, but it occurs, by definition, before 20 weeks of gestation, before the fetus is viable. Miscarriages occur in approximately 10% to 15% of all clinically recognized pregnancies. Miscarriages can be caused by a number of disorders or illnesses outside the mother's control or knowledge. DIF: Cognitive Level: Understand REF: p. 670 TOP: Nursing Process: Assessment MSC: Client Needs: Physiologic Integrity

A woman who is pregnant for the first time is dilated 3 cm and having contractions every 5 minutes. She is groaning and perspiring excessively; she states that she did not attend childbirth classes. What is the optimal intervention for the nurse to provide at this time? a.Notify the woman's health care provider. b.Administer the prescribed narcotic analgesic. c.Assure her that her labor will be over soon. d.Assist her with simple breathing and relaxation instructions.

ANS: D By reducing tension and stress, both focusing and relaxation techniques will allow the woman in labor to rest and conserve energy for the task of giving birth. For those who have had no preparation, instruction in simple breathing and relaxation can be given in early labor and is often successful. The nurse can independently perform many functions in labor and birth, such as teaching and support. Pain medication may be an option for this client. However, the initial response of the nurse should include teaching the client about her options. The length of labor varies among individuals, but the first stage of labor is the longest. At 3 cm of dilation with contractions every 5 minutes, this woman has a significant amount of labor yet to experience.

A woman at 26 weeks of gestation is being assessed to determine whether she is experiencing preterm labor. Which finding indicates that preterm labor is occurring? a.Estriol is not found in maternal saliva. b.Irregular, mild uterine contractions are occurring every 12 to 15 minutes. c.Fetal fibronectin is present in vaginal secretions. d.The cervix is effacing and dilated to 2 cm.

ANS: D Cervical changes such as shortened endocervical length, effacement, and dilation are predictors of imminent preterm labor. Changes in the cervix accompanied by regular contractions indicate labor at any gestation. Estriol is a form of estrogen produced by the fetus that is present in plasma at 9 weeks of gestation. Levels of salivary estriol have been shown to increase before preterm birth. Irregular, mild contractions that do not cause cervical change are not considered a threat. The presence of fetal fibronectin in vaginal secretions between 24 and 36 weeks of gestation could predict preterm labor, but it has only a 20% to 40% positive predictive value. Of more importance are other physiologic clues of preterm labor such as cervical changes.

An 18-year-old pregnant woman, gravida 1, para 0, is admitted to the labor and birth unit with moderate contractions every 5 minutes that last 40 seconds. The client states, "My contractions are so strong, I don't know what to do." Before making a plan of care, what should the nurse's first action be? a.Assess for fetal well-being. b.Encourage the woman to lie on her side. c.Disturb the woman as little as possible. d.Recognize that pain is personalized for each individual.

ANS: D Each woman's pain during childbirth is unique and is influenced by a variety of physiologic, psychosocial, and environmental factors. A critical issue for the nurse is how support can make a difference in the pain of the woman during labor and birth. This scenario includes no information that would indicate fetal distress or a logical reason to be overly concerned about the well-being of the fetus. The left lateral position is used to alleviate fetal distress, not maternal stress. The nurse has an obligation to provide physical, emotional, and psychosocial care and support to the laboring woman. This client clearly needs support.

Certain changes stimulate chemoreceptors in the aorta and carotid bodies to prepare the fetus for initiating respirations immediately after birth. Which change in fetal physiologic activity is not part of this process? a.Fetal lung fluid is cleared from the air passages during labor and vaginal birth. b.Fetal partial pressure of oxygen (PO2) decreases. c.Fetal partial pressure of carbon dioxide in arterial blood (PaCO2) increases. d.Fetal respiratory movements increase during labor.

ANS: D Fetal respiratory movements actually decrease during labor. Fetal lung fluid is cleared from the air passages during labor and vaginal birth. Fetal PO2 decreases, and fetal PaCO2 increases.

The nurse is performing an assessment on a client who thinks she may be experiencing preterm labor. Which information is the most important for the nurse to understand and share with the client? a.Because all women must be considered at risk for preterm labor and prediction is so variable, teaching pregnant women the symptoms of preterm labor probably causes more harm through false alarms. b.Braxton Hicks contractions often signal the onset of preterm labor. c.Because preterm labor is likely to be the start of an extended labor, a woman with symptoms can wait several hours before contacting the primary caregiver. d.Diagnosis of preterm labor is based on gestational age, uterine activity, and progressive cervical change.

ANS: D Gestational age of 20 to 37 weeks, uterine contractions, and a cervix that is 80% effaced or dilated 2 cm indicates preterm labor. It is essential that nurses teach women how to detect the early symptoms of preterm labor. Braxton Hicks contractions resemble preterm labor contractions, but they are not true labor. Waiting too long to see a health care provider could result in essential medications failing to be administered. Preterm labor is not necessarily long-term labor.

7. The client has been on magnesium sulfate for 20 hours for the treatment of preeclampsia. She just delivered a viable infant girl 30 minutes ago. What uterine findings does the nurse expect to observe or assess in this client? a. Absence of uterine bleeding in the postpartum period b. Fundus firm below the level of the umbilicus c. Scant lochia flow d. Boggy uterus with heavy lochia flow

ANS: D High serum levels of magnesium can cause a relaxation of smooth muscle such as the uterus. Because of this tocolytic effect, the client will most likely have a boggy uterus with increased amounts of bleeding. All women experience uterine bleeding in the postpartum period, especially those who have received magnesium therapy. Rather than scant lochial flow, however, this client will most likely have a heavy flow attributable to the relaxation of the uterine wall caused by magnesium administration. DIF: Cognitive Level: Analyze REF: p. 664 TOP: Nursing Process: Assessment MSC: Client Needs: Physiologic Integrity

Which statement correctly describes the effects of various pain factors? a.Higher prostaglandin levels arising from dysmenorrhea can blunt the pain of childbirth. b.Upright positions in labor increase the pain factor because they cause greater fatigue. c.Women who move around trying different positions experience more pain. d.Levels of pain-mitigating beta-endorphins are higher during a spontaneous, natural childbirth.

ANS: D Higher endorphin levels help women tolerate pain and reduce anxiety and irritability. Higher prostaglandin levels correspond to more severe labor pains. Upright positions in labor usually result in improved comfort and less pain. Moving freely to find more comfortable positions is important for reducing pain and muscle tension.

3. A woman with preeclampsia has a seizure. What is the nurse's highest priority during a seizure? a. To insert an oral airway b. To suction the mouth to prevent aspiration c. To administer oxygen by mask d. To stay with the client and call for help

ANS: D If a client becomes eclamptic, then the nurse should stay with the client and call for help. Nursing actions during a convulsion are directed toward ensuring a patent airway and client safety. Insertion of an oral airway during seizure activity is no longer the standard of care. The nurse should attempt to keep the airway patent by turning the client's head to the side to prevent aspiration. Once the seizure has ended, it may be necessary to suction the client's mouth. Oxygen is administered after the convulsion has ended. DIF: Cognitive Level: Apply REF: p. 666 TOP: Nursing Process: Implementation MSC: Client Needs: Physiologic Integrity

Which statement related to the induction of labor is most accurate? a.Can be achieved by external and internal version techniques b.Is also known as a trial of labor (TOL) c.Is almost always performed for medical reasons d.Is rated for viability by a Bishop score

ANS: D Induction of labor is likely to be more successful with a Bishop score of 9 or higher for first-time mothers or 5 or higher for veterans. Version is the turning of the fetus to a better position by a physician for an easier or safer birth. A TOL is the observance of a woman and her fetus for several hours of active labor to assess the safety of vaginal birth. Two thirds of cases of induced labor are elective and not done for medical reasons

When would an internal version be indicated to manipulate the fetus into a vertex position? a.Fetus from a breech to a cephalic presentation before labor begins b.Fetus from a transverse lie to a longitudinal lie before a cesarean birth c.Second twin from an oblique lie to a transverse lie before labor begins d.Second twin from a transverse lie to a breech presentation during a vaginal birth

ANS: D Internal version is used only during a vaginal birth to manipulate the second twin into a presentation that allows it to be vaginally born. For internal version to occur, the cervix needs to be completely dilated.

Which assessment is least likely to be associated with a breech presentation? a.Meconium-stained amniotic fluid b.Fetal heart tones heard at or above the maternal umbilicus c.Preterm labor and birth d.Postterm gestation

ANS: D Postterm gestation is not likely to occur with a breech presentation. The presence of meconium in a breech presentation may be a result of pressure on the fetal wall as it traverses the birth canal. Fetal heart tones heard at the level of the umbilical level of the mother are a typical finding in a breech presentation because the fetal back would be located in the upper abdominal area. Breech presentations often occur in preterm births.

Which collection of risk factors will most likely result in damaging lacerations, including episiotomies? a.Dark-skinned woman who has had more than one pregnancy, who is going through prolonged second-stage labor, and who is attended by a midwife b.Reddish-haired mother of two who is going through a breech birth c.Dark-skinned first-time mother who is going through a long labor d.First-time mother with reddish hair whose rapid labor was overseen by an obstetrician

ANS: D Reddish-haired women have tissue that is less distensible than darker-skinned women and therefore may have less efficient healing. First-time mothers are also at greater risk, especially with breech births, long second-stage labors, or rapid labors during which the time for the perineum to stretch is insufficient. The rate of episiotomies is higher when obstetricians rather than midwives attend the births. The woman in the first scenario (a) is at low risk for either damaging lacerations or an episiotomy. She is multiparous, has dark skin, and is being attended by a midwife, who is less likely to perform an episiotomy. Reddish-haired women have tissue that is less distensible than that of darker-skinned women. Consequently, the client in the second scenario (b) is at increased risk for lacerations; however, she has had two previous deliveries, which result in a lower likelihood of an episiotomy. The fact that the woman in the third scenario (c) is experiencing a prolonged labor might increase her risk for lacerations. Fortunately, she is dark skinned, which indicates that her tissue is more distensible than that of fair-skinned women and therefore less susceptible to injury.

24. A 32-year-old primigravida is admitted with a diagnosis of ectopic pregnancy. Which information assists the nurse in developing the plan of care? a. Bed rest and analgesics are the recommended treatment. b. She will be unable to conceive in the future. c. A D&C will be performed to remove the products of conception. d. Hemorrhage is the primary concern.

ANS: D Severe bleeding occurs if the fallopian tube ruptures. The recommended treatment is to remove the pregnancy before rupture to prevent hemorrhaging. If the tube must be removed, then the woman's fertility will decrease; however, she will not be infertile. A D&C is performed on the inside of the uterine cavity. The ectopic pregnancy is located within the tubes. DIF: Cognitive Level: Apply REF: p. 676 TOP: Nursing Process: Planning MSC: Client Needs: Physiologic Integrity

The obstetric provider has informed the nurse that she will be performing an amniotomy on the client to induce labor. What is the nurse's highest priority intervention after the amniotomy is performed? a.Applying clean linens under the woman b.Taking the client's vital signs c.Performing a vaginal examination d.Assessing the fetal heart rate (FHR)

ANS: D The FHR is assessed before and immediately after the amniotomy to detect any changes that might indicate cord compression or prolapse. Providing comfort measures, such as clean linens, for the client is important but not the priority immediately after an amniotomy. The woman's temperature should be checked every 2 hours after the rupture of membranes but not the priority immediately after an amniotomy. The woman would have had a vaginal examination during the procedure. Unless cord prolapse is suspected, another vaginal examination is not warranted. Additionally, FHR assessment provides clinical cues to a prolapsed cord.

The Valsalva maneuver can be described as the process of making a forceful bearing-down attempt while holding one's breath with a closed glottis and a tightening of the abdominal muscles. When is it appropriate to instruct the client to use this maneuver? a.During the second stage to enhance the movement of the fetus bDuring the third stage to help expel the placenta c.During the fourth stage to expel blood clots d.Not at all

ANS: D The client should not be instructed to use this maneuver. This process stimulates the parasympathetic division of the autonomic nervous system and produces a vagal response (decrease in heart rate and blood pressure.) An alternative method includes instructing the client to perform open-mouth and open-glottis breathing and pushing.

Which statement regarding the care of a client in labor is correct and important to the nurse as he or she formulates the plan of care? a.The woman's blood pressure will increase during contractions and fall back to prelabor normal levels between contractions. b.The use of the Valsalva maneuver is encouraged during the second stage of labor to relieve fetal hypoxia. c.Having the woman point her toes will reduce leg cramps. d.Endogenous endorphins released during labor will raise the woman's pain threshold and produce sedation.

ANS: D The endogenous endorphins released during labor will raise the woman's pain threshold and produce sedation. In addition, physiologic anesthesia of the perineal tissues, caused by the pressure of the presenting part, decreases the mother's perception of pain. Blood pressure levels increase during contractions but remain somewhat elevated between them. The use of the Valsalva maneuver is discouraged during the second stage labor because of a number of unhealthy outcomes, including fetal hypoxia. Pointing the toes can cause leg cramps, as can the process of labor itself.

A new mother asks the nurse when the "soft spot" on her son's head will go away. What is the nurse's best response, based upon her understanding of when the anterior frontal closes? a.2 months b.8 months c.12 months d.18 months

ANS: D The larger of the two fontanels, the anterior fontanel, closes by 18 months after birth. The posterior fontanel closes at 6 to 8 weeks. The remaining three options are too early for the anterior fontanel to close.

The nurse performs a vaginal examination to assess a client's labor progress. Which action should the nurse take next? a.Perform an examination at least once every hour during the active phase of labor. b.Perform the examination with the woman in the supine position. c.Wear two clean gloves for each examination. d.Discuss the findings with the woman and her partner

ANS: D The nurse should discuss the findings of the vaginal examination with the woman and her partner, as well as report the findings to the primary care provider. A vaginal examination should be performed only when indicated by the status of the woman and her fetus. The woman should be positioned so as to avoid supine hypotension. The examiner should wear a sterile glove while performing a vaginal examination for a laboring woman.

In evaluating the effectiveness of magnesium sulfate for the treatment of preterm labor, which finding alerts the nurse to possible side effects? a.Urine output of 160 ml in 4 hours b.DTRs 2+ and no clonus c.Respiratory rate (RR) of 16 breaths per minute d.Serum magnesium level of 10 mg/dl

ANS: D The therapeutic range for magnesium sulfate management is 4 to 7.5 mg/dl. A serum magnesium level of 10 mg/dl could lead to signs and symptoms of magnesium toxicity, including oliguria and respiratory distress. Urine output of 160 ml in 4 hours, DTRs of 2+, and a RR of 16 breaths per minute are all normal findings.

A multiparous woman has been in labor for 8 hours. Her membranes have just ruptured. What is the nurse's highest priority in this situation? a.Prepare the woman for imminent birth. b.Notify the woman's primary health care provider. c.Document the characteristics of the fluid. d.Assess the fetal heart rate (FHR) and pattern.

ANS: D The umbilical cord may prolapse when the membranes rupture. The FHR and pattern should be closely monitored for several minutes immediately after the ROM to ascertain fetal well-being, and the findings should be documented. The ROM may increase the intensity and frequency of the uterine contractions, but it does not indicate that birth is imminent. The nurse may notify the primary health care provider after ROM occurs and the fetal well-being and response to ROM have been assessed. The nurse's priority is to assess fetal well-being. The nurse should document the characteristics of the amniotic fluid, but the initial response is to assess fetal well-being and the response to ROM.

A primigravida at 39 weeks of gestation is observed for 2 hours in the intrapartum unit. The FHR has been normal. Contractions are 5 to 9 minutes apart, 20 to 30 seconds in duration, and of mild intensity. Cervical dilation is 1 to 2 cm and uneffaced (unchanged from admission). Membranes are intact. What disposition would the nurse anticipate? a.Admitted and prepared for a cesarean birth b.Admitted for extended observation c.Discharged home with a sedative d.Discharged home to await the onset of true labor

ANS: D This situation describes a woman with normal assessments who is probably in false labor and will likely not deliver rapidly once true labor begins. No further assessments or observations are indicated; therefore, the client will be discharged along with instructions to return when contractions increase in intensity and frequency. Neither a cesarean birth nor a sedative is required at this time.

A client is in early labor, and her nurse is discussing the pain relief options she is considering. The client states that she wants an epidural "no matter what!" What is the nurse's best response? a."I'll make sure you get your epidural." b."You may only have an epidural if your physician allows it." c."You may only have an epidural if you are going to deliver vaginally." d."The type of analgesia or anesthesia used is determined, in part, by the stage of your labor and the method of birth."

ANS: D To avoid suppressing the progress of labor, pharmacologic measures for pain relief are generally not implemented until labor has advanced to the active phase of the first stage and the cervix is dilated approximately 4 to 5 cm. A plan of care is developed for each woman that addresses her particular clinical and nursing problems. The nurse collaborates with the primary health care provider and the laboring woman in selecting features of care relevant to the woman and her family. The decision whether to use an epidural to relieve labor pain is multifactorial. The nurse should not make a blanket statement guaranteeing the client one pharmacologic option over another until a complete history and physical examination has been obtained. A physician's order is required for pharmacologic options for pain management. However, expressing this requirement is not the nurse's best response. An epidural is an effective pharmacologic pain management option for many laboring women. It can also be used for anesthesia control if the woman undergoes an operative delivery.

Maternity nurses often have to answer questions about the many, sometimes unusual, ways people have tried to make the birthing experience more comfortable. Which information regarding nonpharmacologic pain relief isaccurate? a.Music supplied by the support person has to be discouraged because it could disturb others or upset the hospital routine. b.Women in labor can benefit from sitting in a bathtub, but they must limit immersion to no longer than 15 minutes at a time. c.Effleurage is permissible, but counterpressure is almost always counterproductive. d.Electrodes attached to either side of the spine to provide high-intensity electrical impulses facilitate the release of endorphins.

ANS: D Transcutaneous electrical nerve stimulation (TENS) may help and is most useful for lower back pain that occurs during the first stage of labor. Music may be very helpful for reducing tension and certainly can be accommodated by the hospital. Women can stay in a bath as long as they want, although repeated baths with breaks might be more effective than one long bath. Counterpressure can help the woman cope with lower back pain.

Which characteristic of a uterine contraction is not routinely documented? a.Frequency: how often contractions occur b.Intensity: strength of the contraction at its peak c.Resting tone: tension in the uterine muscle d.Appearance: shape and height

ANS: D Uterine contractions are described in terms of frequency, intensity, duration, and resting tone. Appearance is not routinely charted.

5. A woman at 39 weeks of gestation with a history of preeclampsia is admitted to the labor and birth unit. She suddenly experiences increased contraction frequency of every 1 to 2 minutes, dark red vaginal bleeding, and a tense, painful abdomen. Which clinical change does the nurse anticipate? a. Eclamptic seizure b. Rupture of the uterus c. Placenta previa d. Abruptio placentae

ANS: D Uterine tenderness in the presence of increasing tone may be the earliest sign of abruptio placentae. Women with preeclampsia are at increased risk for an abruption attributable to decreased placental perfusion. Eclamptic seizures are evidenced by the presence of generalized tonic-clonic convulsions. Uterine rupture exhibits hypotonic uterine activity, signs of hypovolemia, and, in many cases, the absence of pain. Placenta previa exhibits bright red, painless vaginal bleeding. DIF: Cognitive Level: Understand REF: p. 662 TOP: Nursing Process: Diagnosis MSC: Client Needs: Physiologic Integrity

A pregnant woman is at 38 weeks of gestation. She wants to know whether there are any signs that "labor is getting close to starting." Which finding is an indication that labor may begin soon? a.Weight gain of 1.5 to 2 kg (3 to 4 lb) b.Increase in fundal height c.Urinary retention d.Surge of energy

ANS: D Women speak of having a burst of energy before labor. The woman may lose 0.5 to 1.5 kg, as a result of water loss caused by electrolyte shifts that, in turn, are caused by changes in the estrogen and progesterone levels. When the fetus descends into the true pelvis (called lightening), the fundal height may decrease. Urinary frequency may return before labor.

10. Which statement best describes chronic hypertension? a. Chronic hypertension is defined as hypertension that begins during pregnancy and lasts for the duration of the pregnancy. b. Chronic hypertension is considered severe when the systolic BP is higher than 140 mm Hg or the diastolic BP is higher than 90 mm Hg. c. Chronic hypertension is general hypertension plus proteinuria. d. Chronic hypertension can occur independently of or simultaneously with preeclampsia.

ANS: D Women with chronic hypertension may develop superimposed preeclampsia, which increases the morbidity for both the mother and the fetus. Chronic hypertension is present before pregnancy or diagnosed before the 20 weeks of gestation and persists longer than 6 weeks postpartum. Chronic hypertension becomes severe with a diastolic BP of 110 mm Hg or higher. Proteinuria is an excessive concentration of protein in the urine and is a complication of hypertension, not a defining characteristic. DIF: Cognitive Level: Understand REF: p. 667 TOP: Nursing Process: Diagnosis | Nursing Process: Planning MSC: Client Needs: Physiologic Integrity

5. As a powerful central nervous system (CNS) stimulant, which of these substances can lead to miscarriage, preterm labor, placental separation (abruption), and stillbirth? a. Heroin b. Alcohol c. Phencyclidine (1-phenylcyclohexylpiperidine; PCP) d. Cocaine

ANS: D Cocaine is a powerful CNS stimulant. Effects on pregnancy associated with cocaine use include abruptio placentae, preterm labor, precipitous birth, and stillbirth. Heroin is an opiate; its use in pregnancy is associated with preeclampsia, intrauterine growth restriction, miscarriage, premature rupture of membranes, infections, breech presentation, and preterm labor. The most serious effect of alcohol use in pregnancy is FAS. The major concern regarding PCP use in pregnant women is its association with polydrug abuse and its neurobehavioral effects on the neonate.

11. A woman who is 28 weeks pregnant has edema in her lower legs bilaterally after working 8 hours a day as a cashier at a local grocery store. She is worried about her legs. What is the nurse's best response? A. "You will be at risk for development of varicose veins when your legs are edematous." B. "I would like to listen to your heart sounds. Edema can indicate a problem with your heart." C. "Edema is usually the result of too much salt and fluids in your diet. You may need to try to cut down on salty foods." D. "As your baby grows, it slows blood return from your legs, causing the swelling. This often occurs with prolonged standing."

ANS: D Edema of the lower extremities occurs because of the enlarging fetus, which impairs venous return. Prolonged standing worsens the edema. Typically, the bilateral, dependent edema experienced with pregnancy is not the result of cardiac pathology

26. During the assessment of a woman in her 22nd week of pregnancy, the nurse is unable to hear fetal heart tones with the fetoscope. The nurse should: A. Notify the physician immediately, then wait 10 minutes and try again. B. Ask the woman if she has felt the baby move today. C. Wait 10 minutes and try again. D. Use ultrasound to verify cardiac activity

ANS: D If no fetal heart tones are heard during auscultation with a fetoscope, then the nurse should verify cardiac activity using ultrasonography. This should be done before notifying the physician immediately or causing the woman distress by asking about fetal movement.

33. During an internal examination of a woman during her first prenatal visit, the nurse finds that thecervix is soft. This is known as _____ sign. A. Hegar's B. Chadwick's C. Homans' D. Goodell's

ANS: D Increased vascularity, congestion, and edema cause the cervix to soften (Goodell's sign) and become bluish purple (Chadwick's sign). The uterus becomes globular in shape, softens, and flexes easily over the cervix (Hegar's sign). This causes compression of the bladder, which results in urinary frequency. Homans' sign is pain in the calf upon dorsiflexion of the foot and is not related to signs of pregnancy

23. A woman in the 15th week of pregnancy has had bouts of severe vomiting for 3 weeks. She comes to the clinic today, and the nurse notices that she is showing signs of dehydration. Her blood pressure is lower than usual, and she is extremely fatigued. The nurse recognizes that this patient is experiencing: A. Preeclampsia. B. Polyhydramnios. C. Proteinuria. D. Hyperemesis

ANS: D Nausea and vomiting are not uncommon in pregnancy and usually resolve between weeks 16 and 20 and may be controlled with dietary and lifestyle changes or oral antiemetics. Hyperemesis is the extreme of vomiting and may require home infusions or hospitalization for fluid replacement. See Table 29-2 for a description of preeclampsia. Polyhydramnios is a condition where there is excessive amniotic fluid. Proteinuria is the presence of protein in the urine, which is not a normalfinding

2. A female patient has nausea, breast tenderness, fatigue, and amenorrhea. Her last menstrual period was 6 weeks ago. The nurse interprets that this patient is experiencing _____ signs of pregnancy. A. Positive B. Possible C. Probable D. Presumptive

ANS: D Presumptive signs of pregnancy are those that the woman experiences, and they include amenorrhea, breast tenderness, fatigue, nausea, and increased urinary frequency. Probable signs are those that are detected by the examiner, such as an enlarged uterus or changes in the cervix. Positive signs of pregnancy are those that document direct evidence of the fetus such as fetal heart tones or positive cardiac activity on ultrasound.

1. Which of these statements best describes the action of the hormone progesterone during pregnancy? A. It produces the hormone human chorionic gonadotropin. B. It stimulates duct formation in the breast. C. It promotes sloughing of the endometrial wall. D. It maintains the endometrium around the fetus.

ANS: D Progesterone prevents the sloughing of the endometrial wall and maintains the endometrium around the fetus. Progesterone increases the alveoli in the breast and keeps the uterus in a quiescent state. The other options are not correct

22. The nurse is palpating the uterus of a woman who is 8 weeks pregnant. Which of these findings would be considered to be most consistent with this stage of pregnancy? A. The uterus seems slightly enlarged and softened. B. It reaches to the pelvic brim and is about the size of a grapefruit. C. It rises above the pelvic brim and is about the size of a cantaloupe. D. It is about the size of an avocado, approximately 8 cm across the fundus.

ANS: D The 8-week pregnant uterus is approximately the size of an avocado, approximately 7 to 8 cm across the fundus. The 6-week pregnant uterus is slightly enlarged and softened. The 10-week pregnant uterus is about the size of a grapefruit and may reach to the pelvic brim. The 12-week pregnant uterus will fill the pelvis. At 12 weeks, the uterus is sized from the abdomen

32. During a woman's 34th week of pregnancy, she is told that she has preeclampsia. The nurse knows that which of these statements about preeclampsia is true? A. Preeclampsia has little effect on the fetus. B. Edema is one of the main indications of preeclampsia. C. Eclampsia only occurs before delivery of the baby. D. Untreated preeclampsia may contribute to restriction of fetal growth.

ANS: D Untreated preeclampsia may progress to eclampsia, which is manifested by generalized tonic-clonic seizures. Eclampsia may develop as late as 10 days postpartum. Before the syndrome becomes clinically manifested, it is affecting the placenta through vasospasm and a series of small infarctions. The placenta's capacity to deliver oxygen and nutrients may be seriously diminished, and fetal growth may be restricted. Edema is common in pregnancy and is not an indicator of preeclampsia

Which medical condition needs careful use of hormone replacement? Breast cancer Cardiac disease Venous thrombus All of the above

All of the above This option is correct because breast cancer, cardiac disease, and venous thrombus are major risks involved in hormone replacement therapy. Therefore, clients with these medical problems should be given hormone replacement therapy with extreme caution.

3. During an inpatient psychiatric hospitalization, what is the most important nursing intervention? a. Contacting the client's significant other b. Supervising and guiding visits with her infant c. Allowing no contact with anyone who annoys her d. Having the infant with the mother at all times

B In the hospital setting, the reintroduction of the infant to the mother can and should occur at the mother's own pace. A schedule is set that increases the number of hours the mother cares for her infant over several days, culminating in the infant staying overnight in the mother's room. These supervised and guided visits allow the mother to experience meeting the infant's needs and giving up sleep for the infant. Reintroducing the mother to her infant while in a supervised setting is essential. Another important task for a mother under psychiatric care is to reestablish positive interactions with others.

14. The use of methamphetamine (meth) has been described as a significant drug problem in the United States. The nurse who provides care to this client population should be cognizant of what regarding methamphetamine use? a. Methamphetamines are similar to opiates. b. Methamphetamines are stimulants with vasoconstrictive characteristics. c. Methamphetamines should not be discontinued during pregnancy. d. Methamphetamines are associated with a low rate of relapse.

B Methamphetamines are stimulants with vasoconstrictive characteristics similar to cocaine and are similarly used. As is the case with cocaine users, methamphetamine users are urged to immediately stop all use during pregnancy. Unfortunately, because methamphetamine users are extremely psychologically addicted, the rate of relapse is extremely high.

The 5 As screening intervention tool is used to implement smoking cessation strategies on the basis of patient response. What do the 5 As stand for? a. Ask, advise, administer, approve, and admit b. Ask, assess, advise, assist, and arrange follow-up c. Assess, assist, advise, apply, and arrange d. Assess, apply, advise, ask, and arrange follow-up

B The five major steps to intervention are the "5 A's": Ask, Advise, Assess, Assist, and Arrange

8. What three measures should the nurse implement to provide intrauterine resuscitation? a. Call the provider, reposition the mother, and perform a vaginal examination. b. Turn the client onto her side, provide oxygen (O2) via face mask, and increase intravenous (IV) fluids. c. Administer O2 to the mother, increase IV fluids, and notify the health care provider. d. Perform a vaginal examination, reposition the mother, and provide O2 via face mask.

B (Basic interventions for the management of any abnormal FHR pattern include administering O2 via a nonrebreather face mask at a rate of 8 to 10 L/min, assisting the woman onto a side-lying (lateral) position, and increasing blood volume by increasing the rate of the primary IV infusion. The purpose of these interventions is to improve uterine blood flow and intervillous space blood flow and to increase maternal oxygenation and cardiac output. The term intrauterine resuscitation is sometimes used to refer to these interventions. If these interventions do not quickly resolve the abnormal FHR issue, then the primary provider should be immediately notified.)

12. A new client and her partner arrive on the labor, delivery, recovery, and postpartum (LDRP) unit for the birth of their first child. The nurse applies the electronic fetal monitor (EFM) to the woman. Her partner asks you to explain what is printing on the graph, referring to the EFM strip. He wants to know what the babys heart rate should be. What is the nurses best response? a. Dont worry about that machine; thats my job. b. The babys heart rate will fluctuate in response to what is happening during labor. c. The top line graphs the babys heart rate, and the bottom line lets me know how strong the contractions are. d. Your physician will explain all of that later.

B (Explaining what indicates a normal FHR teaches the partner about fetal monitoring and provides support and information to alleviate his fears. Telling the partner not to worry discredits his feelings and does not provide the teaching he is requesting. Telling the partner that the graph indicates how strong the contractions are provides inaccurate information and does not address the partners concerns about the FHR. The EFM graphs the frequency and duration of the contractions, not their intensity. Nurses should take every opportunity to provide teaching to the client and her family, especially when information is requested.)

20. Which characteristic correctly matches the type of deceleration with its likely cause? a. Early decelerationumbilical cord compression b. Late decelerationuteroplacental insufficiency c. Variable decelerationhead compression d. Prolonged decelerationunknown cause

B (Late deceleration is caused by uteroplacental insufficiency. Early deceleration is caused by head compression. Variable deceleration is caused by umbilical cord compression. Prolonged deceleration has a variety of either benign or critical causes.)

17. During labor a fetus displays an average FHR of 135 beats per minute over a 10-minute period. Which statement best describes the status of this fetus? a. Bradycardia b. Normal baseline heart rate c. Tachycardia d. Hypoxia

B (The baseline FHR is measured over 10 minutes; a normal range is 110 to 160 beats per minute. Bradycardia is a FHR less than 110 beats per minute for 10 minutes or longer. Tachycardia is a FHR higher than 160 beats per minutes for 10 minutes or longer. Hypoxia is an inadequate supply of oxygen; no indication of hypoxia exists with a baseline FHR in the normal range.)

4. What is the most likely cause for variable FHR decelerations? a. Altered fetal cerebral blood flow b. Umbilical cord compression c. Uteroplacental insufficiency d. Fetal hypoxemia

B (Variable FHR decelerations can occur at any time during the uterine contracting phase and are caused by compression of the umbilical cord. Altered fetal cerebral blood flow results in early decelerations in the FHR. Uteroplacental insufficiency results in late decelerations in the FHR. Fetal hypoxemia initially results in tachycardia and then bradycardia if hypoxia continues.)

The lab reports of a patient in labor reveal 1+ proteinuria. Which physiologic mechanism is responsible for this lab finding? A. Increase in cervical dilation B. Breakdown of muscle tissue C. Increased white blood cell count D. Stagnation in the area of the vaginal introitus

B During normal labor, the patient's muscle tissues break down due to increased physical activity. Urine examination shows proteinuria of 1+, which is a normal finding in a patient who is in labor. Nausea and belching occur as a reflex response to full cervical dilation. An increase in white blood cell count does occur during labor; however, it is not related to normal proteinuria in labor. Great distensibility occurs in the area of the vaginal introitus during labor. Topics

What are the factors that enable the baby to initiate respiration immediately after birth? A. Fetal respiratory movements increase during labor. B. Fetal lung fluid is cleared from the air passage. C. Arterial carbon dioxide pressure is decreased. D. Arterial pH and bicarbonate level is increased

B Fetal lung fluid is cleared from the air passage as the infant passes through the birth canal during labor and vaginal birth. There is a decrease in fetal respiratory movements during labor. Arterial carbon dioxide pressure (Pco2) increases. There is a decrease in arterial pH and bicarbonate levels.

Nurses can help their patients by keeping them informed about the distinctive stages of labor. What description of the phases of the first stage of labor is accurate? A. Latent: mild, regular contractions; no dilation; bloody show; duration of 2 to 4 hours B. Active: moderate, regular contractions; 4 to 7 cm dilation; duration of 3 to 6 hours C. Lull: no contractions; dilation stable; duration of 20 to 60 minutes D. Transition: very strong but irregular contractions; 8 to 10 cm dilation; duration of 1 to 2 hours

B The active phase is characterized by moderate, regular contractions; 4 to 7 cm dilation; and a duration of 3 to 6 hours. The latent phase is characterized by mild to moderate, irregular contractions; dilation up to 3 cm; brownish to pale pink mucus; and a duration of 6 to 8 hours. No official "lull" phase exists in the first stage. The transition phase is characterized by strong to very strong, regular contractions; 8 to 10 cm dilation; and a duration of 20 to 40 minutes.

Concerning the third stage of labor, nurses should be aware that: A. the placenta eventually detaches itself from a flaccid uterus. B. the duration of the third stage may be as short as 3 to 5 minutes. C. it is important that the dark, roughened maternal surface of the placenta appear before the shiny fetal surface. D. the major risk for women during the third stage is a rapid heart rate.

B The duration may be as short as 3 to 5 minutes, although up to 1 hour is considered within normal limits. The third stage of labor lasts from birth of the fetus until the placenta is delivered. The placenta cannot detach itself from a flaccid (relaxed) uterus. Which surface of the placenta comes out first is not clinically important. The major risk for women during the third stage of labor is postpartum hemorrhage. The risk of hemorrhage increases as the length of the third stage increases.

Which pelvic shape is most conducive to vaginal labor and birth? A. Android B. Gynecoid C. Platypelloid D. Anthropoid

B The gynecoid pelvis is round and cylinder shaped, with a wide pubic arch. Prognosis for vaginal birth is good. Only 23% of women have an android-shaped pelvis, which has a poor prognosis for vaginal birth. The platypelloid pelvis is flat, wide, short, and oval. The anthropoid pelvis is a long, narrow oval with a narrow pubic arch. It is more favorable than the android or platypelloid pelvic shape.

The nurse is assessing a pregnant patient who is paralyzed due to a spinal injury at the level of the twelfth thoracic vertebra. Presently, she is in full-term gestation and under nursing care. What should the nurse inform the patient? A. "You may have a prolonged labor." B. "You may have painless uterine contractions." C. "Your uterus may not contract due to paralysis." D. "Your baby may develop neurologic problems."

B The pregnant patient is paralyzed due to a spinal lesion above the twelfth thoracic vertebra. In this case, the patient would not perceive the uterine contractions and thus would have painless uterine contractions. The spinal injury has no effect on the duration of labor. The uterine contractions are not dependent on any external forces, and thus this patient would have normal uterine contractions. Neurologic problems in the fetus are not a complication associated with spinal cord injury.

During the vaginal examination of a laboring patient, the nurse analyzes that the fetus is in the right occiput anterior (ROA) position at -1 station. What is the position of the lowermost portion of the fetal presenting part? A. 2 cm above the ischial spine. B. 1 cm above the ischial spine. C. at the level of the ischial spine. D. 1 cm below the ischial spine.

B When the lowermost portion of the presenting part is 1 cm above the ischial spine, it is noted as being minus (-)1. When positioned 2 cm above the ischial spine, it is -2 station. At the level of the spines the station is referred to as 0 (zero). When the presenting part is 1 cm below the spines, the station is said to be plus (+)1.

3. Which FHR decelerations would require the nurse to change the maternal position? (Select all that apply.) a. Early decelerations b. Late decelerations c. Variable decelerations d. Moderate decelerations e. Prolonged decelerations

B, C, E (Early decelerations (and accelerations) do not generally need any nursing intervention. Late decelerations suggest that the nurse should change the maternal position (lateral). Variable decelerations also require a maternal position change (side to side). Moderate decelerations are not an accepted category. Prolonged decelerations are late or variable decelerations that last for a prolonged period (longer than 2 minutes) and require intervention.)

2. According to the National Institute of Child Health and Human Development (NICHD) Three-Tier System of FHR Classification, category III tracings include all FHR tracings not categorized as category I or II. Which characteristics of the FHR belong in category III? (Select all that apply.) a. Baseline rate of 110 to 160 beats per minute b. Tachycardia c. Absent baseline variability not accompanied by recurrent decelerations d. Variable decelerations with other characteristics such as shoulders or overshoots e. Absent baseline variability with recurrent variable decelerations f. Bradycardia

B, D, E, F (Tachycardia, variable decelerations with other characteristics, absent baseline variability with recurrent variable decelerations, and bradycardia are characteristics that are considered nonreassuring or abnormal and belong in category III. A FHR of 110 to 160 beats per minute is considered normal and belongs in category I. Absent baseline variability not accompanied by recurrent decelerations is a category II characteristic.)

A 35-year-old female is diagnosed with endometeriosis. The present which of the following would be the most appropriate medical treatment for this disease? a. Infection b. Ovulation c. prostagleadiens d. Fertilization

B. Ovulation medical therapies include suppression of ovulation with various medications.

An 18-year- old female is diagnosed with dysmenorrhea. Which of the following symptoms will she most likely experience? a. Absence of menstruation b. Painful menstruation c. Unusually long menstrual period d. Menstrual irregularity

B. Painful menstruation Primary dysmenorrhea is painful menstruation associated with the release of prostaglandins in ovulatory cycles.

While assessing a 7-year-old female client, the primary health-care provider finds that the client has enlargement of the breasts, erection of the nipples, and development of pubic hair. Which hormonal levels can be found high in the client? Estrogen Gonadotropin Both A and B None of the above

Both A and B Secondary sexual characteristics must have developed in the client due to elevated levels of estrogen or gonadotropins. Therefore, both A and B are correct.

27. The nurse observes a sudden increase in variability on the ERM tracing. Which class of medications may cause this finding? a. Narcotics b. Barbiturates c. Methamphetamines d. Tranquilizers

C (Narcotics, barbiturates, and tranquilizers may be causes of decreased variability; whereas methamphetamines may cause increased variability.)

During pregnancy, alcohol withdrawal may be treated using: a. Disulfiram (Antabuse). b. Corticosteroids. c. Benzodiazepines. d. Aminophylline.

C Disulfiram is contraindicated in pregnancy because it is teratogenic. Corticosteroids are not used to treat alcohol withdrawal. Symptoms that occur during alcohol withdrawal can be managed with short-acting barbiturates or benzodiazepines. Aminophylline is not used to treat alcohol withdrawal.

12. A pregnant woman who abuses cocaine admits to exchanging sex to finance her drug habit. This behavior places the client at the greatest risk for what? a. Depression of the CNS b. Hypotension and vasodilation c. Sexually transmitted infections (STIs) d. Postmature birth

C Exchanging sex acts for drugs places the woman at increased risk for STIs because of multiple partners and the lack of protection. Cocaine is a CNS stimulant that causes hypertension and vasoconstriction. Premature delivery of the infant is one of the more common problems associated with cocaine use during pregnancy.

11. A woman at 24 weeks of gestation states that she has a glass of wine with dinner every evening. Why would the nurse counsel the client to eliminate all alcohol? a. Daily consumption of alcohol indicates a risk for alcoholism. b. She will be at risk for abusing other substances as well. c. The fetus is placed at risk for altered brain growth. d. The fetus is at risk for multiple organ anomalies.

C No period exists when consuming alcohol during pregnancy is safe. The documented effects of alcohol consumption during pregnancy include mental retardation, learning disabilities, high activity level, and short attention span. The brain grows most rapidly in the third trimester and is vulnerable to alcohol exposure during this time. Abuse of other substances has not been linked to alcohol use.

7. Which is the most accurate description of PPD without psychotic features? a. Postpartum baby blues requiring the woman to visit with a counselor or psychologist b. Condition that is more common among older Caucasian women because they have higher expectations c. Distinguishable by pervasive sadness along with mood swings d. Condition that disappears without outside help

C PPD is characterized by an intense pervasive sadness along with labile mood swings and is more persistent than postpartum baby blues. PPD, even without psychotic features, is more serious and persistent than postpartum baby blues. PPD is more common among younger mothers and African-American mothers. Most women need professional help to get through PPD, including pharmacologic intervention.

8. While providing care to the maternity client, the nurse should be aware that one of these anxiety disorders is likely to be triggered by the process of labor and birth. Which disorder fits this criterion? a. Phobias b. Panic disorder c. Posttraumatic stress disorder (PTSD) d. Obsessive-compulsive disorder (OCD)

C PTSD can occur as the result of a past trauma such as rape. Symptoms of PTSD include re-experiencing the event, numbing, irritability, angry outbursts, and exaggerated startle reflex. With the increased bodily touch and vaginal examinations that occur during labor, the client may have memories of the original trauma. The process of giving birth may result in her feeling out of control. The nurse should verbalize an understanding and reassure the client as necessary. Phobias are irrational fears that may lead a person to avoid certain events or situations. Panic disorders may occur in as many as 3% to 5% of women in the postpartum period and are described as episodes of intense apprehension, fear, and terror. Symptoms of a panic disorder may include palpitations, chest pain, choking, or smothering. OCD symptoms include recurrent, persistent, and intrusive thoughts. The mother may repeatedly check and recheck her infant once he or she is born, although she realizes that this behavior is irrational. OCD is optimally treated with medications.

9. Which substance used during pregnancy causes vasoconstriction and decreased placental perfusion, resulting in maternal and neonatal complications? a. Alcohol b. Caffeine c. Tobacco d. Chocolate

C Smoking in pregnancy is known to cause a decrease in placental perfusion and is the cause of low-birth-weight infants. Prenatal alcohol exposure is the single greatest preventable cause of mental retardation. Alcohol use during pregnancy can cause high blood pressure, miscarriage, premature birth, stillbirth, and anemia. Caffeine may interfere with certain medications and worsen arrhythmias. Chocolate, particularly dark chocolate, contains caffeine that may interfere with certain medications.

2. When a woman is diagnosed with postpartum depression (PPD) with psychotic features, what is the nurse's primary concern in planning the client's care? a. Displaying outbursts of anger b. Neglecting her hygiene c. Harming her infant d. Losing interest in her husband

C Thoughts of harm to herself or to the infant are among the most serious symptoms of PPD and require immediate assessment and intervention. Although outbursts of anger and neglecting personal hygiene are symptoms attributable to PPD, the major concern remains the potential of harm to herself or her infant. Although this client is likely to lose interest in her spouse, it is not the nurse's primary concern.

24. In which clinical situation would the nurse most likely anticipate a fetal bradycardia? a. Intraamniotic infection b. Fetal anemia c. Prolonged umbilical cord compression d. Tocolytic treatment using terbutaline

C (Fetal bradycardia can be considered a later sign of fetal hypoxia and is known to occur before fetal death. Bradycardia can result from placental transfer of drugs, prolonged compression of the umbilical cord, maternal hypothermia, and maternal hypotension. Intraamniotic infection, fetal anemia, and tocolytic treatment using terbutaline would most likely result in fetal tachycardia.)

9. The nurse who provides care to clients in labor must have a thorough understanding of the physiologic processes of maternal hypotension. Which outcome might occur if the interventions for maternal hypotension are inadequate? a. Early FHR decelerations b. Fetal arrhythmias c. Uteroplacental insufficiency d. Spontaneous rupture of membranes

C (Low maternal blood pressure reduces placental blood flow during uterine contractions, resulting in fetal hypoxemia. Maternal hypotension does not result in early FHR decelerations nor is it associated with fetal arrhythmias. Spontaneous rupture of membranes is not a result of maternal hypotension.)

21. Which information related to a prolonged deceleration is important for the labor nurse to understand? a. Prolonged decelerations present a continuing pattern of benign decelerations that do not require intervention. b. Prolonged decelerations constitute a baseline change when they last longer than 5 minutes. c. A disruption to the fetal oxygen supply causes prolonged decelerations. d. Prolonged decelerations require the customary fetal monitoring by the nurse.

C (Prolonged decelerations are caused by a disruption in the fetal oxygen supply. They usually begin as a reflex response to hypoxia. If the disruption continues, then the fetal cardiac tissue, itself, will become hypoxic, resulting in direct myocardial depression of the FHR. Prolonged decelerations can be caused by prolonged cord compression, uteroplacental insufficiency, or perhaps sustained head compression. Prolonged decelerations lasting longer than 10 minutes are considered a baseline change that may require intervention. A prolonged deceleration is a visually apparent decrease (may be either gradual or abrupt) in the FHR of at least 15 beats per minute below the baseline and lasting longer than 2 minutes but shorter than 10 minutes. Nurses should immediately notify the physician or nurse-midwife and initiate appropriate treatment of abnormal patterns when they see prolonged decelerations.)

15. What is a distinct advantage of external EFM? a. The ultrasound transducer can accurately measure short-term variability and beat-to-beat changes in the FHR. b. The tocotransducer can measure and record the frequency, regularity, intensity, and approximate duration of uterine contractions. c. The tocotransducer is especially valuable for measuring uterine activity during the first stage of labor. d. Once correctly applied by the nurse, the transducer need not be repositioned even when the woman changes positions.

C (The tocotransducer is valuable for measuring uterine activity during the first stage of labor and is especially true when the membranes are intact. Short-term variability and beat-to-beat changes cannot be measured with this technology. The tocotransducer cannot measure and record the intensity of uterine contractions. The transducer must be repositioned when the woman or the fetus changes position.)

5. The nurse providing care for a high-risk laboring woman is alert for late FHR decelerations. Which clinical finding might be the cause for these late decelerations? a. Altered cerebral blood flow b. Umbilical cord compression c. Uteroplacental insufficiency d. Meconium fluid

C (Uteroplacental insufficiency results in late FHR decelerations. Altered fetal cerebral blood flow results in early FHR decelerations. Umbilical cord compression results in variable FHR decelerations. Meconium-stained fluid may or may not produce changes in the FHR, depending on the gestational age of the fetus and whether other causative factors associated with fetal distress are present.)

The nurse assisting a laboring patient recognizes the Ferguson reflex in the patient. What is the Ferguson reflex? A. Release of endogenous oxytocin B. Involuntary uterine contractions C. Maternal urge to bear down D. Mechanical stretching of the cervix

C The maternal urge to bear down is known as the Ferguson reflex. The Ferguson reflex occurs when stretch receptors in the posterior vagina cause the release of endogenous oxytocin. The involuntary uterine contractions or primary powers originate at certain pacemaker points in the thickened muscle layers of the upper uterine segment. Intrauterine pressure caused by contractions exerts pressure on the descending fetus and the cervix. When the presenting part of the fetus reaches the perineal floor, mechanical stretching of the cervix occurs.

In which stage of labor does the nurse expect the placenta to be expelled? A. First B. Second C. Third D. Fourth

C The placenta is expelled in the third stage of labor. The placenta normally separates with the third or fourth strong uterine contraction after the infant has been born. The first stage of labor lasts from the time dilation begins to the time when the cervix is fully dilated. The second stage of labor lasts from the time of full cervical dilation to the birth of the infant. The fourth stage of labor lasts for the first 2 hours after birth.

The nurse is assessing a client who is 6 months pregnant. The nurse determines that the fetus is lying in a longitudinal position with the sacrum as the presenting part and with general flexion. What fetal position should the nurse document? A. Cephalic presentation B. Shoulder presentation C. Complete breech position D. Single footling breech position

C While assessing a pregnant client, the nurse should check the fetal lie, fetal attitude, and the presenting part in order to determine the fetal position. If the fetus lies in the longitudinal or vertical position with sacrum and feet as the presenting part and with general flexion it indicates that the fetus is in complete breech position. If the presenting part is the head instead of sacrum, then the fetus is in the cephalic position. If the fetus lies in the longitudinal or vertical position with the sacrum as the presenting part but with only one leg extended at the hip and knee, the fetus is in single footling breech.

Where do most deliveries for pregnant women who have mental health issues take place? a. Mental health hospital setting b. Midwife assisted births c. Community hospital settings d. Psychiatric hospitals on locked units

C - Community hospital settings Most pregnant women who have mental health issues receive care and deliver in community settings. Although midwives are trained to provide obstetrical care, they typically do not take care of complex patients, and a woman who has a diagnosed mental health issue would be classified as a complex patient. Unless there is some specific psychiatric mental health issue that requires a client being on a locked unit, most pregnant women who have mental health issues deliver in community settings.

Nurses must be cognizant of the growing problem of methamphetamine use during pregnancy. When caring for a woman who uses methamphetamines, it is important for the nurse to be aware of which factor related to the abuse of this substance? a. Methamphetamine is a depressant. b. All methamphetamines are vasodilators. c. Methamphetamine users are extremely psychologically addicted. d. Rehabilitation is usually successful.

C Methamphetamine users are extremely psychologically addicted. Typically these women display poor control over their behavior and a low threshold for pain. This substance is relatively inexpensive and easy to obtain. Methamphetamines are vasoconstrictors. The rate of relapse for methamphetamine users is very high.

A nurse is advising a pregnant patient who has a substance abuse problem about a contingency management program. Which statement identifies an aspect of this type of program? a. Pregnant woman are confined to an inpatient treatment method during their pregnancy. b. Pregnant woman are given biofeedback modalities as stimulus responses to control their addiction. c. Pregnant woman are given motivational incentives as a primary approach to stop their drug abuse problem. d. Pregnant woman must follow a strict medication nutritional program during the course of pregnancy.

C Pregnant woman are given motivational incentives as a primary approach to stop their drug abuse problem. A contingency management program utilizes a motivational incentive approach with clients to support their efforts to maintain abstinence. The incentives may include small cash amounts, privileges, or prizes. Contingency management programs are not limited to inclient settings and do not involve biofeedback modalities or medication nutritional programs.

5. The baseline FHR is the average rate during a 10-minute segment. Changes in FHR are categorized as periodic or episodic. These patterns include both accelerations and decelerations. The labor nurse is evaluating the clients most recent 10-minute segment on the monitor strip and notes a late deceleration. Which is likely to have caused this change? (Select all that apply.) a. Spontaneous fetal movement b. Compression of the fetal head c. Placental abruption d. Cord around the babys neck e. Maternal supine hypotension

C, E (Late decelerations are almost always caused by uteroplacental insufficiency. Insufficiency is caused by uterine tachysystole, maternal hypotension, epidural or spinal anesthesia, IUGR, intraamniotic infection, or placental abruption. Spontaneous fetal movement, vaginal examination, fetal scalp stimulation, fetal reaction to external sounds, uterine contractions, fundal pressure, and abdominal palpation are all likely to cause accelerations of the FHR. Early decelerations are most often the result of fetal head compression and may be caused by uterine contractions, fundal pressure, vaginal examination, and the placement of an internal electrode. A variable deceleration is likely caused by umbilical cord compression, which may happen when the umbilical cord is around the babys neck, arm, leg, or other body part or when a short cord, a knot in the cord, or a prolapsed cord is present.)

The most commonly occurring cancer of the female reproductive tract is cancer. a. Cervical b. Ovarian c. Endometrial d. Fallopian

C. Endometrial Carcinoma of the endometrium is the most common type of uterine cancer and most prevalent gynecologic malignancy.

A 20-year-old female presents with pelvic and back pain severe enough to miss work beginning with the onset of menses, Physcial examination supports the diagnosis of primary dysmenorrhea. The most likely cause of her condition is : a. lack of estrogen b. stress c. elevated prostaglandins d. poor nutrition

C. elevated prostaglandins Primary dysmenorrhea is painful menstruation associated with the release of prostaglandins in ovulatory cycles.

The 40-year-old female presents with cramping and excessive vaginal bleeding. Ultrasounds reveals benign uterine tumors in the smooth muscle cells of the myometrium. These tumors are commonly called: a. Adenomyosis b. Endometriosis c. Leionyomas d. adenomas

C. leiomyomas Leionyomas are benign tumors that develop from smooth muscle cells in the myrometrium.

In the majority of cases delayed puberty is due to: a. a disruption in the hypothalamus b.a disruption of the pituitary c. slow maturation d. ambiguity in sex determination

C. slow maturation

1. Reports have linked third trimester use of selective serotonin uptake inhibitors (SSRIs) with a constellation of neonatal signs. The nurse is about to perform an assessment on the infant of a mother with a history of a mood disorder. Which signs and symptoms in the neonate may be the result of maternal SSRI use? (Select all that apply.) a. Hypotonia b. Hyperglycemia c. Shivering d. Fever e. Irritability

CDE Neonatal signs of maternal SSRI use include continuous crying, irritability, jitteriness, shivering, fever, hypertonia, respiratory distress, feeding difficulty, hypoglycemia, and seizures. The onset of signs and symptoms ranges from several hours to several days after birth, but the signs generally resolve within 2 weeks.

Which drugs are used to treat hyperprolactinemia? Select all that apply. Clozapine Cabergoline Risperidone Bromocriptine Metoclopramide

Cabergoline Bromocriptine

To provide adequate postpartum care, the nurse should be aware that postpartum depression (PPD) with psychotic features: A. Is more likely to occur in women with more than two children. B. Is rarely delusional and then usually about someone trying to harm her (the mother). C. Although serious, is not likely to need psychiatric hospitalization. D. May include bipolar disorder (formerly called "manic depression").

D

Which of these medications would be classified as a Category X substance that is not to be used during pregnancy? a. Lorazepam (Ativan) b. Alprazolam (Xanax) c. Chlordiazepoxide (Librium) d. Temazepam (Restoril)

D Restoril is classified as a Category X drug and is contraindicated during pregnancy on the basis of clinical studies. Ativan, Xanax, and Librium are classified as Category D drugs and as such would not be given during pregnancy unless a benefits to risks ratio was established.

10. As part of the discharge teaching, the nurse can prepare the mother for her upcoming adjustment to her new role by instructing her regarding self-care activities to help prevent PPD. Which statement regarding this condition is most helpful for the client? a. Stay home, and avoid outside activities to ensure adequate rest. b. Be certain that you are the only caregiver for your baby to facilitate infant attachment. c. Keep your feelings of sadness and adjustment to your new role to yourself. d. Realize that PPD is a common occurrence that affects many women.

D Should the new mother experience symptoms of the baby blues, it is important that she be aware that these symptoms are nothing to be ashamed of. As many as 10% to 15% of new mothers experience similar symptoms. Although obtaining enough rest is important for the mother, she should not distance herself from her family and friends. Her spouse or partner can communicate the best visiting times to enable the new mother to obtain adequate rest. It is also important that she not isolate herself at home by herself during this time of role adjustment. Even if breastfeeding, other family members can participate in the infant's care. If depression occurs, then the symptoms will often interfere with mothering functions; therefore, family support is essential. The new mother should share her feelings with someone else and avoid overcommitting herself or feel as though she has to be superwoman. A telephone call to the hospital "warm line" may provide reassurance with lactation issues and other infant care questions. Should symptoms continue, a referral to a professional therapist may be necessary.

19. Which definition of an acceleration in the fetal heart rate (FHR) is accurate? a. FHR accelerations are indications of fetal well-being when they are periodic. b. FHR accelerations are greater and longer in preterm gestations. c. FHR accelerations are usually observed with breech presentations when they are episodic. d. An acceleration in the FHR presents a visually apparent and abrupt peak.

D (Acceleration of the FHR is defined as a visually apparent abrupt (only to peak 30 seconds) increase in the FHR above the baseline rate. Periodic accelerations occur with uterine contractions and are usually observed with breech presentations. Episodic accelerations occur during fetal movement and are indications of fetal well-being. Preterm accelerations peak at 10 beats per minute above the baseline and last for at least 10 seconds.)

29. What physiologic change occurs as the result of increasing the infusion rate of nonadditive IV fluids? a. Maintaining normal maternal temperature b. Preventing normal maternal hypoglycemia c. Increasing the oxygen-carrying capacity of the maternal blood d. Expanding maternal blood volume

D (Filling the mothers vascular system increases the amount of blood available to perfuse the placenta and may correct hypotension. Increasing fluid volume may alter the maternal temperature only if she is dehydrated. Most IV fluids for laboring women are isotonic and do not provide extra glucose. Oxygen-carrying capacity is increased by adding more red blood cells.)

7. Which FHR finding is the most concerning to the nurse who is providing care to a laboring client? a. Accelerations with fetal movement b. Early decelerations c. Average FHR of 126 beats per minute d. Late decelerations

D (Late decelerations are caused by uteroplacental insufficiency and are associated with fetal hypoxemia. Late FHR decelerations are considered ominous if they are persistent and left uncorrected. Accelerations with fetal movement are an indication of fetal well-being. Early decelerations in the FHR are associated with head compression as the fetus descends into the maternal pelvic outlet; they are not generally a concern during normal labor. An FHR finding of 126 beats per minute is normal and not a concern.)

14. The nurse is using intermittent auscultation (IA) to locate the fetal heartbeat. Which statement regarding this method of surveillance is accurate? a. The nurse can be expected to cover only two or three clients when IA is the primary method of fetal assessment. b. The best course is to use the descriptive terms associated with EFM when documenting results. c. If the heartbeat cannot be immediately found, then a shift must be made to EFM. d. Ultrasound can be used to find the FHR and to reassure the mother if the initial difficulty is a factor.

D (Locating fetal heartbeats often takes time. Mothers can be verbally reassured and reassured by viewing the ultrasound pictures if that device is used to help locate the heartbeat. When used as the primary method of fetal assessment, IA requires a nurse-to-client ratio of one to one. Documentation should use only terms that can be numerically defined; the usual visual descriptions of EFM are inappropriate.)

11. The perinatal nurse realizes that an FHR that is tachycardic, bradycardic, has late decelerations, or loss of variability is nonreassuring and is associated with which condition? a. Hypotension b. Cord compression c. Maternal drug use d. Hypoxemia

D (Nonreassuring FHR patterns are associated with fetal hypoxemia. Fetal bradycardia may be associated with maternal hypotension. Variable FHR decelerations are associated with cord compression. Maternal drug use is associated with fetal tachycardia.)

22. In which situation would the nurse be called on to stimulate the fetal scalp? a. As part of fetal scalp blood sampling b. In response to tocolysis c. In preparation for fetal oxygen saturation monitoring d. To elicit an acceleration in the FHR

D (The scalp can be stimulated using digital pressure during a vaginal examination. Fetal scalp blood sampling involves swabbing the scalp with disinfectant before a sample is collected. The nurse stimulates the fetal scalp to elicit an acceleration of the FHR. Tocolysis is relaxation of the uterus. Fetal oxygen saturation monitoring involves the insertion of a sensor.)

A primigravida asks the nurse about signs she can look for that indicate that the onset of labor is getting closer. The nurse should describe: A. weight gain of 1 to 3 lbs. B. quickening. C. fatigue and lethargy. D. bloody show.

D Passage of the mucous plug (operculum) also termed pink/bloody show occurs as the cervix ripens. Women usually experience a weight loss of 1 to 3 lbs. Quickening is the perception of fetal movement by the mother, which occurs at 16 to 20 weeks of gestation. Women usually experience a burst of energy or the nesting instinct.

In order to assess the health of the mother accurately during labor, the nurse should be aware that: A. the woman's blood pressure increases during contractions and falls back to prelabor normal between contractions. B. use of the Valsalva maneuver is encouraged during the second stage of labor to relieve fetal hypoxia. C. having the woman point her toes reduces leg cramps. D. the endogenous endorphins released during labor raise the woman's pain threshold and produce sedation.

D Physiologic anesthesia of the perineal tissues, caused by the pressure of the presenting part, decreases the mother's perception of pain. Blood pressure increases during contractions but remains somewhat elevated between them. Use of the Valsalva maneuver is discouraged during second stage labor because of a number of potentially unhealthy outcomes, including fetal hypoxia. Pointing the toes can cause leg cramps, as can the process of labor itself.

The nurse is monitoring the fetal heart rate (FHR) of a patient in week 20 of gestation. What FHR can the nurse expect at this stage? A. 100 beats/min B. 120 beats/min C. 140 beats/min D. 160 beats/min

D The FHR is higher earlier in the gestation. At 20 weeks' gestation, the FHR on an average is 160 beats/min approximately. The rate decreases progressively as the maturing fetus reaches term. An FHR of 100 beats/min is below the normal FHR. The normal range of FHR is 110 to 160 beats/min. An FHR of 120 beats/min at 20 weeks' gestation is not normal. The average FHR at term is 140 beats/min.

During the vaginal examination of a patient in labor, the nurse identifies the presenting part as the scapula. Which fetal presentation does the nurse recognize? A. Cephalic B. Frank breech C. Complete breech D. Shoulder

D The presenting part can be defined as that part of the fetus that lies closest to the internal os of the cervix. In the shoulder presentation, the presenting part is the scapula. In a cephalic presentation, the presenting part is usually the occiput. In a breech presentation, the presenting part is the sacrum. The sacrum is the presenting part in a frank breech presentation. The sacrum and feet are the presenting parts in a complete breech presentation.

A 25-year-old female presents with amenorrhea and hirsutism and is diagnosed with polycystic ovary syndrome (PCOS), Lab testing willl most likely reveal: a. estrogen deficit b. genetic cancerous mutations c. cortisol excess d. hyperinsulinemia

D hyperinsulinemia Glucose intolerance/insulin resistance (IR) and hyperinsulinemia often run parallel and markly aggravate the hyperandrogenic state, thus contributing to the severity of signs and symptoms of PCOS.

Which of the following signs is usually the first clinical manifestation of breast cancer? a. Dimpling b. Nipple discharge c. Chest pain d. A painless lump

D. A painless lump. The first sign of breast cancer is usually a painless lump.

During a routine pelvic exam, a 34-year-old female is found to have an ovarian cyst containing skin, hair, cartilage, and bone. The sis is classified as a_________ cyst. a. Follicular b. Dermoid c. Corpus luteal d. Micro

D. dermoid Dermoid cysts are growths that may contain mutate tissues included skin, hair, sebaceous and sweat glands, muscle fibers, cartilage, and bone.

A female client reports missed menstrual periods after discontinuing hormonal contraceptives. What could be the reason behind the missed menstrual periods? Obstruction in the cervix Damage to the fallopian tube Hypothalamic-pituitary-ovarian dysfunction Extra growth of uterine tissue

Hypothalamic-pituitary-ovarian dysfunction Discontinuation of hormonal contraceptives may lead to a dysfunction of the hypothalamic-pituitary-ovarian axis, preventing the release of the egg by the ovaries. This results in missed menstrual periods.

A woman who has tested positive on a serum beta-human chorionic gonadotropin (b-HCG) test had her last menstrual period on April 18, 2015. What would be the estimated date of childbirth, according to Naegele's rule? January 15, 2016 January 20, 2016 January 25, 2016 January 30, 2016

January 25, 2016 Naegele's rule calculates the childbirth by adding 1 year to the last menstrual period, subtracting 3 months from it, and then adding 7 days to it. Therefore, if the last menstrual period is April 18, 2015, then adding 1 year to it would be April 18, 2016. Subtracting 3 months from April 18, 2016 is January 18, 2016. Adding 7 days to that would be January 25, 2016. Therefore, January 25, 2016 is the estimated date of childbirth.

While assessing a female client for anomalies of the genital tract, the primary health-care professional observes that the uterus is absent in the client and the vagina is foreshortened. However, the client's breast is normal in shape and contour. Which menstrual disorder would the nurse suspect in the client? Dysmenorrhea Mayer-Rokitansky-Hauser syndrome Premenstrual syndrome Menorrhea

Mayer-Rokitansky-Hauser syndrome Absence of the uterus and a foreshortened vagina are the signs of Mayer-Rokitansky-Hauser syndrome. This syndrome can be responsible for amenorrhea, which is the absence of a menstrual cycle.

A client diagnosed with dysmenorrhea is prescribed ibuprofen and hormonal contraceptives. Which outcome in the client indicates the effectiveness of the therapy? No painful experience during the first 2 days of her menstrual period No behavioral change during the luteal phase of the menstrual cycle No formation of multiple regions of unreleased egg on the ovarian surface All of the above

No painful experience during the first 2 days of her menstrual period Painful menstruation is called dysmenorrhea. Nonsteroidal anti-inflammatory drugs like ibuprofen, in addition to hormonal contraceptives, are recommended to reduce the pain in clients with dysmenorrhea. Therefore, if the client experiences no pain during the first 2 days of her menstrual period, it indicates an effective outcome.

Which complications should a nurse expect in a female client caused by atrophic vaginitis during the perimenopausal stage? Select all that apply. Hot flashes Night sweats Irregular menses Painful sexual intercourse Vaginal yeast infection

Painful sexual intercourse Vaginal yeast infection

Which medication is associated with the growth of endometrial polyps? Tamoxifen Ibuprofen Haloperidol Matoclopramide

Tamoxifen Tamoxifen is used in the treatment of breast cancer. It stimulates the growth of endometrial tissue resulting in the growth of endometrial polyps.

While reviewing a client's medical record, a nurse finds the client to be described as gravida 2, para 2, or G2 P2. What will the nurse interpret from this description? The client had two deliveries and two abortions. The client had two pregnancies and both of them were aborted. The client had one delivery after 24 weeks and two pregnancies. The client had two pregnancies and two deliveries after 24 weeks.

The client had two pregnancies and two deliveries after 24 weeks.

A client with premenstrual syndrome (PMS) is prescribed selective serotonin receptor inhibitors (SSRIs) and nutritional supplements. Which symptom in the client indicates the effectiveness of the therapy? The client has decreased irritability and mood swings. The client no longer experiences painful menstruation. The client's blood report shows no serum beta-human chorionic gonadotropin (b-HCG). The client has elevated levels of progesterone.

The client has decreased irritability and mood swings. PMS is characterized by irritability and mood swings. Therapy with SSRIs and nutritional supplements will provide relief from symptoms of PMS.

A night-shift nurse documents "primigravida" for a client. Which client is being referred to by the nurse? The client who hasn't had a pregnancy. The client who had multiple pregnancies. The client who is carrying her first pregnancy. The client who has delivered a baby after her first pregnancy.

The client who is carrying her first pregnancy.

The transabdominal ultrasound examination of a female client shows the presence of fibroid tumors. Which treatment options should the nurse anticipate from the primary health-care provider to suppress the growth of the tumors? Select all that apply. Myomectomy Hysterectomy Ablation of the endometrium Uterine artery embolization Injection of hypothalamic gonadotropic-releasing hormone (GnRH)

Uterine artery embolization Injection of hypothalamic gonadotropic-releasing hormone (GnRH)

21. A pregnant woman's biophysical profile score is 8. She asks the nurse to explain the results. The nurse's best response is: a. "The test results are within normal limits." b. "Immediate delivery by cesarean birth is being considered." c. "Further testing will be performed to determine the meaning of this score." d. "An obstetric specialist will evaluate the results of this profile and, within the next week, will inform you of your options regarding delivery."

a. "The test results are within normal limits."

8. A client asks her nurse, "My doctor told me that he is concerned with the grade of my placenta because I am overdue. What does that mean?" The best response by the nurse is: a. "Your placenta changes as your pregnancy progresses, and it is given a score that indicates the amount of calcium deposits it has. The more calcium deposits, the higher the grade, or number, that is assigned to the placenta. It also means that less blood and oxygen can be delivered to your baby." b. "Your placenta isn't working properly, and your baby is in danger." c. "This means that we will need to perform an amniocentesis to detect if you have any placental damage." d. "Don't worry about it. Everything is fine."

a. "Your placenta changes as your pregnancy progresses, and it is given a score that indicates the amount of calcium deposits it has. The more calcium deposits, the higher the grade, or number, that is assigned to the placenta. It also means that less blood and oxygen can be delivered to your baby."

13. Nurses should be aware that the biophysical profile (BPP): a. Is an accurate indicator of impending fetal death. b. Is a compilation of health risk factors of the mother during the later stages of pregnancy. c. Consists of a Doppler blood flow analysis and an amniotic fluid index. d. Involves an invasive form of ultrasound examination.

a. Is an accurate indicator of impending fetal death.

22. Which analysis of maternal serum may predict chromosomal abnormalities in the fetus? a. Multiple-marker screening b. Lecithin/sphingomyelin (L/S) ratio c. Biophysical profile d. Type and crossmatch of maternal and fetal serum

a. Multiple-marker screening

9. A woman is undergoing a nipple-stimulated contraction stress test (CST). She is having contractions that occur every 3 minutes. The fetal heart rate (FHR) has a baseline of approximately 120 beats/min without any decelerations. The interpretation of this test is said to be: a. Negative. b. Positive. c. Satisfactory. d. Unsatisfactory.

a. Negative.

2. A 39-year-old primigravida thinks that she is about 8 weeks pregnant, although she has had irregular menstrual periods all her life. She has a history of smoking approximately one pack of cigarettes a day, but she tells you that she is trying to cut down. Her laboratory data are within normal limits. What diagnostic technique could be used with this pregnant woman at this time? a. Ultrasound examination b. Maternal serum alpha-fetoprotein (MSAFP) screening c. Amniocentesis d. Nonstress test (NST)

a. Ultrasound examination

Conditions that contribute to dysfunctional uterine bleeding (DUB) include: a. polycystic ovarian syndrome (PCOS) b. obesity c.thyroid disease d. excessive exercise

a. polycystic ovarian syndrome (PCOS). b. obesity c. thyroid disease

4. A 41-week pregnant multigravida presents in the labor and delivery unit after a nonstress test indicated that her fetus could be experiencing some difficulties in utero. Which diagnostic tool would yield more detailed information about the fetus? a. Ultrasound for fetal anomalies b. Biophysical profile (BPP) c. Maternal serum alpha-fetoprotein (MSAFP) screening d. Percutaneous umbilical blood sampling (PUBS)

b. Biophysical profile (BPP)

18. A woman has been diagnosed with a high risk pregnancy. She and her husband come into the office in a very anxious state. She seems to be coping by withdrawing from the discussion, showing declining interest. The nurse can best help the couple by: a. Telling her that the physician will isolate the problem with more tests. b. Encouraging her and urging her to continue with childbirth classes. c. Becoming assertive and laying out the decisions the couple needs to make. d. Downplaying her risks by citing success rate studies.

b. Encouraging her and urging her to continue with childbirth classes.

24. Which nursing intervention is necessary before a second-trimester transabdominal ultrasound? a. Place the woman NPO for 12 hours. b. Instruct the woman to drink 1 to 2 quarts of water. c. Administer an enema. d. Perform an abdominal preparation.

b. Instruct the woman to drink 1 to 2 quarts of water.

12. In the first trimester, ultrasonography can be used to gain information on: a. Amniotic fluid volume. b. Location of Gestational sacs c. Placental location and maturity. d. Cervical length.

b. Location of Gestational sacs

20. Risk factors tend to be interrelated and cumulative in their effect. While planning the care for a laboring client with diabetes mellitus, the nurse is aware that she is at a greater risk for: a. Oligohydramnios. b. Polyhydramnios. c. Postterm pregnancy. d. Chromosomal abnormalities.

b. Polyhydramnios.

23. While working with the pregnant woman in her first trimester, the nurse is aware that chorionic villus sampling (CVS) can be performed during pregnancy at: a. 4 weeks b. 8 weeks c. 10 weeks d. 14 weeks

c. 10 weeks

5. At 35 weeks of pregnancy a woman experiences preterm labor. Tocolytics are administered and she is placed on bed rest, but she continues to experience regular uterine contractions, and her cervix is beginning to dilate and efface. What would be an important test for fetal well-being at this time? a. Percutaneous umbilical blood sampling (PUBS) b. Ultrasound for fetal size c. Amniocentesis for fetal lung maturity d. Nonstress test (NST)

c. Amniocentesis for fetal lung maturity

19. In the past, factors to determine whether a woman was likely to develop a high risk pregnancy were evaluated primarily from a medical point of view. A broader, more comprehensive approach to high-risk pregnancy has been adopted today. There are now four categories based on threats to the health of the woman and the outcome of pregnancy. Which of the following is not one of these categories? a. Biophysical b. Psychosocial c. Geographic d. Environmental

c. Geographic

17. The nurse providing care for the antepartum woman should understand that contraction stress test (CST): a. Sometimes uses vibroacoustic stimulation. b. Is an invasive test; however, contractions are stimulated. c. Is considered negative if no late decelerations are observed with the contractions. d. Is more effective than nonstress test (NST) if the membranes have already been ruptured.

c. Is considered negative if no late decelerations are observed with the contractions.

10. When nurses help their expectant mothers assess the daily fetal movement counts, they should be aware that: a. Alcohol or cigarette smoke can irritate the fetus into greater activity. b. "Kick counts" should be taken every half hour and averaged every 6 hours, with every other 6-hour stretch off. c. The fetal alarm signal should go off when fetal movements stop entirely for 12 hours. d. Obese mothers familiar with their bodies can assess fetal movement as well as average-size women.

c. The fetal alarm signal should go off when fetal movements stop entirely for 12 hours.

14. With regard to amniocentesis, nurses should be aware that: a. Because of new imaging techniques, amniocentesis is now possible in the first trimester. b. Despite the use of ultrasound, complications still occur in the mother or infant in 5% to 10% of cases. c. The shake test, or bubble stability test, is a quick means of determining fetal maturity. d. The presence of meconium in the amniotic fluid is always cause for concern.

c. The shake test, or bubble stability test, is a quick means of determining fetal maturity.

1. A woman arrives at the clinic seeking confirmation that she is pregnant. The following information is obtained: She is 24 years old with a body mass index (BMI) of 17.5. She admits to having used cocaine "several times" during the past year and drinks alcohol occasionally. Her blood pressure (BP) is 108/70 mm Hg, her pulse rate is 72 beats/min, and her respiratory rate is 16 breaths/min. The family history is positive for diabetes mellitus and cancer. Her sister recently gave birth to an infant with a neural tube defect (NTD). Which characteristics place the woman in a high risk category? a. Blood pressure, age, BMI b. Drug/alcohol use, age, family history c. Family history, blood pressure, BMI d. Family history, BMI, drug/alcohol abuse

d. Family history, BMI, drug/alcohol abuse

15. Nurses should be aware of the strengths and limitations of various biochemical assessments during pregnancy, including that: a. Chorionic villus sampling (CVS) is becoming more popular because it provides early diagnosis. b. Maternal serum alpha-fetoprotein (MSAFP) screening is recommended only for women at risk for neural tube defects. c. Percutaneous umbilical blood sampling (PUBS) is one of the triple-marker tests for Down syndrome. d. MSAFP is a screening tool only; it identifies candidates for more definitive procedures.

d. MSAFP is a screening tool only; it identifies candidates for more definitive procedures.

25. The nurse recognizes that a nonstress test (NST) in which two or more fetal heart rate (FHR) accelerations of 15 beats/min or more occur with fetal movement in a 20-minute period is: a. Nonreactive b. Positive c. Negative d. Reactive

d. Reactive

11. In comparing the abdominal and transvaginal methods of ultrasound examination, nurses should explain to their clients that: a. Both require the woman to have a full bladder. b. The abdominal examination is more useful in the first trimester. c. Initially the transvaginal examination can be painful. d. The transvaginal examination allows pelvic anatomy to be evaluated in greater detail.

d. The transvaginal examination allows pelvic anatomy to be evaluated in greater detail.

6. A 40-year-old woman is 10 weeks pregnant. Which diagnostic tool would be appropriate to suggest to her at this time? a. Biophysical profile (BPP) b. Amniocentesis c. Maternal serum alpha-fetoprotein (MSAFP) screening d. Transvaginal ultrasound

d. Transvaginal ultrasound


Set pelajaran terkait

The Ultimate Guide To Marketing Metrics and KPIs

View Set

Chapter 9: Muscles and Muscle Tissue

View Set

Social impact of the industrial revolution

View Set

dev psych learning curve chapter 1-5

View Set

DEFINITION OF STRATEGIC MANAGEMENT

View Set

PSYC210 - Final - Statistical Concepts

View Set